Download as pdf or txt
Download as pdf or txt
You are on page 1of 91

UNIT I

1. SALIENT FEATURES OF THE INDIAN CONSTITUTION (L PUFF DA4S)


Ans. The Indian Constitution is a document having a special legal sanctity which sets out the frame-work and the
principal functions of the organs of the government of a State and declares principles governing the operation of
those organs.
1) It is the lengthiest Constitution in the world
2) Parliamentary form of Government
3) Unique blend of rigidity and flexibility
4) Fundamental rights
5) Fundamental duties
6) Directive Principles of State Policy
7) A federation with strong centralizing tendency
8) Adult suffrage
9) An independent Judiciary
10) A secular State
11) Single citizenship

1) It is the lengthiest Constitution in the world


 The original Constitution had 395 articles, 22 parts and 8 schedules.
 At present there are 448 articles, 25 parts and 12 schedules. 98 Amendments
 This extraordinary bulk (length) is due to various reasons such as:
 It gained experiences from the working of all known Constitutions in the world. It incorporated
the good provisions and avoided the defects. E.g. They took fundamental rights model from
American Constitution, directive principles from Ireland the provision relating to emergencies
from Germany.
 The Indian Constitution lays down the structure not only of the Central Government but also the
States.
 The vastness of the country and peculiar problems relating to the language have added to the
bulk of the Constitution.
 It has a long list of fundamental rights and a number of directive principles.
 The Preamble declares India to be a Sovereign, Socialist, Secular, Democratic and Republic.

2) Parliamentary form of Government


 The Constitution establishes parliamentary form of government both at the Centre and State.
 President is the Constitutional head of the State
 Executive powers are in the Council of members headed by the Prime Minister.
 Council of Ministers are responsible to the lower house elected by the people.
 The Parliamentary form of Government has the following advantages:
 It is a responsible government
 Members accountable to the Legislature.
 Prime Minsters more powerful than the President. He gets 2/3 Majority.
 Prime Minsters can choose any talent and get them elected or nominated.
 Defection removed by appropriate legislature.
 The Parliamentary form of Government has the following disadvantages:
 Multiplicity of political parties

CONSTITUTION | 1
 Evils of defection
 Lack of cohesiveness and leadership in National Parties
 Illiteracy of voters
 Growth of regional parties
 Criminalization of Politics
 Advantages of Presidential Form of Government
 Fixed term
 Choose his team from the country and not just elected members
 No defections and discipline is maintained among political parties

3) Unique blend of rigidity and flexibility


 The Constitution though written is sufficiently flexible.
 It can be amended by special majority of Parliament, only a few provisions need consent of half of the
State Legislatures.
 The fact that Indian Constitution has been amended 98 times in the past proves that it is flexible and
not completely rigid.

4) Fundamental rights
 The incorporation of the formal declaration of fundamental rights in Part III is deemed to be a
distinguishing feature of democratic State.
 These rights are prohibitions against State. The State cannot take away the rights of the citizens
guaranteed in Part III of the Constitution.
 The Supreme Court has the power to grant effective remedies in the nature of habeas corpus,
Mandamus, Prohibitions, Quo Warranto and Certiorari whenever these rights are violated.
 The rights are not absolute. They are subject to restriction.

5) Fundamental duties
 The Constitution (42nd Amendment) Act, 1976 has introduced a code of ten fundamental duties for
citizens.
 It is a call to all citizens to observe basic norms of democratic conduct and behavior.

6) Directive Principles of State Policy


 They are contained in Part IV of the Constitution.
 They set out the aims and objectives to be taken up by the States in the governance of the country.
 These rights are not justifiable. If they are not implemented than no action can be taken against the
State.
 But it is accepted that the idea of welfare State can only be achieved if the State endeavors to
implement them with high sense of moral duty.

7) A federation with strong centralizing tendency


 The Indian Constitution is federal Constitution but with a strong Centre.
 During an Emergency the Union Parliament is empowered to legislate on any subject mentioned it the
State List. It is empowered to give directions to States as to the manner in which it should exercise its
executive power. The financial arrangements between the Centre and States can also be altered by the
Union Government.

CONSTITUTION | 2
8) Adult suffrage
 Every person above the age of 18 has the right to elect representatives for the legislature.
 Every person has right to vote without distinction.

9) An independent Judiciary
 The very existence of a right depends upon the remedy for its enforcement and to safeguard
otherwise it is useless. Maxim “unless there is a remedy, there is no right”.
 We have an independent and impartial judiciary.
 It is the custodian of the rights of citizens.

10) A secular State


 A secular State has no religion of its own as recognized religion of State.
 It treats all religions equally.
 In a secular State the States only regular the relationship between man and man. It is not concerned
with relationship of man with God.
 In the name of religion nothing can be done which is against public order, morality and health.

11) Single citizenship
 There is dual polity. Centre and States, but there is provision for single citizenship for the whole of
India.
 There is no State citizenship.
 All enjoy same rights wherever one is and can move to any place in India.

CONSTITUTION | 3
2. GIVE ADVANTAGES AND DISADVANTAGES OF PARLIAMENTARY VERSUS PRESIDENTIAL SYSTEM.
Ans There are various advantages and disadvantages of the parliamentary form of government compared to the
Presidential system.
 Advantages of Presidential Form of Government
 The President is elected directly by the people.
 He has a fixed term, therefore there is stability.
 The chief executive in a presidential system is relatively free from sectional and party disputes.
 Choose his team from the country and not just elected members without pulls and pressures of
elected representatives.
 No defections and discipline is maintained among political parties.

 Advantages of Parliamentary form of Government.


 The Constitution establishes Parliamentary form of Government both at the Centre and State.
President is the Constitutional head of the State.
 Executive powers in are vested in the Council of members headed by the Prime Minister.
 It is a responsible Government as the Council of Ministers are responsible to the lower house
elected by the people.
 Members accountable to the Legislature.
 Prime Minister is more powerful than the President. He gets 2/3 Majority.
 Prime Minister can choose any talent and get them elected or nominated.
 Disease of defection can be removed by appropriate legislature.

 The Parliamentary form of Government has the following disadvantages


 Multiplicity of Political Parties:
There are too many political parties which has resulted in unstable government.
Earlier this is confined to States, but since 1979 this evil has affected the Central Government
too. 4 national governments have fallen. There are many internal dissentions.

 Evils of Defection:
Very often the member of a political party leave the party and go to other parties for varied
reasons, basically it is for political gains. To cure this evil parliament enacted the 52nd
amendment in 1977 and added 1oth schedule which provides that any member of the legislative
party voluntarily leaves the party or votes against the whip of the party or nominated member
joins other within six months of election he will lose membership of the legislation. But there is
an exception to this that if 1/3 of the members goes out of the party it will be called a split and
not defection and the separated group will be a new entity.

 Lack of cohesiveness and leadership in National Parties:


There is a lack of unity and cohesion among the parties, especially the congress party. This has
weakened the authority of the leader. He is more solving problems of the party than taking care
of the country. He has to satisfy different groups and sections of the party.

 Illiteracy of Voters:
The people are more worried about the local matters rather than national issues. They are easily
swayed by politics of caste, money and religion. The national issues are all together forgotten.

CONSTITUTION | 4
 Growth of Regional Parties:
There has been a vast growth of regional parties arousing parochial and provincial feelings which
has weakened the functioning of the parliamentary form of government. They demand more
autonomy for States and threaten unity and integrity of the nation.

 Criminalization of Politics
It has affected free and fair elections. All parties have connection with criminals. They are even
given tickets. These spread violence and cause many evils such as booth capturing, bogus voting,
murder, etc.

CONSTITUTION | 5
3. DISCUSS THE NATURE OF THE INDIAN CONSTITUTION.
IS THE INDIAN CONSTITUTION FEDERAL?
Ans Indian Constitution
 According to the traditional classification followed by the political scientists, Constitutions are either
unitary or federal.
 In unitary Constitution the powers of the Government are centralized in one government i.e. the Central
Government. The provinces or States are subordinate to the Centre.
 In a federal Constitution there is a division of power between the federal and the State Government and
both and independent in their own spheres.
 Some are of the opinion that the Indian Constitution is quasi-federal and contains more unitary features
than federal. The other view of the framers of the Constitution is that the Indian Constitution is a Federal
Constitution. Dr. Ambedkar, the Chairman of the Drafting committee observed thus, “I think it is agreed
that our Constitution notwithstanding the many provisions which are contained in it whereby the Centre
has been given powers to override the Provinces (States) nonetheless, is a Federal Constitution.
 Essential Characteristics of a Federal Constitution are:
1. Distribution of Power: Federalism means the distribution of the powers of the State among a
number of co-ordinate bodies each originating in and controlled by the Constitution. The basis of such
distribution of powers is that in matters of national importance, in which a uniform policy is desirable
in the interest of the units, authority is entrusted to the Union, and matters of local concern remains
with the States.

2. Supremacy of Constitution: A federal State derives its existence from the Constitution. Hence every
power, executive, legislative or judicial whether it belongs to the nation or to the individual State is
subordinate to and controlled by the Constitution. The Constitution in a federal State constitutes the
supreme law of the land.

3. A Written Constitution: a federal Constitution must almost necessarily be a written Constitution. The
foundations of a federal State are complicated contracts. It will be practically impossible to maintain
the supremacy of the Constitution unless the terms of the Constitution have been reduced to writing.

4. Rigidity: A Constitution which is supreme should also be rigid. In a rigid Constitution the procedure of
amendment is very complicated and difficult. This does not mean that the Constitution should be
legally unalterable. It simply means that the power of amending the Constitution should not remain
exclusively with the Central or State Government.

5. Authority of the Courts: In a federal State the legal supremacy of the Constitution is essential for the
existence of the federal system. It is necessary to maintain the division of powers between Centre and
State Governments. This must be done by some independent and impartial authority above and
beyond the ordinary bodies whether federal or State legislature existing under the Constitution. The
Judiciary in a federal polity has the final power to interpret the Constitution and grant entrenched
provisions of the Constitution.

 The Indian Constitution possesses all the essential characteristics of a federal Constitution mention above.
The Constitution establishes a dual polity, a system of double Government with the Central Government
at one level and the State Government at the other. There is a division of powers between Central and
the State Governments. Each level of Government is supreme in its own sphere. The Constitution of India
is written and it’s supreme. The provisions of the Constitution which are concerned with federal principles

CONSTITUTION | 6
cannot be altered without the consent of the majority of the States. The Supreme Court decides disputes
between the Union and States, or the States inter se interpret finally the provisions of the Constitution.
 But some scholars hesitate to call Indian Constitution truly federal because according to them it is unitary
state with subsidiary federal features rather than a federal state with subsidiary unity features. Jennings
calls it ‘federation with a strong centralizing tendency’. There are various reasons for the same. They are:
a. Appointment of Governors: The Governors of States are appointed by the President (Article 155 and
156) and answerable to him. The Governor is only Constitutional head of the State who shall normally
act on the advice of his Minsters. There are provisions in the Constitution under which the Governor
is required to send certain State laws for the assent of the President. The President has the power to
veto those State laws
b. Parliament’s power to legislate in the national interest: Under Article 249 parliament is empowered
to make laws with respect to every matter enumerated in the State List if the Rajya Sabha passes a
resolution by 2/3 majority that it is necessary in the national interest. There cannot be any objection
to this provision: First, no one will deny that if a subject in the State List assumes national character.
Parliament should make a law on it. In normal course this cannot be done unless the Constitution is
amended. But in this provision we have devised an expedient way by which without formally
amending the Constitution we can achieve the desired effect, namely, the acquisition by the Centre of
the power to administer and legislate upon a subject which assumed national importance. Secondly, it
should also be noted that this power is given to Parliament by the Council of States itself by passing a
resolution supported by 2/3 majority of the members present. Thus, in effect by this devise the
Constitution is amendment by the agreement of majority of the States. We, therefore, fail to
understand how Article 249 places the State in subordinate positions.
c. Parliaments’ power to form new States and alter boundaries of existing States: The Parliament of
India may form new States; it may increase or diminish the area of any State and it may alter the
boundaries or name of any State (Article 3). The very existence of any State thus depends upon the
sweet will of the Union Government.
d. Emergency Provisions: The Constitution envisages there types of emergencies: 1) emergency caused
by war or external aggression or armed rebellion (Article 352), 2) emergency caused by failure of
Constitutional machinery in States (Article 356), and 3) financial emergency (Article 360). During
emergency the normal distribution of powers between Centre and States undergo a vital change and
even suspended. The Parliament is empowered to make laws with respect to any matter enumerated
in the State List. The Centre is empowered to give directions to any State as to manner in which the
State’s executive power is to be exercised.
Therefore, it may be concluded that the Constitution of India is neither purely federal nor purely unitary
but is a combination of both. It is a union of composite State of a novel type. It enshrines the principle
that in spite of federalism, the national interest ought to be paramount. Thus, the Indian Constitution is
mainly federal with unique safeguards for enforcing national unity and growth.

CONSTITUTION | 7
4. WHAT IS PREAMBLE? STATE ITS PURPOSE AND OBJECTIVES WITH A REFERENCE TO INDIAN CONSTITUTION
AND EXAMINE THE AMENDMENT MADE IN ITS PREAMBLE UNDER THE 42ND AMENDMENT OF THE
CONSTITUTION.
Ans. Preamble
 The Preamble to an Act sets out the main objectives which the legislation is intended to achieve.
 It is a sort of introduction to the statute and many a times very helpful to understand the policy and
legislative intent.
 It is a key to open the minds and indicating noble ideals of the ancestors and architects of the
Constitution.
 Words and objects of the Preamble
“We the people of India having solemnly resolved to constitute India into a Sovereign, Socialist, Secular,
Democratic Republic and to secure to all citizens:
Justice – social, economic and political;
Liberty – of thought, expression, belief, faith and worship;
Equality – of status and of opportunity; and to promote among all
Fraternity – assuring the dignity of the Individual and the unity and the integrity of the Nation.
In our Constitution Assembly this twenty-sixth day of November, 1949 do Hereby, Adopt, Enact and Give
ourselves this Constitution.”

 Preamble how far useful in interpreting the Constitution.


The Preamble is the key to open the mind of the makers. But it does not mean that the Preamble can
override the express provisions of the Act.
In case of Berubari the Supreme Court held that the Preamble was not part of the Constitution and
therefore it could never be regarded as a source of any substantive powers. It cannot be considered to be
an important and powerful means as a source of interpretation. Such powers are given in the body of the
Constitution. What is true of power is also true of prohibitions. When there is any ambiguous word in the
Constitution or double meaning then partly the Preamble can be used.
In case of Kesavananda Bharati, the Supreme Court rejected the above view and held that the Preamble
is the part of the Constitution. However, in general no much importance is given to the Preamble,
whereas in the Constitutional statute all importance is attached to the Preamble of the Constitution. In
the opinion of Chief Justice Sikri, “in our Constitution the position of Preamble is having supreme
importance regarding the process of giving the meaning of the legal interpretation in the Constitution and
its reading, keeping in mind the expressed great and noble view point into limelight. In fact, the Preamble
was relied on in imposing implied limitations on the amending power of Parliament under Article 368 of
the Constitution. In that case, it was held that the ‘basic elements’ in the Preamble cannot be amended
under Article 368.
In Randhir Singh v/s Union of India the Supreme Court relying on the Preamble and Article 14 held that
Article 39(a) envisages a Constitutional right of ‘equal pay for equal work’ for both men and women.
 The purpose of the Preamble: The preamble serves the following purposes:
a. It indicates the source from which the Constitution comes i.e. the people of India.
b. It contains the enacting clause which brings the Constitution into force.
c. It declares the great rights and freedoms which the people of India intended to secure to all citizens
and the basic type of Government and polity which was to be established.
 The Preamble speaks of India as Sovereign, Socialist, Secular, Democratic Republic.
- Sovereign means it is absolute and uncontrolled. Having no superior. It thus emphasis that India is no
more dependent upon any outside authority.

CONSTITUTION | 8
- Socialist means collective or Governmental ownership and administration of the means of
production and distribution of goods.
- Secular means that the State does not recognize any religion as State religion.
- Democratic means by the people, for the people. It is a form of Government in which the supreme
power is vested in the people and exercised by them directly or indirectly through a system of
representation usually involving periodically held free and fair elections.
- Republic means the head of the State is not a monarch but person elected for a fixed term. In India it
is the President who is the head of the Executive and who is elected, unlike the Monarch who
become so by hereditary.

 Can the Preamble be amended?


- In case of Kesavananda Bharati v/s State of Kerala, the above question was raised for the first time.
In that case the Attorney General of India argued that by the virtue of the amending power in Article
368 even the Preamble can be amended. It was said that since the Preamble was a part of the
Constitution it could be amended like any other provision of the Constitution.
- The petitioners however contended that amending power of Article 368 is limited. The Preamble
contains the basic elements or the fundamental features of the Constitution. Therefore, amending
power cannot be used so as to damage the basic feature in the Constitution. They said that Preamble
is not part of the Constitution. The Supreme Court however held that the Preamble is the part of the
Constitution and the point of Berubari was wrong. Therefore, the preamble can be amended but the
basic features

 42nd Amendment and the Preamble?


- The amendment has inserted three new words in the Preamble. They are Secularism, Socialism and
Integrity.
- Secularism means no State Religion. It treats all religions equally. It entails the liberty of belief, faith
and worship. All can practice and propagate their own religion. In S. R. Bommai v/s Union of India,
the Supreme Court held that ‘secularism is the basic feature of the Constitution’. In Aruna Roy v/s
Union of India, the Supreme Court has said that secularism has a positive meaning that is developing,
understanding and respect towards different religions.
- Socialism means such system wherein, on majority of the resources of manufacturing there is a
control of State. In India it means ‘democratic socialism’ and not like Communist Socialism. It deals
with ‘economic justice’. The means of production is wholly or partly controlled by the State. It is
based on welfare State which prevents on the excess of exploitation and free competition without
destroying individual initiative and without detriment to the political freedoms. In Excel Wear v/s
Union of India the Supreme Court considered the effect of the word ‘socialism’ in the Preamble. The
court held that the addition of the word ‘socialism’ might enable the courts to lean more in favor of
nationalization and State ownership of an industry. But so long as private ownership of industries is
recognized and governs large proportion of our economic structure, the principles of socialism and
social justice cannot be ignored completely. In D. N. Nakara v/s Union of India, the Supreme Court
held that the basic framework of socialism is to provide a decent standard of life to the working
people and especially provide security from cradle to the grave. This amongst others on economic
side envisaged economic equality and equitable distribution of income. This is blend of Marxism and
Gandhism heavily leaning towards Gandhian Secularism.
- Integrity means national unity and Union. It puts an end to separatist tendencies and make people
feel that every part of India is their home. In federalism of India, this element is encouraged. The
framers have used the words “India shall be a Union of States in Article 1 of the Constitution with a
view to make it clear that the states have no right to secede from the federation. Under Article 19

CONSTITUTION | 9
the States have the power to put reasonable restrictions on speech and expression of the citizens for
the protection of national unity and sovereignty.

CONSTITUTION | 10
UNIT II
5. DISCUSS IN DETAIL EQUALITY BEFORE LAW AND PROTECTION OF LAW (ARTICLE 14) ALONG WITH EXCEPTIONS
TO THE RULE.
Ans Right to Equality is enshrined in Article 14.
 Article 14 declares that the State shall not deny to any person equality before the law or the equal
protection of the laws within the territory of India.
 This article uses two expressions ‘equality before the law’ and ‘equal protection of the law’.
 Both these expressions have been used in the Universal Declaration of Human Rights.
 While both the expressions may seem to be somewhat identical, they do not convey the same meaning.
 One thing is common to both and that is ‘equal justice’.

A. Equality Before the Law


 This expression is of English origin.
 Equality before law is somewhat a negative concept implying the absence of any special privilege by
reason of birth, creed or the like in favor of individuals and also the equal subject of all classes to
the ordinary law.
 Dr. Jennings puts it: equality before the law means that among the equals the law should be equal
and should be equally administered, that like should be treated alike. The right to sue and be sued,
to prosecute and be prosecuted for the same kind of action should be same for all citizens of full
age and understanding without distinctions of race, religion, wealth, social status or political
influence.
 Rule of Law: The guarantee of equality before law is an aspect of what Dicey calls the rule of law in
England. It means that no man is above the law and that every person, whatever be his rank or
condition, is subject to the jurisdiction of ordinary courts. Every official from the Prime Minister to
the constable or a Collector of taxes is under the same responsibility for every act done without
legal justification as any other citizen. Rule of Law also means that no person shall be subjected to
harsh, uncivilized or discriminatory treatment even when the object is the securing of the
paramount exigencies of law and order.
There are three meanings as given by Professor Dicey:
a. Supremacy of the law and absence of arbitrary power.
b. Equality before the law
c. The Constitution is the result of the ordinary law of the land.
The first two apply to Indian System not the third. The Constitution is the Supreme Law of the land
its source is the rights of individuals. All the laws passed by the legislature must be consistent with
the provisions of the Constitution.

B. Equal protection of the Law


 Equal protection of the law is a more positive concept implying equality of treatment in equal
circumstances. In Basawaraj v/s the Special Land Acquisition Officer was a case delay of 5 and half
years in filing the appeals before the Karnataka High Court which dismissed the appeals under
Section 54 of Limitations Act. The appellant quoted a large number of cases of the High Court
wherein the delay had been condoned without considering the most relevant factor i.e. sufficient
cause. The Supreme Court dismissed the appeal and held: Article 14 is not meant to perpetuate
illegality or fraud, even be wrong decisions made in other cases. The said provision does not
envisage negative equality but has only a positive aspect. Thus, if some other similarly situated
persons have been granted relief/benefit inadvertently or by mistake, such an order does not

CONSTITUTION | 11
confer any legal right on others to get the same relief as well. If a wrong is committed in any earlier
case, it cannot be perpetuated. Equality is a trite, which cannot be claimed in illegality and
therefore, cannot be enforced by a citizen or court in a negative manner.
 It means that all persons similarly circumstanced shall be treated alike both in the privileges
conferred and liabilities imposed by the law. Equal law should be applied to all in the same
situation, and there should be no discrimination between one person and another. As regards the
subject-matter of the legislation their position is the same. Thus, the rule is that like should be
treated alike not that unlike should be treated alike.
 The rule of law imposes a duty upon the State to take special measures to prevent and punish
brutality by police methodology. The Rule of Law embodied in Article 14 is the ‘basic feature’ of the
Indian Constitution and hence it cannot be destroyed even by an amendment of the Constitution
under Article 368 of the Constitution.
 The word ‘any person’ includes all person which includes citizens and non-citizens and to natural
and legal persons. The equality before the law is guaranteed to all without regard to race, colour,
and nationality.

Exceptions to the rule of law:


 The rule of equality is not an absolute rule and there are number of exceptions to it:
1. Equality before the law does not mean that the ‘powers of the private citizens are the same as the
powers of the public officials’. The police officer has the power to arrest, but a private citizen does
not have and if the police officer violates the rule of law he is punished by ordinary courts as any
illegal act done by private person.
2. The rule of law does not prevent certain classes of persons being subject to special rules. Thus the
members of armed forces are controlled by Military laws. Medical practitioners are subject to Rules
and regulations of the Medical Council of India and are immune from the jurisdiction of ordinary
courts. Article 361 gives immunity to the President of India and the State Governors. They are not
answerable to any Court for the exercise and performance of the powers and duties of the office or
for any act done or purporting to be done by him in the exercise and performance of those powers
and duties. No criminal proceedings against them in any Court during their term in office.
3. Ministers and other executive bodies are given very wide discretionary powers by statute. A
Minister may be allowed by law ‘to act as he thinks fit’ or ‘if he is satisfied’. Such power is
sometimes abused.
4. Certain members of society are governed by special rules in their profession such as lawyers,
doctors, nurses, members of armed forces, and police. Such classes of people are treated differently
from ordinary citizens.
 The Constitution itself contains provisions, which under certain circumstances, limit the effectiveness of
Article 14. They are:
1. The scope of right to equality under Article 14 has been considerably restricted by the 42nd
Amendment Act, 1976. The new Article 31 – C added by the Amendment Act provides that laws
made by the State for implementing the Directive Principles contained in clause (b) or clause (c) of
Article 39 cannot be challenged on the ground that they are violative of Article 14. Such laws will be
an exception to article 14 of the Constitution. In Sanjeev Coke Mfg. Co. v/s Bharat Cooking Coal
Ltd. the Supreme Court held that ‘where Article 31-C comes in, Article 14 goes out’.
2. Article 359(1) provides that where a proclamation of emergency is in operation the President, may
by order, declare that the right to move any court for the enforcement of such rights conferred by
Part III (except Articles 20 and 21) shall remain suspended. Thus, if the President of India issues an
order, where a Proclamation of Emergency is in operation, enforcement of Article 14 may be
suspended for the period during which the Proclamation is in force.

CONSTITUTION | 12
3. Article 361 lays down that the President and Governors are exempted from any criminal
proceedings during the tenure of their office.
4. Under International law, foreign sovereign and ambassadors enjoy full immunity from any judicial
process. This is also available to enemy aliens for acts of war.

C. OTHER PROVISIONS UNDER ARTICLE 14


1. Article 14 permits classification but prohibits class legislation.
- The equal protection of laws guaranteed by Article 14 does not mean that all laws must be
general in character. It does not mean that the same law should apply to all persons. It does not
mean that the law should have universal application to all persons.
- Different persons and classes have different needs and require different treatment. So a
reasonable classification is not only permitted but is necessary if society is to progress.
- Article 14 forbids class legislation but does not forbid reasonable classification.
- The classification should not be ‘artificial, arbitrary, or evasive’. It must be based on some real
and substantive distinction bearing a just and reasonable relation to the object sought to be
achieved by the legislation.
- Test of classification is based on two conditions. They are:
a. The classification must be founded on an intelligible differentia which distinguishes person
or things that are grouped together from others left out of the group.
b. The differentia must have a rational relation to the object sought to be achieved by the Act.
The differentia which is the basis of the classification and the object of the Act are two different
things. What is necessary is that there is connection between the two.
- In re Special Court Bill case, justice Chandrachud reformulated new propositions to be followed
regarding the applicability of Article 14. This has been criticized by Mr. Seervai as making the
well-settled principles unsettled and creating confusion and uncertainty and encouraging
litigation.

2. Article 14: New Concept of Equality: Protection against Arbitrariness.


- In E. P Royappa v/s State of Tamil Nadu, the Supreme Court has drifted from the traditional
concept of equality which was based on reasonable classification.
- Justice Bhagwati explains the new concept of equality as: “Equality is a dynamic concept with
many aspects and dimensions and it cannot be ‘cribbed, cabined, confined’ within traditional
and doctrinaire limits. From positive point of view equality is antithesis (opposite) to
arbitrariness. They are sworn enemies, one belongs to rule of law and the other to whim and
fancies of an absolute monarch.
- This was again propounded (proposed) in Maneka Gandhi v/s Union of India. Justice Bhagwati
added to the above the following ‘Article 14 strikes at the root of arbitrariness in State action
and ensures fairness and equality of treatment. The principle of reasonableness, is essential
element of equality or non-arbitrariness.
- In International Airport Authority case Justice Bhagwati reiterated the same principle in the
following words: “arbitrariness involves negation of equality. If the classification is not
reasonable and does not satisfy the two conditions referred above, then it would be arbitrary
and the guarantee of equality under Article 14 would be breached”.
- In D. S. Nakara v/s Union of India, the Supreme Court struck down the Rule 34 of Central
Services (Pension) Rules, 1972 as unconstitutional on the ground that classification made in
between pensioners retiring before particular date and retiring after that date was not rational
principle and was arbitrary and violative of Article 14.

CONSTITUTION | 13
- In Suneel Jatley v/s State of Haryana, the reservation of 25 seats for admission to M.B.B.S and
B.D.S. course for students who were educated from classes I to VIII in common rural schools was
held to be violative of Article 14 and invalid as the classification between the rural and urban
educated students of this purpose was wholly arbitrary and irrational having no nexus to the
object sought to be achieved of providing extra facilities to students coming from rural schools
to enter medical college.
- In Ajay Hasia v/s Khalid Mujib, the Regional Engineering College made admissions of candidates
on the basis of oral interview after a written test. The test of oral interview was challenged on
the ground that it was arbitrary and unreasonable because high percentage of marks were
allocated for oral test and candidates were interviewed for 3 or 3 minutes. The court held that
oral interview cannot be regarded a very satisfactory test for assessing and evaluating capacity
and caliber of candidates and it is subjective and capable of abuse.
- U and A category films in K. A. Abbas v/s Union of India, the validity of cinematograph Act, 1952
was challenged on the ground that it was unreasonable classification. Under this Act films were
to be of two categories A and U. A films was for those who are adults and U was open to all. It
was argued that motion pictures as a form of expression was entitled to equal treatment with
other forms of expression. The court held that motion pictures must be different from that of
other forms of art and expression due to its versatility, realism, and combination of visual and
real senses. The motion pictures have the capacity to stir up emotions more deeply than any
other product of art. Its effect on children and adolescents is very great since their immaturity
makes them more willingly suspend their disbelief than adults. They imitate the same.

3. AGE OF RETIREMENT AND PREGNANCY BAR OF AIR HOSTESS.


- In Air India v/s Nargesh Meerza, the Supreme Court struck down the Air India and Indian
Airlines Regulations on the age of retirement and pregnancy bar on the services of the Air
Hostesses as unconstitutional on the ground that they conditions laid down therein were
entirely unreasonable and arbitrary. The rule (46) was that the air hostess would retire from the
service upon attaining the age of 35 years or on marriage, if it took place within four years of
service or on first pregnancy, whichever occurred earlier. Under section 47 the Managing
Director had the discretion to extent the age of retirement by one year at the time beyond the
age of retirement upto the age of 45 years if an air hostess was found medically fit. The
termination on first pregnancy was unreasonable and arbitrary. The power of managing Director
too was seen as unconstitutional and the persons were completely at his mercy and the sweet
will. It could be abused.

4. WHOLESOME RESERVATION ON THE BASIS OF DOMICILE OR INSTITUTIONAL PREFERENCE.


- In Pradeep Jain v/s Union of India, the Supreme Court held that the wholesome reservation (all
seats) in MBBS and BDS made by State of Karnataka, UP and Delhi on the basis of domicile or
residence within the State and excluding those due to non-residence regardless of merit, was
unconstitutional being violative of Article 14 of the Constitution. The criterion should be to
choose the best students for the admission to medical college. Residence doesn’t matter. The
rule should be merit based selection. The departure from this rule can be justified only on the
grounds of regions claim of backwardness and State need to provide medical service to its
people. The quantum of reservation for MBBS should not exceed 70% and also to be reviewed
every three years if it needs to be reduced. MS and MD courses should not be based on
residential requirement at all. Their excellence cannot be reduced or compromised. Yet the
court in keeping the need of equality of an opportunity directed that 50% of seats might be

CONSTITUTION | 14
reserved on basis of institutional preference for admission to the post graduate course. This too
will have to be reviewed.

CONSTITUTION | 15
6. VISHAKA V/S STATE OF RAJASTHAN
Ans In Vishaka v/s State of Rajasthan case the judgment delivered by J. S. Verma CJ, on behalf of Sujata Manohar
and B. N. Kirpal, JJ., on a writ petition filed by “Vishaka” – a non-governmental organization working for gender
equality by way of PIL seeking enforcement of fundamental rights of working women under Article 21 of the
Constitution. The immediate cause was the alleged gang rape of a social worker of Rajasthan. They sought
enforcement of Fundamental Rights of working women under Articles 14, 19, 21 of the Constitution.

The Supreme Court has laid down the following guidelines under Article 141 of the Constitution to prevent
sexual harassment of working women in the place of their work until legislation is enacted for the purpose.

It stated that it is necessary and expedient for employers in work places as well as other responsible persons or
institution to observe certain guidelines in insure the prevention of sexual harassment. It shall be the duty of the
employer or any other responsible person in work places or other institution to prevent or direct the
commission of acts of sexual harassment by taking all steps required. These guidelines will not prejudice any
rights available under the Protection of Human Rights Act, 1993

1. Preventive Steps - All employers, person’s in charge of work place, whether in the public or private sector,
should take appropriate steps to prevent sexual harassment without prejudice to the generality of her
obligation: he should take the following steps –
(i) Express prohibition of sexual harassment which includes physical contact and advances, a demand or
request for sexual favors, sexually colored remarks, showing pornography or other unwelcomed
physical, verbal or non-verbal conduct of sexual nature at work place, should be notified, published
and circulated in appropriate ways.
(ii) The Rules/Regulations of the Government and Public Sector bodies relating to conduct and discipline
should include Rules/ Regulations prohibiting sexual harassment and provide for appropriate
penalties in such rules against the offender.
(iii) As regards private employers, steps should be taken to include the aforesaid prohibitions in the
standing orders under the Industrial Employment (Standing orders) Act, 1946.
(iv) Appropriate work conditions should be provided in respect of work, leisure, health and hygiene to
further ensure that there is no hostile environment towards women at work place and no employee
woman should have reasonable grounds to believe that she is disadvantaged in connection with her
employment.

2. Criminal Proceeding - Where such conduct amounts to specific offence under the IPC or under any other
law, the employer shall initiate appropriate action in accordance with law by making a complaint with
appropriate authority.

3. Worker’s Initiative - Employees should be allowed to raise issues of sexual harassment at worker‘s meeting
and in other appropriate forum and it should be affirmatively discussed in Employer- Employee meeting.
The victims of sexual harassment should have the option to seek transfer of the perpetrator or their own
transfer.

4. Disciplinary Action - Where such conducts amounts to misconduct in employment as defined by the
relevant service rules, appropriate disciplinary action should be initiated by the employer in accordance
with those rules.

5. Complaint Mechanism - Whether or not such conduct constitutes an offence under law or a breach of the
service rules, an appropriate complaint mechanism should be created in the employer‘s organization for

CONSTITUTION | 16
redress of the complaint made by the victim. Such complaint Mechanism should ensure time bound
treatment of the complaints.

6. Complaints Committee - The Complaint mechanism should be adequate to provide, where necessary, a
complaints committee, a social counsellor or other support service, including the maintenance of
confidentiality. The complaints Committee should be headed by a woman and not less than half of its
member should be women. Further to prevent the possibility of any undue pressure or influence from
senior levels, such complaints committee should involve a third party, either NGO or other body who is
familiar with the issue of sexual harassment.

7. Awareness - Awareness of the right of female employees in this regard should be created in particular by
prominently notifying the guidelines in a suitable manner.

8. Third Party Harassment - Where Sexual harassment occurs as a result of an act or omission by any third
party or outsider the employer and person in charge will take all steps necessary and reasonable to assist
the affected person in terms of support and preventive action.

9. Duties of Government - The Central/State Government are requested to consider adopting suitable
measures including legislation to ensure that the guidelines laid down by this order are also observed by
the employers in private sectors.

10. These Guidelines are enforceable as law till legislation is enacted - The Honourable Supreme Court held
that these guidelines and norms would be strictly observed in all works places for the preservation and
enforcement of the right to gender equality of the working women. These directions would be binding and
enforceable in law until suitable legislation is enacted to occupy the field.

“The Fundamental Rights to carry on any occupation, trade or profession depends on the availability of a safe
working condition. Right to life means life with dignity. The primary responsibility for ensuring such safe and
dignity through suitable legislation, and the creation of the mechanism for its enforcement, the responsibility of
the Legislature and Executive. When, however instances of sexual harassment resulting in violation of
Fundamental Rights of women workers under Articles 14, 19, 21 are brought before the court for redressal
under Article 32 of the Constitution ineffective redressal requires that some guidelines should be laid down for
the protection of these rights to fill the legislative vacuum,” Mr. Justice Verma said.

CONSTITUTION | 17
7. SHORT NOTES ON NATIONAL LEGAL SERVICE AUTHORITY V/S UNION OF INDIA
Ans National Legal Service Authority v/s Union of India talks of Hijras and Transgenders are the persons entitled to
legal protection under Article 14.

 The Supreme Court has held that Article 14 does not restrict the word ‘person’ and its application only to
male or female and Hijras/Transgender persons who are neither male nor female fall within the expression
‘person’.

 They are entitled to legal protection of laws in all spheres of State activity including employment, heath
care, education as well as equal civil citizenship rights, as enjoyed by any other citizen of this country.

 The expression ‘sex’ under Article 15 and 16 therefore, includes discrimination on the ground of gender
identity. The expression ‘sex’ is not limited to biological sex of male or female but intended to include
people who consider themselves to be neither male nor female. Transgenders are denied rights under
Section 16 (2) and discriminated against in respect of employment or office under the State on the ground
of sex.

 They are also entitled to reservation in matter of appointment under Article 16 (4). State is bound to take
affirmative action to give them due representation in public services.

 Article 19 (1) (a) a transgender has freedom to express ones’ choses gender identity through varied ways
and means by ways of expression, speech, mannerism, clothing, etc. Values of privacy, self-identity,
autonomy and personal integrity are Fundamental Rights guaranteed to the transgenders under Article 19
(1)(a) and State is bound to protect and recognize those rights.

 Self-determination of gender is an integral part of personal autonomy and self-expression and falls within
the realm of personal liberty guaranteed under Article 21 of the Constitution.

 The Court made the following declaration:


1. Hijras, Eunuchs, apart from binary gender, be treated as ‘third gender’ for the purpose of safeguarding
their rights under Part III of our Constitution and the laws made by the Parliament and State
Legislature.
2. Transgender person’s right to decide their self-identity gender is also upheld and the Centre and State
Government are directed to grant legal recognition of their gender identity such as male, female or as
third gender.
3. We direct the Centre and the State Government to take steps to treat them as socially and
educationally backward classes of citizens and extend all kinds of reservation in cases of admission in
educational institutions and for public appointments.
4. Centre and State Government are directed to operate separate HIV Sero-surveillance Centres since
Hijras/transgender face several sexual health issues.
5. Centre and State Government should seriously address the problems being faced by
Hijras/Transgenders such as fear, shame, gender dysphoria, social pressures, depression, suicide
tendencies, social stigma, etc
6. Centre and State Government should take proper measures to provide medical care to TGs in the
hospitals and also provide them separate them separate public toilets and other facilities.

CONSTITUTION | 18
7. Centre and State Government should also take steps for framing various social welfare schemes for
their betterment.
8. Centre and State Government should take steps to create public awareness so that TGs will feel that
they are also part and parcel of the social life and be not treated as untouchables.
9. Centre and State Government should also take measures to regain their respect and place in the
society which once they enjoyed in our cultural and social life.

CONSTITUTION | 19
8. WRITE SHORT NOTES ON THE MANDAL CASE – INDRA SAWHNEY V/S UNION OF INDIA
Ans Section 16 (4) of the Constitution enables the state to make provisions for the reservation of posts in
government jobs in favor of any backward class of citizens which, in the opinion of the State is not adequately
represented in the service of the State. It makes provisions for SC and ST for the same.
 The scope and extent of Article 16 (4) has been examined thoroughly by the Supreme Court in the historic
case of Indra Sawhney v/s Union of India, popularly known as Mandal Case.
 The facts of the case are: On January 1, 1979 the Government headed by the Prime Minster Shri Morarji
Desai appointed the second Backward Classes Commission under Article 340 of the Constitution under the
chairmanship of Shri B. P. Mandal to investigate the socially and educationally backward classes within the
territory of India and recommend steps to be taken for their advancement, including making provisions for
reservation of seats for them in Government jobs. The commission recommended 27% Government jobs
for them.
 Before this could be implemented the Janata Party Government collapsed and the congress came to power
under Mrs. Indira Gandhi. They did not implement the Mandal case report. Later the congress lost in
parliament elections and Mr. V. P. Singh came to power. He decided to implement it. He issued a Office
Memoranda on August 13, 1990 reserving 27% for them as per the Mandal commission report. Due to this
there was turmoil in the country and a violent anti-reservation movement rocked. A writ petition was filed
in the Supreme Court challenging the validity of Office Memoranda. The Janata Government again
collapsed and the congress came to power again.
 The Congress under the leadership of Shri. P. V. Narsimha Rao issued another Office Memoranda on
September 25, 1991 but made two changes by introducing economic criterion granting reservation by
giving preference to poorest section of SEBC in 27% quota and reserved another 10% of vacancies for other
socially and educationally backward classes (SEBC) economically backward sections of higher castes.
 This was referred to 9 Constitution bench of the Supreme Court by 6:3 majority they held the decision to
reserve 27% Government jobs for backward classes is constitutionally valid provided socially advanced
persons – creamy layer among them – are eliminated is constitutionally valid. The reservations are only for
initial appointment and not for promotions and total reservation shall not exceed 50%. The court struck
down the congresses demand for 10% for economically backward classes among higher castes.
 The court examined the scope and extent of article 16(4) in detail and clarified various aspects on which
there were differences of opinion in various earlier judgments. The majority opinions of the Supreme Court
are as follows:
i. Backward class of citizens in Article 16 (4) can be identified on the basis of caste and not only on
economic basis but caste alone cannot be the basis for consideration.
- Caste alone cannot be basis for consideration as there are among non-Hindus, Muslims, Christians,
and Sikhs and if they are socially backward, they are entitled for reservation under Article 16(4).
- The authority must identify the backward people. It can be done on basis of caste, class, occupation
groups, etc.
- The court struck down the economic criterion for reservation on the ground that Article 16(4) does
not mention it.
ii. Article 16 (4) is not an exception to Article 16 (1). It is an instance of classification. Reservation can be
made under Article 16 (1).
- Reservation can be made under clause (1) of article 16 on the basis of reasonable Classification.
- It is also based on equality enshrined in Article 14 and permits reasonable classification.
iii. Backward classes in Article 16 (4) are not similar to as socially and educationally backward in Article
15(4).

CONSTITUTION | 20
- It is much wider. It takes in SC’s and ST’s and all other backward classes of citizens including the
socially and educationally backward classes. Thus certain classes may not qualify for Article 15(4)
but they may qualify for Article 16(4).
iv. Creamy layer must be excluded from backward classes.
- The majority held that while identifying the backward classes, the socially advanced persons which
is the creamy layer among them should be excluded.
- The court directed that the basis of exclusion of advanced section i.e. creamy layer, from other
backward classes for the purpose of reservation should not be merely economic unless the
economic advancement is so high that it necessarily means social advancement. Income of a person
can be taken as a measure of his social advancement.
v. Article 16(4) permits classification of backward classes into backward and more backward classes.
- The sub-classification between backward classes is necessary to help the more backward classes,
otherwise the advanced sections of backward classes might take all the benefits of reservation.
vi. A backward class of citizens cannot be identified only and exclusively with reference to economic
criteria.
- To identify backward classes exclusively with reference to economic criterion would defeat the very
object of Article 16(4) to give adequate representation to backward classes in the services.
- Article 16(4) is not aimed only at economic upliftment or alleviation of poverty. It is meant to give a
due share in the state power to those who have remained out of it mainly on account of their social
and therefore, educational and economic backwardness.

vii. Reservations shall not exceed 50%.


- The maximum limit of reservation shall not exceed 50%.
- In extra-ordinary situations it can be relaxed in favor of people living in far and remote and far flung
areas of country who require different treatment.
- Article 16(4) speaks of adequate treatment and not proportionate representation. If a SC/ST is
selected on the basis of merit, he will not be counted against the reservation quota.

viii. Reservation can be made by ‘Executive Order’.


- It needs to be made by Parliament or Legislature.

ix. No reservation in promotions.


- The majority agreed that there can be no reservation in promotions under section 16(4).
- The reservation is only for initial appointments. Once they are appointed, efficiency demands, that
these members too compete with others and earn promotions like all others.

x. Permanent Statutory body to examine complaints of over-inclusion/under-inclusion.


- The Supreme Court directed the Union Government, State Governments and Union Territories to
appoint a permanent statutory body to examine complaints of wrong inclusion or non-inclusion of
groups in list of other backward classes.

xi. Mandal Commission Report – No opinion Expressed.


- The Court in view of the guidelines laid down in its decision held not need to express any opinion
on the correctness or adequacy of the exercise done by the Mandal Commission.
- It was held the report valid and recommended the appointment of a fresh Commission for
identifying the backward classes.

CONSTITUTION | 21
xii. Disputes regarding new criteria can be raised only in the Supreme Court.
- Disputes would be taken up only in the Supreme Court and not before any High Court or tribunal.
- Similarly a petition challenging the validity, operation or implementation of the two OM’s shall be
filed only before the Supreme Court.

 Position after Indra Sawhney Case


- The parliament enacted the Constitution 77th Amendment Act, 1995 in order to bypass the Court’s ruling
on the point of no reservation in promotion in government service.
- This amendment has added a new clause (4-A) to Article 16 of the Constitution which provides that
“Nothing in this article shall prevent the State from making any provision for reservation in matters of
promotion to any class of classes of posts in the service of the State in favaour of the SC/ST which in the
opinion of the State, are not adequately represented in the service under the State”.
- Thus the reservation in promotion in Government jobs will continue in favour of SC/ST even after the
verdict of Indra Sawhney case if the government wants to do so.

CONSTITUTION | 22
UNIT III

9. Freedoms guaranteed to a citizen of India under article 19and reasonable restriction provision laid down in it.
Ans Articles 19 contain the six freedoms.
 Personal Liberty is the most important of all fundamental rights. Article 19 deal with different aspects of
this basic right.
 There are six Fundamental Rights in the nature of freedoms. They are:
1. Freedom of Speech and Expression
2. Freedom of Assembly
3. Freedom to form Association or Unions or Co-operatives
4. Freedom of movement
5. Freedom to reside and to settle
6. Freedom of profession, occupation, trade or business.

 These six freedoms are however, not absolute. There cannot be any right which is injurious to the
community as a whole. If people are given complete and absolute liberty without any social control the
result would be ruined. Liberty of one should not offend the liberty of others. Pantajali Shastri J., in A. K.
Gopalan case observed, ‘man as a rational being desires to do many things, but in a civil society his desires
have to be controlled, regulated and reconciled with the exercise of similar desires by other individuals.’
The restrictions are provided in clauses 2 to 6 of Article 19 of the Constitution.

 The restriction must be reasonable restrictions.

Test of Reasonable Restriction:


- The restriction on the rights under Article 19(1) can only be imposed by ‘Law’ and not executive or
departmental instructions.
- The Court is to determine whether any restriction is reasonable or not. If it is not reasonable then the
court can declare it void. However there is no absolute test to judge the reasonableness of a
restriction. The Supreme Court has laid down the following guidelines for determining the
reasonableness of restrictions. They are:
1. It is the Courts and not the Legislature which has to judge finally whether a restriction is
reasonable or not.
2. The term ‘reasonable restriction’ in Article 19 (6) connotes that the limitation imposed on a person
should not be arbitrary or of an excessive nature, beyond what is required in the interests of the
public. The word ‘reasonable’ implies intelligent care and deliberation which reason dictates. It
must be wanting in quality. There should be a balance between the individuals rights guaranteed
by Article 19 and social control permitted by clause (2) and (6) of Article 19. The restriction must
have a reasonable relation with the object which the legislation seeks to achieve and must never
exceed it.
3. There is no exact standard or general pattern of reasonableness that can be laid down for all cases.
Each case is to be judged by its merits. The standard varies with the nature of the right infringed,
the underlying purpose of the restriction imposed, the extent and the urgency of the evil sought to
be remedied, the disposition, the prevailing condition at the time.
4. The restriction must be reasonable form the substantive as well as procedural stand-point. The
court should consider not only the duration and extent of the restriction but also the
circumstances under which, and the manner in which that imposition has been authorized.

CONSTITUTION | 23
5. A restriction which is imposed for securing the objects and laid down in the Directive Principles of
State Policy may be regarded as reasonable restriction.
6. The court must determine the reasonableness of a restriction by objective standard and not by
subjective one. It is based on whether a normal reasonable man would regard the restriction as
reasonable.
7. A restriction to be reasonable must have a rational relation with the object which the legislature
seeks to achieve and must not be in excess of that object. The grounds for such restrictions are
mentioned in clause (2) to (6) of Article 19.
8. The Court determines the reasonableness of restrictions and not the reasonableness of the law.
9. Restrictions may also amount to prohibition under certain circumstances.

 Article 19 (2) which empowers the State to put ‘reasonable restrictions’ on the following grounds –
a) Security of State,
b) Friendly Relations with Foreign States,
c) Public Order,
d) Decency and Morality,
e) Contempt of Court,
f) Defamation,
g) Incitement to Offence and
h) Integrity and Sovereignty of India.

a) Security of State: in Romesh Thapper v/s State of Madras the Supreme Court has interpreted the
meaning of Security of State. It is not just offence against public order. Every public disorder is not
amount to threaten the security of State. The security of State refers only to serious and
aggravated forms of public disorder e.g. rebellion, waging war against State, assembly riot, affray,
armed revolution is sufficient to attract the term security of State.

b) Friendly Relations with Foreign States: The object is to prohibit unrestrained malicious
propaganda against a foreign friendly State which may jeopardize the maintenance of good
relation between India and that country. Fair criticism of foreign criticism policy of the
Government is however allowed. The Commonwealth nations are not foreign States.

c) Public Order: In Romesh Thappar Case it was held that ordinary or local breaches of public order
were of ground for imposing restriction on the freedom of speech, expression guaranteed by the
Constitution. The Supreme Court said that ‘public order’ is an expression of wide connotation and
signifies ‘that state of tranquility which prevails among the members of political society as a result
of internal regulations enforced by the Government which they have established.

d) Decency and Morality: The words ‘morality or decency’ are words of wide meaning. The
‘obscenity’ of English law is identical with the word ‘indecency’ under the Indian Constitution. The
test of obscenity is ‘whether the tendency of matter charged as obscene is to deprave and corrupt
those whose minds are open to such immoral influences’ and into whose hands a publication of
this sort is likely to fall. This test was laid down in an English case of R. Hicklin (Publication of
Obscene books).

e) Contempt of Court: According to section 2 Contempt of Court Act, 1971 defines the expression
as: it may be civil or criminal contempt. Civil contempt means willful disobedience to any
judgment, decree, direction, order writ or other process of a court or willful breach of an

CONSTITUTION | 24
undertaking given to a court. Criminal contempt means the publication (whether by words or
actions, or sings or visible representation or otherwise) or any matter or the doing of any other
act whatsoever which – scandalizes or tends to scandalize, or lowers or tends to lower the
authority of any court; prejudices or interferers or tends to interfere with the due course of any
judicial proceedings or administration of justice. These are not contempt of court: innocent
publication and distributing of matter; publication of fair and accurate report of judicial
proceedings; fair criticism of judicial act.

f) Defamation: It is a statement which injures a man’s reputation amounts to defamation.


Defamation consists in exposing a man to hatred, ridicule or contempt. In India Section 499 of IPC
recognizes defamation as slander and libel.

g) Incitement to Offence: Freedom of Speech and expression cannot confer a license to incite
people to commit offence i.e. anything punishable by any law.

h) Integrity and Sovereignty of India: It denies anyone from challenging the integrity of India or to
preach cession of any part of India from the Union.

i) Sedition: In India, Section 124-A of the IPC defines the offence of sedition as follows: Whoever by
words either spoken or written, or by signs, or by visible representation or otherwise brings or
attempts to bring into hatred or contempt or excite or attempts to excite disaffection towards the
Government established by law in India shall be punished. On the grounds of sedition a person
can be restricted on freedom of speech and expression.

1. FREEDOM OF SPEECH AND EXPRESSION


 Freedom of Speech and Expression are available in Article 19 (1) and 19 (2).
 Freedom of Speech and Expression is indispensable in a democracy.
 Article 19 (1) (a) says that all Citizens shall have the right to freedom of speech and expression, but
this right is subject to limitations imposed under Article 19 (2) which empowers the State to put
‘reasonable restrictions’ on the following grounds – security of State, friendly relations with foreign
States, public order, decency and morality, contempt of court, defamation, incitement to offence
and integrity and sovereignty of India.
 Meaning and Scope of this Article:
- Freedom of Speech and Expression means the right to express one’s own convictions and
opinions freely by words of mouth, writing, printing, pictures or any other mode.
- It includes expression of one’s ideas through any communicable medium or visible
representation such as gesture, signs, and the like.
- It also includes publication and thus the freedom of press is included in this category. Free
propagation of ideas is the necessary objective and this may be done on the platform or
through the press. The freedom of propagation of ideas is secured by freedom of circulation.
Liberty of Circulation is essential to that freedom as they liberty of publication. Indeed without
circulation the publication would be of little value.
- The freedom of speech and expression includes liberty to propagate not only one’s views but
also the right to propagate or publish the views of others.
- It includes a person’s Fundamental right to know.
 Special Purposes of Freedom of expression:
 It helps an individual, to attain self-fulfillment;

CONSTITUTION | 25
 It assists in the discovery of truth;
 It strengthens the capacity of an individual in participating in decision making
 It provides a mechanism by which it would be possible to establish a reasonable balance
between stability and social change.
 Various rights ensured by Article 19 (1):
a) FREEDOM OF PRESS
- As mentioned above the freedom to speech and expression also includes publication
and thus the freedom of press is included in this category. Free propagation of ideas is
the necessary objective and this may be done on the platform or through the press.
The freedom of propagation of ideas is secured by freedom of circulation. Liberty of
Circulation is essential to that freedom as they liberty of publication. Indeed without
circulation the publication would be of little value.
- The freedom of speech and expression includes liberty to propagate not only one’s
views but also the right to propagate or publish the views of others.
- Freedom of Press is essential for political liberty and proper functioning of democracy.
The American Press Commission has said, ‘Freedom of press is essential to political
liberty. When men cannot freely convey their thoughts to one another, no freedom is
secured, where freedom of expression exists the beginning of a free society and means
for every retention of liberty are already present. Free expression is therefore, unique
among liberties’. The Indian Press Commission too has also expressed a similar view. It
says that, ‘Democracy can thrive not only under the vigilant eye of the Legislature, but
also the care and guidance of public opinion and the press is par excellence, the
vehicle through which opinion can become articulate’.
- The liberty of press means liberty to print and publish what one pleases, without
previous permission. The freedom of press is not confined to newspapers and
periodicals. It includes also pamphlets and circulars and every sort of publication which
affords a vehicle of information and opinion.
- Cases
The utility of freedom of press: In Indian Express Newspaper v/s Union of India,
speaking of the utility of freedom of press the court observed: ‘The expression of
freedom of press has not been used in Article 19 but it is comprehended within Article
19 (1)(a). The expression means freedom from interference from authority which
would have the effect of interference with the content and circulation of newspapers.
There cannot be any interference with that freedom in the name of public interest.
The purpose of the press is to advance the public interest by publishing facts and
opinions without which a democratic electorate cannot make responsible judgments.
Freedom of the press is the heart of social and political inter-course. It is the primary
duty of the courts to uphold the freedom of the press and invalidate all laws or
administrative actions which interfere with it contrary to the Constitution mandate’.

Publication of sexually exploitative material: In Ajai Goswani v/s Union of India, a


petition was filed to seek protection from the Court to ensure that minors are not
exposed to sexually exploitative materials, whether or not the same was obscene and
prohibited by the law. Where art and obscenity are mixed, what must be seen is
whether the artistic, literary or social merit of the work in question outweighs its
‘obscene’ content. The test for judging a work should be that an ordinary man of
common sense of prudence and not an out of the ordinary or hypersensitive man. The
blanket ban on publication of obscene materials or article in order to shield juvenile

CONSTITUTION | 26
innocence cannot be imposed. No news item should be viewed in isolation. Publication
must be judged as a whole. Fictitious imagination of anybody especially of minors
should not be agitated in Court of Law. However, the Court suggested the Press
Council to amend provisions of Act.

Prior-restraint on publication of defamatory material against its officials: Auto –


Shankar Case: in R. Rajagopal v/s State of Tamil Nadu., the Supreme Court has held –
the Government has no authority in law to impose a prior-restraint upon publication of
defamatory material against its officials. Public authorities who apprehend that they or
their colleagues may be defamed, cannot prevent the Press upon publication of such
material, but they can take action for damages after the publication of such material
on the proof of publication based on false facts. No action can be initiated against the
press if the publication was based on public records including court of records.

Freedom of Press, the Court said, means absence of interference by the State except
insofar as it is authorized by the Constitution and bye-laws. The ruling was given by the
Court while allowing a writ petition by a Tamil Weekly Magazine ‘Nakheeran’ seeking a
direction to the Tamil Nadu Government from interfering with the publication of the
autobiography of the condemned prisoner, Auto Shanker, in the magazine who had
been charged and tried for as many as six murders. His mercy petition to the President
of India was pending consideration. The petitioner was editor, printer and publisher of
the Magazine. Their case is that the condemned prisoner Auto Shanker who was
convinced for six murders and sentenced to death had written his autobiography in Jail
and had handed over the same to his wife with the knowledge and approval of the Jail
authorities, for being delivered to his Advocate with a request to publish it in the
petitioner’s magazine. The autobiography depicted close nexus between the prisoner
and several IAS, IPS and other officers, some of whom were indeed his partners in
several crimes. The petitioners decided to commence the serial publication of the
autobiography and for this purpose they announced that very soon the magazine
would be coming out with the sensational life and history or Auto Shankar. This
announcement sent shock waves among several police and prison officials who were
afraid that their links with the condemned prisoner would be exposed. They forced
Auto Shanker, by applying third degree methods, to write letters addressed to the IG
of Prisons and the Editor of the magazine requesting that his life story should not be
published in the magazine. The IG of Prisons wrote a letter to the Editor on July 15,
1994 informing him that the autobiography was false and Auto Shanker had denied
that he had written any such book and therefore asked him to stop the publication of
serial.

The Division Bench of the Supreme Court consisting of Mr. Justice B. P. Jeevan Reddy
and Mr. Justice Suhas C. Sen held – the Petitioners have a right to publish the
autobiography of Auto Shanker in so far as it appears from the public records even
without his consent or authorization. But if they go beyond that and publish his life
story, they may be invalidating his right to privacy and will be liable for the
consequences in according with the law. Similarly the State and its officials who
apprehend that they may be defamed cannot impose prior restraint upon the
publication of the alleged autobiography. The remedy of the affected officials, if any, is
after the publication.

CONSTITUTION | 27
The court said that the citizens right to privacy, is included in Article 21 of the
Constitution and ‘a citizens has a right to safeguard, the privacy of his own, his family,
marriage, procreation, motherhood, child bearing and education among other
matters’. None can publish anything concerning the above matters without the person
consent, whether truthful or otherwise and whether laudatory or critical. If he does so
he would be violating the right to privacy of the person concerned and would be liable
for action for damages. It is enough for the press to prove that it acted after a
reasonable verification of the facts; it is not necessary for the Press to prove that what
is published was true.

Matters of decency is an exception to this rule. The name of female who was victim of
sexual assault, kidnapping, abduction or life offence should not be published in the
Press. The right of privacy of private persons stands on different footings. In a
democratic society public officials must always be open to criticism.

The freedom of press even though implicit in the freedom of speech and expression
does not stand on higher footing than the freedom of speech and expression of a
citizen. It is subject to limitations as mentioned in clasue (2) of Article 19.

Banning entry and circulation of Journal: In Romesh Thapper v/s State of Madras, a
law banning entry and circulation of journal in a State was held to be invalid. The
petition was filed under Section 32 of the Constitution ban on the entry and circulation
of petitioner’s weekly journal ‘Cross roads’. The order was passed under Madras
Maintenance of Public Order Act, 1949 Section 9 (1-A). The Court held that the ban
instituted a violation of Right to Freedom and it was broad and vague. It was not
legitimate as there was no objective public safety threat. It also held that there can be,
no doubt, that freedom of speech and expression includes freedom of propagation of
ideas, and that freedom is ensured by the freedom of circulation. Liberty of circulation
is essential to that freedom as the liberty of publication. Indeed without circulation the
publication would be of little value. Restriction on freedom of speech and expression
can only be imposed on grounds mentioned in Article 19(2) of the Constitution. A lay
which authorizes imposition of restrictions on grounds of ‘public safety’ or the
‘maintenance of public order’ falls outside the scope of authorized restrictions under
clause (2) and is void and unconstitutional. The honorable Court said that ‘Freedom of
speech and press lay at the foundation of all democratic organization for without free
political discussion no public education, so essential for the proper functioning of the
process of popular Government, is possible.

2. FREEDOM OF ASSEMBLY
 Freedom of Assembly is available in Article 19 (1) (b) and 19(3).
 Article 19(1)(b) guarantees to all citizen of India right ‘to assemble peaceably and without arms’.
The right of assembly includes the right to hold meetings and to take out processions. This right is
however subject to the following restriction:
1. The assembly must be peaceable.
2. It must be unarmed.
3. Reasonable restriction can be imposed under Clause 3 of Article 19.

CONSTITUTION | 28
 The right of assembly is implied in the very idea of the democratic Government. The right of
assembly thus includes right to hold meetings and to take out processions. This right, like other
individuals rights is not absolute but restrictive. The assembly must be non-violent and must not
cause any breach of public peace. If the assembly is disorderly or riotous then it is not protected
under Article 19(1)(b) and reasonable restriction may be imposed under clause (3) of Article 19 in
the interests of ‘sovereignty and integrity of India’ or ‘public order’.
 When a lawful assembly becomes unlawful - Article 19(1)(b) saves existing Indian law regulating
public meetings in the interest of public order if the restrictions are reasonable. If an assembly
becomes unlawful it can be dispersed. Chapter VIII of the Indian Penal Code lays down the
conditions when an assembly becomes ‘unlawful’. Under Section 141 of the Indian Penal Code, an
assembly of five or more persons becomes an unlawful assembly if the common object of the
persons composing assembly is –
(a) To resist the execution of any law or legal process,
(b) To commit any mischief or criminal trespass,
(c) Obtaining possession of any property by force,
(d) To compel a person to do what he is not legally bound to do or omit which he is legally entitled
to do,
(e) By means of criminal force or show of criminal force to overawe the Government or any public
servant in the exercise of his lawful powers.
 An assembly which was not unlawful when assembled may subsequently become unlawful if it
becomes violent or is likely to result in disturbance. Under Section 129 of the Criminal Procedure
Code, 1973, such as assembly may be ordered to be dispersed if the disturbance to the public
peace is reasonably apprehend. Section 151 of the Indian Penal Code makes it an offence not
disperse after a lawful command to disperse has been given.
 Section 107 of the Criminal Procedure Code empowers Magistrate to obtain security for keeping
the peace from any person who is likely to commit a breach of peace. Section 144, Criminal
Procedure Code, 1973 empowers the Magistrate to restraint an assembly, meeting or procession if
there is a risk of obstruction, annoyance or injury to any person lawfully employed or danger to
human life, health or safety or a disturbance of the public tranquility or a riot or an affray.
 The Police Act, 1861 empowers a public officer to direct the conduct and prescribe the route and
time of all assemblies and processions in the interest of public order. Under 30 of this Act, a prior
license has to be taken by member of the public to take out a procession. A Law conferring
authority on the Magistrate to grant or refuse a licence to hold a meeting or a law imposing a
restriction that no public procession could be taken out without a licence from a Magistrate has
been to be valid.
 The Prevention of Seditious Meeting Act, 1911, prohibits public meeting likely to promote sedition
or to cause a disturbance of public tranquility. It empowers the State Government to declare the
whole or any part of the State to be a ‘proclaimed area’. Thereupon, no public meeting for the
furtherance or discussion of any subject likely to cause disturbance for the excitement, for the
exhibition or distribution of any writing or printed matter relating to any such subject, shall be held
in any such proclaimed area,
(a) unless written notice of the intention to hold such meeting and of the time and place of such
meeting has been given to the District Magistrate or the Commissioner of Police, as the case
may be, at least three days previously,
(b) unless permission to hold such meeting has been obtained in writing from the above officers
as the case may be. All the above mentioned statutory provisions impose reasonable
restrictions under clause (3) of Article 19 on the right to freedom of assembly.

CONSTITUTION | 29
3. FREEDOM TO FORM ASSOCIATION
 Freedom to Form Association available in Article 19 (1) (c) and 19(4).
 Article 19 (1) (c) of the Constitution of India guarantees to all its citizens the right ‘to form
association or unions or Co-operate Societies’. Under clause (4) of Article 19 however, the State
may by law impose reasonable restrictions on this right in the interest of public order or morality or
the sovereignty and integrity of India.
 The right to association pre-supposes organization. It is an organization or permanent relationship
between its members in matters of common concern. It thus includes the right to form companies,
society, partnership, trade union, and political parties. The right guaranteed is not merely the right
to form association but also to continue with the association as such. The freedom to form
association implies also the freedom to form or not to form, to join or not to join, an association or
union.
 In Damayanti v/s Union of India, the validity of Hindi Sahitya Sammelan Act, 1962 was challenged
as violative of Article 19)(1)(c). The Petitioner was a member of an association. The Act changed the
composition of the association and introduced new members. The result of this alteration was that
the members who voluntarily formed the association were not compelled to act in the association
with other members in whose admission they had a say. The Supreme Court held that the Act
violated the rights of the original members of the society to form an association guaranteed under
Article 19(1)(c). The Court said, ‘The rights to form an association necessarily implies that the
person forming the association have also the right to be associated with only those whom they
voluntarily admit in the association. Any law by which members are introduced in the voluntary
association without any option being given to the members to keep them out, or any law which
takes away the membership of those who have voluntarily joined it, will be a law violating the right
to form an association’. The Hindi Sahitya Sammelan Act, 1962 does not merely regulate the
administration of the affairs of the original society, what it does is to alter the composition of the
society itself. The result of this change in the composition is that het members who voluntarily
formed the association are now compelled to act in the association with other members who have
been imposed as members by the Act and in whose admission to membership they had no say.
Such alteration in the composition of the association itself clearly interferes with the right to
continue to function as members of the association which was voluntarily formed by the original
founders. The Act, therefore, violates the right of the original members of the society to form an
association guaranteed under Article 19(1)(c).
 Restrictions on the Freedom of Association: The right of association, like other individuals
freedom, is not unrestricted. Clause (4) of the Article 19 empowers the State to impose reasonable
restrictions on the right of freedom of association and union in the interest of ‘public order’ or
‘morality’ or ‘sovereignty or integrity’ of India.
The Criminal Law (Amendment) Act, 1908, as amended by the Madras Act, 1950, provided that if
the State Government was of opinion that any association interfered with the administration of law
or with the maintenance of law and order or that it constituted a danger to the public peace it
could, by notification in the Official Gazette declare such association to be unlawful. Such a
notification was to be placed before an Advisory Board. Representations against such a notification
could be made. If the Advisory Board was of opinion that the association was not unlawful, the
Government would cancel the notification. The Validity of the above Act was challenged it the case
of State of Madras v/s V. G. Row, the Supreme Court held that the restriction imposed by section
16(2)(b) of the Act unreasonable. The test under it was subjective satisfaction of the Government
and the factual existence of the grounds was not a justifiable issue. Therefore, the vesting of power
in the Government to impose restriction on this right, without allowing the grounds tested in a

CONSTITUTION | 30
judicial enquiry, was a strong element to be taken into consideration in judging the reasonableness
of the restriction on the right to form association or union. The existence of Advisory Board could
not be a substitute of judicial inquiry.

 The Right to form union does not carry with it the right to achieve every object. Thus the Trade
Unions have no guaranteed right to an effective bargaining or right to strike or right to declare a
lock-out.

4. FREEDOM OF MOVEMENT
 Freedom to Form Association available in Article 19 (1) (d) and 19(5).
 Freedom of Movement guarantees to all citizens of India the right ‘to move freely throughout the
territory of India’. This right is however, subject to reasonable restriction mentioned in clause (5) of
Article 19 i.e. in the interest of general public or for the protection of the interest of any Scheduled
Tribe.
 Article 19 (1)(d) of the Constitution guarantees to its citizens a right to go wherever they like in
Indian territory without any kind of restriction whatsoever. They can move not merely from one
State to another but also one place to another within the same State. This freedom cannot be
curtailed by any law except within the limits prescribed under Article 19 (5). What the Constitution
lays stress upon is that the entire territory one unit so far the citizen national minded and not to be
petty and parochial.
 Ground of Restrictions – The State may under clause (5) of Article 19 impose reasonable restriction
on the freedom of movement on two grounds:
1) In the interest of general public.
2) For the protection of the interest of Scheduled Tribes.
 In N. B. Khare v/s State of Delhi, the petitioner was served with an order of externment by the
Disctrict Magistrate, Delhi, to remove himself immediately from Delhi district and not to return
there for a period of three months. The order was made under the East Punjab Safety Act, 1949.
The petitioner contended that the order imposed unreasonable restrictions on the right to move
freely, because (a) the externment order depended on the subjective satisfactory of the Executive
and (b) the Act did not fix any maximum time beyond which the order could continue. The Supreme
Court held that the mere fact that the power to make the order of externment was given to the
State Government or District Magistrate whose satisfactory was final did not make the restriction
unreasonable because the desirability of passing such an individual order in emergency had to be
left to an officer. The second contention was rejected on the ground that the Act was of limited
duration, therefore, there was no possible of an order of externment being made for definite
period. But in State of M. P. v/s Baldeo Prasad, the Court held – A law providing for externment of
‘dangerous character’ from a particular locality cannot be called reasonable if it does not specially
define as to what is meant by dangerous character as it gives the administrative authority arbitrary
power to determine as to whether a citizen of dangerous character.
 In State of Uttar Pradesh v/s Kaushalya the Supreme Court has held that the right of movement of
prostitutes may be restricted on ground of public health and in the interest of public morals. The
right of a citizen to move freely may also be restricted for the protection of the interest of
‘scheduled tribes’. The object is to protect the original tribes which are mostly settled in Assam.
These tribes have their own culture, language, customs and manners. It was considered necessary
to impose restriction upon the entry of outsiders to these areas. It was feared that uncontrolled

CONSTITUTION | 31
mixing of the tribes with the people of other areas might produce undesirable effect upon the tribal
people.

5. FREEDOM OF RESIDENCE
 Freedom to Form Association available in Article 19 (1) (e) and 19(5).
 According to Article 19 (1) (e) every citizen of India has the right ‘to reside and settle in any part of
the territory of India’. However, under clause (5) of Article 19 reasonable restriction may be
imposed on this right by law in the interest of the general public or for the protection of the
interest of any Schedule Tribe.
 The object of the clause is to remove internal barriers within India or any of its parts. The words
‘territory of India’ as used in this Article indicate freedom to reside anywhere and in any part of the
State of India.
 The right to move and reside throughout the country are complementary and often go together.
Therefore, most of the cases considered under Article 19(1)(d) are relevant to Article 19(1)(e) also.
This right is subject to reasonable restrictions imposed by law in the interest of general public or for
the protection of the interests of any Scheduled Tribes. Thus, where a prostitute, under the
Suppression of Immoral Traffic in Women and Girls Act, 1956, was ordered to remove herself from
the limits of a busy city or the restrictions was placed on her movements and residence, it was held
to be a reasonable restriction.
 In M.P. v/s Bharat Singh, Section 3(1) of the M.P. Public Security Act, 1959, empowered the state
Government to issue an order requiring a person to reside or remain in such a place as might be
specified in the order or to ask him to leave the place and to go to another place selected by
authorities in the interest of security of the State or public order. The Supreme Court held that
Section 3(1)(b) of the Act imposed unreasonable restriction on the right guaranteed by Article 19
(1) (d) and, therefore, void. The Act did not give an opportunity to the person concerned of being
heard about the place where he was asked to reside. The place selected for him might have no
residential accommodation and no means of livelihood, etc. the section did not indicate the extent
of the place, area, or its distance from the residence of the person externed.

6. FREEDOM OF PROFESSION, OCCUPATION, TRADE OR BUSINESS


 Freedom of Profession, Occupation, Trade or Business is available in Article 19 (1)(g) and 19(6).
 Article 19 (1)(g) guarantees that all citizens shall have the right ‘to practise any profession, or to
carry out any occupation, trade or business’. However, the right to carry on a profession, trade, etc.
is not unqualified. It can be rejected and restricted by authority of law.
 The state can under clause (6) make any law (a) imposing reasonable restriction on this right in the
interest of the public, (b) prescribing professional or technical qualifications necessary for practicing
any profession or carrying on any occupation, trade, business. (c) enabling the State to carry on any
trade or business to the exclusion of citizens wholly or partially.
 In P. A. Majumdar v/s State of Maharastra it has been held the right to establish an educational
institution, for charity or for profit, being an occupation is guaranteed under the Constitution to all
citizens under Article 19(1)(g) and to minorities under Article 30. ‘Education’ even though is an
occupation but it cannot be equated with trade or business.
 The right to carry out business includes that the person can close the business at any time. The
State cannot compel a citizen to carry on business against his will. This right too can be restricted,
regulated or controlled by law in public interest. The company Excel Wear v/s Union of India the
petitioner was a registered firm and it was going in loss and wanted to close down the firm and so
gave a notice to the State for approval of the closure. But it was refused on grounds that it had to

CONSTITUTION | 32
give three months’ notice and the State would decide. The State on no sufficient and adequate
grounds and for public interest did not allow the closure. The Supreme Court held that no had the
right to carry out a business if he cannot even pay minimum wages to the laborers. He must close
it.
 In Sodan Singh v/s New Delhi Municipal Corporation, the Supreme Court held that hawkers have a
fundamental right to carry on trade on pavement of roads, but subject to reasonable restrictions
under Article 19(6) of the Constitution
 Ban on Masala and Gutkha. In Godawar Pan SA Masala v/s Union of India, the petitioners
challenged the Constitutional validity of notifications issued by the Food (Health) Authority under
Section 7(iv) of the Prevention of Food Adulteration Act by which the manufacture, sale and
storage and distribution of Pan Masala and Gutkha containing tobacco were banned for five years
as violative of Article 19(1) of the Constitution. The Supreme Court held that the ban was not
violative of Article 19(1) of the Constitution. Restriction could have been on sale to under-aged
persons and not by way of total ban. It was therefore unconstitutional and invalid.
 Trade in liquor is not a Fundamental Right.
 Dancing is a Fundamental Right as it was for livelihood.
 Slaughter of cow, calf and other milch animals: in State of Gujarat v/s Mirzapur Moti Qureshi
Kasab Jamat, the petitioners who were butchers challenged the Constitutional validity of the
‘Bombay Animal Preservation Gujarat Amendment Act, 1994 as applicable to the State of Gujarat.
Under this legislature the State of Gujarat imposed a total ban on slaughter of cows and calves and
other milch and draught cattle’s. The Supreme Court held that the prohibition of slaughter of cows
and her progeny does not amount to total ban on activity of butchers. The ban imposed by section
5 of the Act is total with regard to slaughter of particular class of cattle. They are left free to
slaughter cattle other than those specified in the Act. It is in interest of the general public and is
reasonable restriction within the meaning of Article 19(1)(g) of the Constitution.

CONSTITUTION | 33
UNIT IV

10. ARTICLE 15 – NO DISCRIMINATION ON THE GROUDNS OF RELIGION, RACE, CASTE.


Ans Article 15
 Articles 15 provides for a particular application of the general principle embodied in Article 14. When the
discrimination is based upon one of the grounds mentioned in Article 15, the reasonableness of the
classification will be tested under Article 14.
 The guarantee under Article 15 is available to citizens only
15(1) directs the State not to discriminate against a citizen on grounds of religion, race, caste, sex, place or
birth or any of them.
15(2) prohibits citizens as well as the States from making such discrimination with regards to access to
shops, hotels, etc., and all places of public entertainment, of public resort, wells, tanks, roads, etc.
15(3) empowers the State to make special provisions for the protection of women and children.
15(4) empowers the State to make special provisions for the backward classes of citizens
 Article 15 Clause 1:
- 15(1) directs the State not to discriminate against a citizen on grounds of religion, race, caste, sex,
place or birth or any of them. If the State makes any discrimination on the above grounds then it is
invalid.
- In Nainsukhdas v/s State of U.P. a law which provided for elections on the basis of separate
electorates for members of different religious communities was held to be unconstitutional.
- In State of Rajasthan v/s Pratap Singh, the Supreme Court invalidated a notification of the Police Act
1861 which declared certain areas as disturbed and made the inhabitants of those areas to bear the
cost of additional police stationed there but exempted all Harijans and Muslims. The exemption was
given on the basis of caste or religion and hence contrary to Article 15(1).

 Article 15 Clause 2:
- 15(2) prohibits citizens as well as the States from making such discrimination with regards to access to
shops, hotels, etc., and all places of public entertainment, of public resort, wells, tanks, roads, etc.
- This Article prohibits both State and Individual from making any discrimination.
- The object is to eradicate the abuse of the Hindu social system and to herald a united nation.

 Article 15 Clause 3:
- 15(3) – It empowers the State to make Special provisions for women and Children. It is an exception to
the rule laid down in clauses 1 and 2.
- The reason of this is that ‘women’s’ physical structure and the performance of maternal functions
place her at a disadvantage in the struggle for subsistence and her physical well-being becomes an
object of public interest and care in order to preserve the strength and vigor of the race.
- In Yusuf Abdul v/s State of Bombay, Section 497 of IPC which only punishes man for adultery and
exempts the woman from punishment even though she may be equally guilty as an abettor was held
to be valid since the classification not based on the grounds of sex alone. The children too have a right
for free education or measure for prevention of their exploitation.

 Article 15 Clause 4:
- 15(4) - It empowers the State to make special provisions for advancement of Backward Classes. This is
another exemption to clause 1 and 2.
- In State of Madras v/s Champakam Dorairajan, the Madras Government had reserved seats in State
Medical and Engineering Colleges for different communities in certain proportions on the basis of

CONSTITUTION | 34
religion, race and caste. The State defended that it was enacted with a view to promote the social
justice for all sections of the people as required by Article 46 of the Directive Principles of State Policy.
The Supreme Court held the law void because it classified students on the basis of caste and religion
irrespective of merit. The Directive Principles of State Policy cannot override the Fundamental Rights.
- The provisions made in clause 4 of Article 15 is only an enabling provision and does not impose any
obligation on the State to take any special action under it. It merely confers a discretion to act if
necessary by way of making special provision for backward classes. A writ cannot be issued by State to
make reservations.
- Under this Article (15,4) two things are to be determined:
a. Who are socially and educationally backward classes?
b. What is the limit of reservation?
- Backward and more backward classification is not bad: In Balaji v/s State of Mysore, the Mysore
Government issued an order under Article 15(4) reserving seats in the Medical and Engineering College
in the State as follows: Backward classes 28%, more backward classes 20%, SC and ST 18%. Thus 68%
of the seat of the college and only 32% for merit pool. The validity of the order was challenged by
candidates who had secured more marks than those admitted under the order but did not get
admission.
The Court held that the sub-classification of backward and more-backward was not justified under
Article 15(4) and it should be based on both social and educational and not social or educational. It
also said that though caste is the relevant factor, it cannot be sole test for ascertaining whether
particular class is a backward class or not. Poverty, occupation, place of inhabitation may all be
relevant factor. Reservation of 68% of seats to the exclusion of all other candidates if a single
candidate from the ST was available, would amount to fraud upon the Constitution. Article 15(4) only
enables the State to make special and not exclusive provisions for the backward classes. The State
would not be justified ignoring altogether advancement of the rest of the society in its zeal to promote
the welfare of the backward classes. National interest would suffer if students were excluded from
admission in institutions of higher education. The Court said that the special provisions should be less
than 50% how much less than 50% would depend upon the relevant prevailing circumstances in each
case.
- In Indra Sawhney v/s Union of India, Mandal Commission case the Supreme Court by 6:3 majority held
that sub-classification of backward classes into more backward can be done. But the reservation
should not exceed 50%. Backward also include SC and ST.
- High Caste girl marrying ST – not entitled to reservation: In Dr. Neelima v/s Dean of P.G. Studies
Agriculture University, Hyderabad, it was held that a high caste girl marrying a boy belonging to
scheduled Tribe is not entitled to the benefit of reservation available to STs. The girl had sought
admission to M.Sc. course in Agriculture University, under reservation quota for ST. The court held that
she was not entitled for the same.
In Meera Kanwaria v/s Sunita, it has been held that if a female of high caste Hindu marries a person
belonging to SC she is not entitled to take the benefits of reservation under Article 15(4) and 16(4) of
the Constitution. She was not accepted by the community of her husband. Therefore, it was held that
she could not acquire the status of SC and could not contest election on a seat reserved for SC. Mere
acceptance of family of husband is not sufficient. Strict proof of acceptance by community of husband
is necessary.
 Article 15 Clause 5:
- This Article was inserted by the Constitution (93rd Amendment) Act, 2005, the Parliament inserted
clause (5) in Article 15 with effect from 20.01.2006. It provides – “Nothing in this Article or in sub-
clause (g) of clause (1) of Article 19 shall prevent the State from making any special provisions by law,
for the advancement of any socially and educationally backward classes of citizens or for the scheduled

CONSTITUTION | 35
Castes or Scheduled Tribes in so far as such special provisions relate to their admission to educational
institutions including private educational institutions referred to in clause (1) of Artic 30”.

In T.M.A. Pai Foundation v/s state of Karnataka, and P. A. Inamdar v/s State of Maharashtra. It was
held that the State cannot make reservation of seats in admission in privately run educational
institutions. There the admissions can be done on the basis of common admission test conducted by
the State or these institutions and on the basis of merit. In Islamic Academy v/s State of Karnataka,
the court held that the State can fix quota for admissions to these educational institutions but it
cannot fix fee and also admissions can be done on the basis of common admission test on the basis of
merit.

CONSTITUTION | 36
11. INDIA IS A SECULAR STATE. DISCUSS THE STATEMENT IN LIGHT OF FUNDAMENTAL RIGHTS GUARANTEED
UNDER INDIAN CONSTITUTION.
INDIAN IS A SECULAR STATE. EXPLAIN THE STATEMENT IN LIGHT OF ARTICLE 25 – 28.
Ans India is a secular State:
 Articles 25 to 28 contain the freedom of religion.
 The concept of Secularism is implicit in the Preamble of the Constitution which declares the resolve of
the people to secure to all its citizens ‘liberty of thought, belief, faith and worship’
 The 42nd Amendment Act, 1976 has inserted the word ‘secular’ in the Preamble.
 A secular State was never considered as an irreligious or atheistic State. It only means that in matters of
religion it is neutral.
 The State protects all religions but interferes with none. It eliminates God from matters of the State and
ensures that on one shall be discriminated against on grounds of religion. The State has no religion of its
own. It should treat all religions equally.
 In a secular State, the state is only concerned with the relation between man and man, not man and
God.
 Secularism therefore means that encouragement will not be given by the State to any specific religion
but equal view and sense of respect will be shown to each religion. The Constitution is totally above the
religious or communal attitude.
 In S. R. Bommai v/s Union of India, the Supreme Court has held that ‘secularism is a basic feature of the
Constitution’. The State treats equally all religions and religious denominations. Religion is a matter of
individual faith and cannot be mixed with secular activities. The Indian Constitution embodies positive
concept of secularism and not the negative concept. It is not anti-God. It separates Spiritualism with
individual faith, and not separation between State and Religion like America. The State is neither anti-
religion nor pro-religion. It treats all equally.
 In Aruna Roy v/s Union of India, the validity of the New National Education Policy 2002 which provided
for value based education to school children based on basis of all religions was challenged as violative of
Article 28 (No religious instruction shall be provided in any educational institution wholly maintained out
of State funds…) anti-secular. The Court held that study of religions in school education is not against
the secular philosophy of the Constitution. Justice Dharmadhikari said that from the from the last 50
years of experience of Constitution it is clear that complete neutrality towards religion and apathy for all
kinds of religious teachings in institutions of the State have not helped in removing misunderstandings
and intolerance inter se between sections of people of different religions, faiths and beliefs. Secularism
therefore is susceptible to positive meaning that is developing, understanding and respect towards
different religions. The essence of secularism is non-discrimination of people by the State on the basis of
religious differences. Secularism has to be practiced by positive approach by making all to understand
and respect all religions and faith. Thus basic distrust and intolerance can gradually be eliminated. He
quoted Gandhiji saying, ‘the real meaning of secularism is “Sarva Dharma Sambhav” meaning equal
treatment and respect for all religions and not “Sarva Dharma Sambhav” meaning negation of all
religions. Study of religions would strengthen the concept of secularism in its true spirit. How can we
develop cultured human beings of moral character without teaching them from childhood the
fundamental and spiritual values. The honorable Judge said that ‘Democracy cannot survive and
Constitution cannot work unless Indian citizens are not only learned and intelligent, but they are also of
moral character and imbibe the inherent virtues of human beings such as truth, love and compassion’.
 Article 25 (1) guarantees to every person freedom of conscience and the right freely to profess,
practice and propagate religion.
Religion is not defined in the Constitution. The Supreme Court has defined it broadly. Religion is a
matter of faith with individuals and communities and it is not necessarily theistic. A religion has its basis

CONSTITUTION | 37
in a system of beliefs or doctrines which are regarded by those who profess that religion as conducive to
their spiritual well-being’. It is a matter of personal faith and belief.
Freedom of Conscience means absolute inner freedom to mould his own relations with God in whatever
manner he like. To profess means to declare freely and openly one’s faith and belief. He has right to
practice his belief by practical expressions in any manner he likes. It consists of performing prescribed
religious duties, rites, rituals. To propagate means to spread and publicize his religious views for the
edification of others. This should be done without coercion. It does not give you the right to convert, but
only transmit and spread ones religion by exposing its tenants.

National Anthem case: the Supreme Court has held that no person can be compelled to sing the
National Anthem if he has genuine, conscientious religious objection.

The Case: three children belonging to the ‘Jehova’s Witnesses’ of the Christian community were
expelled from the school for refusing to sing the National Anthem. They challenged the validity of their
expulsion on the ground that it was violative of their Fundamental Rights under section 25(1). They had
stood up respectfully when the national Anthem was being sung every morning at their school but they
did not join in the singing of it as it was against the tenants of their religious faith which did not permit
them to join in any rituals except if it be in their prayer to Jehova, their God. The headmistress seeing it
as very bad influence asked them to give in writing that they would participate in the singing of the
National Anthem. It was seen as ignorance of the Constitution. If religious practice run counter to public
order, morality, health or a policy of the Government to uphold the sovereignty and integrity and unity
of the Nation then the said religious practice must give way for the benefit of the people and the nation
as a whole.

On Appeal, the Supreme Court reversed the High Court’s decision and held that there is no legal
obligation in India for a citizen to sing the National Anthem. The right under Article 25(1) cannot be
regulated by executive instructions which had no force of law. The court said that standing up while the
National Anthem is sung meant that they had shown proper respect thus not violated the Fundamental
Duties laid down in Article 51-A of the Constitution. Their conduct did not show insult to the National
Honour Act 1971 not caused any disturbance to the assembly singing the National Anthem. The Court
ordered the children to be allowed to pursue their studies.

In this context the ruling of the Supreme Court would add only fuel to the fire and give strength to the
forces who would interpret as a license for all and everyone to disregard the National Anthem and the
National Flag and thereby endangering the unity and integrity of the country. The judgment of the
Kerala court was right and the Supreme Court had erred as it had taken care about the National
interests more than the individual right which must give way for the benefit of the nation. Religion
should not be given preference over the national interests.

In Gulam Kadar Ahmadbhai Menon v/s Surat Municipal Corporation, the Gujarat High court has held
that the right to religion guaranteed to citizens under Articles 25 and 26 of the Constitution does not
prohibit State to acquire any place of worship for public purpose. In this case the petitioner had
challenged the validity of Section 212 of the Bombay Provincial Municipal Corporation Act, 1949. Under
the Act the Municipal Corporation had ordered the demolition of some parts of the two mosques
situated in main road in Surat district in the State of Gujarat. The court held that the acquisition of a
religious place or a part thereof is not prohibited by the Constitution and therefore can be acquired in
the public interest for widening the road. Accordingly, it was held that the order for the demolition of
certain portions of the two mosques for widening the road was valid. The protection in Articles 25 and

CONSTITUTION | 38
26 is for religious practice which forms an essential or integral part or practice of that religion. While
offer of prayers or worship is a religious practice, it’s offering at every location is not an essential part of
such religious practice unless the place has a particular significance for that religion so as to form an
essential or integral part thereof. It also said that the Corporation has the power to acquire a place of
worship or a part for the purpose of widening the road, it is expected to examine the needs for such
acquisition. Need to examine if the public need overrides religious need. If yes then the order to
demolish is valid and Constitutional.

 Freedom to manage religious affairs. Article 26 speaks of Subject to public order, morality and health,
every religious denomination or any section thereof shall have the right.
b. to establish and maintain institutions for religious and charitable purposes;
c. to manage its own affairs in matters of religion;
d. to own and acquire movable and immovable property; and
e. to administer such property in accordance with law.

This right is of the ‘organized body’ like the religious denomination or any section thereof. In Article 26
religious denomination should have the following conditions: 1. It must be collection of individuals who
have a system of beliefs which they regards as conducive to their spiritual well-being, that is, a common
faith; 2. It must have a common organization and 3. It must be designated by a distinctive name. This
was established in Bramchari Sidheswar Sahai v/s State of W.B. popularly known as the Ramkrishna
Mission.

b. to establish and maintain institutions for religious and charitable purposes;


Under Article 26 Clause (a) every religious denomination has right to establish and maintain
institutions for religious and charitable purposes. The words ‘establish and maintain’ must be read
together and therefore it is only those institutions which a religious denomination establishes
which is can claim to maintain it.

c. to manage its own affairs in matters of religion;


Under Article 26 Clause (b) a religious denomination or organization is free to manage its own
affairs in matters of religion. The State cannot interfere in the exercise of this unless they run
counter to public order, health or morality. Accordingly every religious organization enjoys
complete freedom in the matters of deciding what rites and ceremonies are essential according to
the tenets f other religion they hold. The court, has the right to determine whether a particular
rite or ceremony is regarded as essential by the tenets of a particular religion.
‘Matters of religion’ or religion includes practices, rites, and ceremonies considered essential for
practice of religion. The State can regulate if used for any other purpose such as political or anti-
national activities under Article 25 (1 and 2)

d. to own and acquire movable and immovable property; and


Under Article 26 Clause (c) a religious denomination or organization is free to own and acquire
movable and immovable property for its religious affairs and matters of religion.

e. to administer such property in accordance with law.


Under Article 26 Clause (d) a religious denomination or organization is free to administer such
property in accordance with law. This is a limited right, and it is subject to the regulatory power of
the State in Article 25 clause (2) (a) and also any general property.

CONSTITUTION | 39
 Article 27: No person shall be compelled to pay any taxes, the proceeds of which are specifically
appropriated in payment of expenses for the promotion or maintenance of any particular religion or
religious denomination.
This Article emphasizes the secular character of the State. The Public money collected by way of tax
cannot be spent by the State for the promotion of any particular religion. The reason underlying this
provision is that India being secular State and there being freedom of religion guaranteed by the
Constitution both to individual and groups it is against the policy of the Constitution to pay out of public
funds any money for the promotion or maintenance of the particular religion or religious denomination.
On the basis of distinction between tax and fee the Supreme Court in Sri Jagannath v/s State of Orissa,
held that the levy under the Orissa Hindu Religious Endowment Act, 1939, was in the nature of a fee and
not tax. The payment was demanded only for the purpose of meeting the expenses of the Commissioner
and his office which was the machinery set up for due administration of the affairs of the religious
instruction. The object was not the fostering or preservation of Hindu religion or the denomination
within it, but to see that religious institutions were properly administered.

 Article 28.
i. No religious instruction shall be provided in any educational institution wholly maintained out of
State funds.
ii. Nothing in clause (1) shall apply to an educational institution which is administered by the State
but has been established under any endowment or trust which requires that religious instruction
shall be imparted in such institution.
iii. No person attending any educational institution recognized by the State or receiving aid out of
State funds shall be required to take part in any religious instruction that may be imparted in such
institution or to attend any religious worship that may be conducted in such institution or in any
premises attached thereto unless such person or, if such person is a minor, his guardian has given
his consent thereto.

This clause shall apply to any educational institution wholly maintained out of State funds and not to
educational institutions which is administered by the State but has been established under any
endowment or trust which requires that religious instruction shall be imparted in such institutions.

There are four types of institutions:


a. Institutions wholly maintained by the State.
b. Institutions recognized by the State.
c. Institutions that are receiving aid out of the State Funds.
d. Institutions that are administered by the State but are established under any trust or endowment.
In (a) type no religious instruction can be imparted. Ina (b) and (c) type religious instructions can be
imparted only with the consent of the individuals. In (d) institutions there is no restriction on religious
instructions. Case of Aruna Roy v/s Union of India.
In D.A.V. College, Bhatinda v/s State of Punjab, the validity of Section 4 of the Guru Nanak university
Act directed the State to make provision for the study and research on the life and teaching of Guru
Nanak was challenged on the ground that it was violative of Article 28 which prohibited religious
instruction in State aided educational institutions. The court rejected the contention and held that the
Section 4 which enjoined the university to encourage an academic study of life and teaching of Guru
Nanak does not amount to religious instruction or promotion of any particular religion and, therefore, is
Constitutionally valid.

CONSTITUTION | 40
 Restrictions on Freedom of Religion:
This right is not absolute. There are basically three restrictions on freedom of Religion. They are:
a. Religious liberty subject to public order, morality and health.
b. Regulation of economic, financial, political or other secular activity which may be associated with
religious practice;
c. Provide for social welfare and social.

a. Religious liberty subject to public order, morality and health. (Article 25 (1))
In the name of religion no act can be done against public, order, morality and health of the public.

Thus section 34 of the Police Act prohibits the slaughter of cattle or indecent exposure ones’
person in a public place. These acts cannot be justified on the plea of practice of religious rites.

In the name of religion ‘untouchability or traffic in human being e.g. system of Devdasis cannot be
tolerated.

The freedom to practice religion cannot affect the exercise of these freedom by others. These
freedom are reasonable restrictions under clause (2) of Article 19 (right to freedom).

Noise Pollution in the name of Religion not allowed: In a significant judgment in Church of God in
India v/s KKRMC Welfare Association the Supreme Court has held that in the exercise of the right
to religious freedom under Article 25 and 26, no person can be allowed to create noise pollution
or disturb the peace of others. The custom of religious prayer through the use of loudspeakers is
not an essential element of any religion. In that the appellant is the Church of God (Full Gospel)
located at K.K.R. Nagar, Madhavam, High Road, Chennai. It has a prayer hall for the Pentecostal
Christians and provided with musical instruments such as drums set, triple bango, guitar, etc. The
at K.K.R. Majestic Colony Welfare Association made a complaint to the Tamilnadu Pollution
Control Board stating therein that prayers in the Church were recited by using loudspeakers,
drums and other sound producing instruments causing noise pollution and nuisance to the normal
life of the residents of the said colony. The Madras High Court directed the police authorities to
take action to stop noise pollution. On behalf of the Church it was contented that the petition was
filed with a view to prevent a religious minority institution from pursuing its religious activities
and the Court cannot prevent the Church from practicing its religious beliefs. It was also said that
the noise pollution was due to plying of vehicles and not due to use of loudspeakers etc. the
Supreme Court held that a person’s religious freedom is subject to “public order, morality and
health”. Even if there is any such religious practice it cannot be used to violate right to others or
to disturb their peace. The Court said that “no rights is an organized society can be absolute”. In
view of this the order of the authorities under the Madras Town Nuisance Act, 1889 and the Noise
Pollution (Regulation and Control) Rules, 2000 is valid and Constitution. The direction given by the
High Court to authorities to follow guidelines for control of noise pollution issued in 1995 AIHC
4168 and to make Church to keep speakers of lower level are legal in view of the pollution control
provision.

Forced Conversion not allowed: In Rev. Stainislaus v/s State of M.P. the validity of the two Acts -
Madhya Pradesh Dharma Swatantrya Adhiniyam, 1968 and the Orissa Freedom of Religions Act,
1967 passed by the State legislative authority of MP and Orissa respectively was challenged on
the ground that they were violative of the Fundamental Rights of the appellant guaranteed under
Art. 25 (1) of the Constitution. These acts were passed to prohibit forcible conversion of any

CONSTITUTION | 41
person to one’s own religion. He contended that right to propagate one’s religion meant the right
to convert person to one’s own religion and was a Fundamental Rights under section 25 (1) of the
Constitution. Secondly he argued that the State legislature had no competence to enact such a
law as it did not fall within the purview of Entry I of List II and Entry I of List III of Seventh
Schedule. It is covered in Entry 97 of List I so Parliament alone had the power to make the law and
not the State Legislature. Rejecting the contentions of the appellant the Supreme Court held that
impugned Acts, fell within the purview of Entry I of List II as they were meant to avoid
disturbances to public order by prohibiting conversion from one’s religion to another in a manner
reprehensible to the conscience of the community. Thus if an attempt is made to raise communal
passions, e.g. on the ground that someone has been ‘forcibly’ converted to another religion, it
would, in all probability, give rise to an apprehension of a breach of the public order, affecting the
community at large. Therefore, legislation prohibiting forcible conversion of one’s own religion in
the interest of public order can be passed and is valid.

Two Children norms for elective post not violative of Article 25 - in Javed v/s Haryana, the
Supreme Court has held that section 175 (110 (q) of the Haryana Panchayati Raj Act, 1994 which
disqualifies persons having more than two children from contesting election for the post of
Sarpanch and Panch in Panchayat does not violate Article 25 of the Constitution. The validity of
the provision was challenged inter alia, on the ground that it interferes with freedom of religion
and hence violates Article 25 of the Constitution. It was said that the personal law of Muslims
permits performance of marriage with four women obviously for procreating children and any
restriction thereon would be violative of right to freedom of religion enshrined in Article 25 of the
Constitution. The Supreme Court however, held that the provision is Constitution. The freedom of
religion is subject to public order, morality and health. So the Article itself permits a legislation in
the interest of social welfare and reform. The Muslim law permits marrying four women but does
not anywhere mandates it as a duty to perform four marriages. Such practices which encourage
bigamy or polygamy can be regulated by making law. The right to contest election for any office in
Panchayat is neither fundamental right nor a common law right. It is statutory right and subject to
qualifications and disqualifications prescribed by law. If a person cannot fulfil then he can be
prevented from contesting elections for such posts.

b. Regulation of economic, financial, political or other secular activity which may be associated
with religious practice (Article 25, Clause 2a)
The freedom to practice extends only to those activities which are the essence of religion. It does
not cover secular activities which do not form the essence of religion. It is not always easy to say
which activities fall under religious practice or which are secular, commercial or political in nature.
Each has to be judged by its own facts and circumstances.

In Mohd. Hanif Quareshi v/s State of Bihar, the petitioner claimed that the sacrifice of cows on
the occasions of Bakri Id was an essential part of his religion and therefore the State law
forbidding the slaughter of cows was violative of his right to practice religion. The court rejected
this argument and held that the sacrifice of cow on the Bakri Id day was not an essential part of
the Mohammedan religion and hence could be prohibited by State under clause 2a of Article 25.

In Adelaid Co. v/s Common wealth, it was held that a person could not be allowed in exercise of
his freedom of religious practice and profession to carry on an anti-war propaganda in the guise of
religion when the nation is at war. Thus the political activities though arising out of religious belief
by a particular organization were held not to be protected by the Constitution.

CONSTITUTION | 42
c. Provide for social welfare and social (Article 25, clause 2 b)
Under this clause the State is empowered to make laws for social welfare and social reform. Thus
under this clause the State can eradicate social practices and dogmas which stand in the path of
the countries onwards progress. Such laws do not affect the essence of any religion. This clause
declares that where there is conflict between the need of social welfare and reform and religious
practice, religion must yield. Social evils cannot be practiced in the name fo religion.

In State of Bombay v/s Vasrasu Bapamali an Act which prohibited bigamy was held valid under
clause (2) (b), polygamy is not an essential part of the Hindu religion, therefore it can be regulated
by law.

Prohibition of civil practices such as Sati and system of Devadasi has been held to be justified
under this clause.

Under this clause the State is empowered to throw open all Hindu religious institutions of a public
character, to all classes and sections of Hindus. It ensures an individual’s legal right to enter into a
temple unobstructed irrespective of his caste, social inequalities, or under privileged class. The
expression Hindu includes Jain, Sikh and Buddhists. This right is only to worship and this right is
not absolute or unlimited in character.

CONSTITUTION | 43
UNIT V
12. STATE THE RIGHTS OF THE ARRESTED PERSONS UNDER ARTICLE 22.
Ans Rights of the Arrested Person
 Articles 22 contains the rights of the arrested person.
 According to Article 22 contains the procedural requirements which must be adopted and included in
any procedure enacted by the Legislature. If these procedural requirements are not met with then it
would be deprivation of personal liberty.
 Article 22 deals with two separate matters: 1. Persons arrested under the ordinary law of crimes and 2.
Persons detained under the law of Preventive Detention.
 Article 22 Clauses (1) and (2) guarantee four rights on a person who is arrested for any offence under an
ordinary law:
1. The right to be informed ‘as soon as may be, of the grounds of arrest.
2. The right to consult, and to be defended by, a legal practitioner (lawyer) of his choice.
3. The right to be produced before the nearest magistrate within a period of twenty-four hours of
such arrest.
4. The freedom from detention beyond the said period except by the order of a Magistrate.

1. The right to be informed ‘as soon as may be, of the grounds of arrest.
When any person is arrested then till the time the reason for the arrest are not given to the said
person, he cannot be kept in detention. It is the duty of the authority to disclose the grounds of
arrest of a person immediately. If it is delayed there must be ‘reasonable circumstances’. If it is
not done then the person can initiate the legal proceedings for getting the bail from the court and
can pray for the relief of habeas corpus writ for his defense.

In a notable judgment in Joginder Kumar v/s State of UP the Supreme Court has laid down
guidelines governing arrest of a person during the investigation. This is intended to strike balance
between the needs of police on one hand and the protection of human rights from oppression
and injustice at the hands of law enforcing agencies. The arrest should be necessary and justified.

The National Police Commission in its third report has pointed out that power of arrest is one of
the chief source of corruption in the police. Therefore Court laid down the following guidelines to
be followed in making arrests.
a. An arrested person being held in custody is entitled, if he so request to have one friend,
relative or other person who is known to him or likely to take an interest in his welfare told
as far as is practicable that he has been arrested and where he is being detained.
b. Police office shall inform the arrested person when he is brought to police station of his
right.
c. An entry shall be required to be made in the police diary as to who was informed of the
arrest.

The court made it clear that these guidelines are not exhaustive. The DGP of all the States shall
issue necessary instructions requiring due observances of this requirement.

CONSTITUTION | 44
2. The right to consult, and to be defended by, a legal practitioner (lawyer) of his choice.
If a person is arrested he must be afforded opportunity to consult lawyer of his choice and if he is
unable to employ a counsel it is the duty of court to employ a lawyer for him. Prior to Maneka
Gandhi’s decision in India the view of the Court and a series of cases following that case it is clear
that the Courts will be bound to provide the assistance of a lawyer to a person arrested under an
ordinary law also.
In Hussainara Khatoon v/s Home Secretary, Bihar, the Supreme Court has held that it is the
Constitutional right of every accused person who is unable to engage a lawyer and secure legal
services on account of reasons such as poverty, indigence or incommunicado situation, to have
free legal services provided to him by the State and the State is under Constitutional duty to
provide a lawyer to such person if the needs of justice so require. If not than the trial is vitiated
under section 21.

Resolution of Bar Council not to defend some persons in Criminal cases – in A. S. Mohammed
Rafi v/s State of Tamil Nadu, the Bar Association of Coimbatore had passed a resolution that no
member of the Coimbatore Bar would defend the accused policeman in a case. The Supreme
Court judges held that this was wholly illegal against all traditions of Bar and against professional
ethics. Every person however wicked, depraved, vile, degenerate, perverted, loathsome,
execrable vicious or repulsive he may be regarded by society has a right to be defended in a court
of law and correspondingly it is the duty of the lawyer to defend him. Not defending such a
person is against the Constitution, the statute and professional ethics. Such a resolution is a
disgrace to the legal community. All such resolutions are null and void and the right minded
lawyers should ignore and defy such resolutions if they want democracy and rule of law to be
upheld in this country.

3. The right to be produced before the nearest magistrate within a period of twenty-four hours of
such arrest.
In addition to the furnishing of the grounds of arrest the arrested person must be produced
before the Magistrate within 24 hours of his arrest. It can be extended beyond 24 hours only
under the judicial custody. It affords a possibility, if not an opportunity, for immediate release in
case the arrest is not justified.
It helps the Magistrate to use judicial intelligence and expertise to determine whether the arrest
is made regularly, legally, and lawfully.

4. The freedom from detention beyond the said period except by the order of a Magistrate.
In the Opinion of the Allahabad High Court, if such a person arrested or detained is not produced
before the Magistrate within 24 hours then he gets the right to get himself released.
This means that if there is necessity of detention beyond 24 hours it is only possible under judicial
custody.
The expression ‘arrest and detention’ was held not to apply to a person arrested under a warrant
issued by the court on a criminal or quasi-criminal complaint or under security proceedings.
If the person is not produced before the Magistrate within 24 hours then the arrest would be
illegal.
In a significant judgment in CBI v/s Anupam J. Kulkarni the Supreme Court has laid down detailed
guidelines governing arrest of an accused when investigation cannot be completed within 24
hours. The Court has held that when a person is arrested under section 57 of Cr. P.C. he should be
produced before the Magistrate within 24 hours. The Judicial magistrate can authorize the
detention of the accused in such custody i.e. either police or judicial from time to time but the

CONSTITUTION | 45
total period of detention cannot exceed 15 days in the whole. After the expiry of 15 days, the
further remand can only be in judicial custody. There cannot be any detention in the police
custody after the expiry of first 15 days. If the investigation is not completed within the 90 days or
60 days then the accused has to be released on bail as provided under Section 167 (2) of the Cr.
P.C. The period of 90 or 60 days has to be computed from the date of detention as per the order
of the Magistrate and not from the date of arrest by the police. Thus after the expiry of first 15
days it should be only under Judicial custody.

The exception of this clause is: These rights are not available to 1. An enemy alien, 2. A person
arrested and detained under a Preventive Detention law

CONSTITUTION | 46
13. CRITICALLY EVALUATE ARTICLE 21 OF THE INDIAN CONSTITUTION WITH THE HELP OF RELEVANT CASE LAWS.
Ans
 Article 21 speaks of Protection to Life and Personal Liberty.
 It declares that the “No person shall be deprived of his life or personal liberty except according to
procedure established by law”.
 This right is guaranteed to ‘citizen’s as well as ‘non-citizens’.
 Maneka Gandhi Case:
Prior to Maneka Gandhi case, Article 21 guaranteed the right to life and personal liberty to citizens only
against the arbitrary action of the executive and not from legislative action. The State could interfere with
the liberty of citizens if it could support its action by a valid law.

The meaning of words “personal liberty” came up for consideration of the Supreme Court for the first
time in A. k. Gopalan v/s Union of India. In that case a communist leader was detained under the
Preventive Detention Act, 1950. The petitioner challenged the validity of his detention under Act on the
ground that it was violative of his right to freedom of movement under Article 19 (1) (d) which is the very
essence of personal liberty guaranteed under Article 21 of the Constitution. In this case the Supreme
Court interpreted the ‘law’ as ‘state made law’ and rejected the plea that by the term ‘law’ in Article 21
meant not the state made law but jus natural or the principles of natural justice. Fazal Ali, J, in his
dissenting judgment held that the Act was liable to be challenged as violating the provisions of Article 19.
He gave a wide and comprehensive meaning to the words ‘personal liberty’ as consisting of freedom of
movement and locomotion. Therefore, any law which deprives a person of his personal liberty must
satisfy the requirement of Articles 10 and 21 both. But this restrictive interpretation of the expression
‘personal liberty’ in Gopalan’s case has not been followed by the Supreme Court in its later decisions.

But after the Maneka Gandhi case – In Maneka Gandhi v/s Union of India, the Supreme Court has not
only overruled Gopalan’s case but has widened the scope of the words ‘personal liberty’ considerable.
Bhagwati, J. Observed: “the expression ‘personal liberty’ in Article 21 is of widest amplitude and it covers
a variety of rights which go to constitute the personal liberty of man and some of them have raised to the
status of distinct Fundamental Rights and given additional protection under Article 19.”

“The attempt of the Court should be expand the reach and ambit of the Fundamental Rights rather than
to attenuate their meaning and content by a process of Judicial construction. The Court lays down great
stress on the procedural safeguards. The procedure must satisfy the requirement of natural justice, i.e. it
must be just, fair and reasonable.

In kharak Singh v/s State of U.P. it was held that the expression ‘life’ was not limited to bodily restraint or
confinement to prison only but something more than mere animal existence. In that case the petitioner,
Kharak Singh, had been charged in dacoity case but was released as there was no evidence against him.
Under the U.P. Police Regulations, the Police opened a history-sheet for him and he was kept under Police
surveillance which included secret picketing of his house by the police, domiciliary visits at nights and
verification of his movements and activities. The Supreme Court held that domiciliary visits of the
policemen were an invasion on the petitioner’s personal liberty. By the term ‘life’ as used here something
more is meant than mere animal existence. The inhibition against its deprivation extends to all those
limits and faculties by which life is enjoyed. The provision equally prohibits the mutilation of the body or
amputation of an arm or leg. It is true that in Article 21 the word ‘liberty’ is qualified by a word ‘personal’
but his qualification is employed in order to avoid overlapping between those incidents of liberty which
are mentioned in Article 19. An authorized intrusion into a person’s home and the disturbance caused to
him is violation of the personal liberty of the individual. Therefore the Police Regulation authorizing

CONSTITUTION | 47
domiciliary visits was plainly violative of Article 21 as there was no law on which ti could be justified and it
must be struck down as unconstitutional.

But in Govind v/s State of M.P., the Supreme Court held that M.P. Police Regulations 855 and 856
authorizing domiciliary visits were Constitutional as they had the force of law. These regulations were
framed by the Government under section 46(2)(e) of the Police Act. The petitioner challenged the validity
of these Regulations on the ground that they were violative of his Fundamental Rights guaranteed in
Article 21 which also included the ‘right of privacy’. The Supreme Court held – Regulations 855 and 856
gave the force of law, and therefore, they were valid. The right of privacy would necessarily have to go
through a process if case by case development. Therefore, even assuming that the right to personal
liberty, the right to move freely throughout the territory of India and the freedom of speech create an
independent right of privacy which can be characterized as a Fundamental Rights, the right is not
absolute. Depending upon the character and antecedents of the person subject to surveillance and the
object and limitations under which surveillance is made, it cannot be said that surveillance by domiciliary
visits would always be unreasonable restriction upon the right of privacy. The impugned Regulation 855
empowers surveillance only of persons against whom reasonable material exists to include the opinion
that hate show ‘a determination to lead a criminal life’. The petitioner was shown to be a dangerous
criminal whose conduct showed that he was determined to lead a criminal life. The Regulations impose
reasonable restrictions on the Fundamental Rights of petitioner guaranteed in Article 21 and therefore,
they are valid.

Right to travel abroad – In Satwant Singh v/s Assistant Passport Officer, New Delhi. The Supreme Court
further extended the scope of this Article and held that the ‘right to travel abroad’ was part of a person’s
‘personal liberty’ within the meaning of Article 21 of the Constitution. In that case the petitioner who was
a citizen of India had to travel abroad frequently for business purposes. The Government ordered him to
surrender his passport. He challenged the action of the Government that it was violative of Article 21. The
contention of the Government was that the right to travel abroad was not included in the expressions
‘personal liberty’ and that a passport was a political document to which no one had a legal, much less a
Constitutional right. The Supreme Court held accepted the petitioners contention and held that the right
to travel abroad is part of a person’s ‘personal liberty’ within the meaning of Article 21 and therefore, no
person can be deprived of his right to travel abroad except according to the procedure established by law.
Article 21 takes in the right of locomotion – to go where and when one pleases, and the right to travel
abroad is included in it. It only excludes the ingredients of liberty mentioned in Article 19.

Maneka Gandhi Case – a New Dimension: In Maneka Gandhi v/s Union of India, the meaning and
content of the words ‘personal liberty’ again came up for the consideration of the Supreme Court. In that
case the Court has given the widest possible interpretation of the words ‘personal liberty’. The Supreme
Court held that the Government was not justified in withholding the reasons for impounding the passport
from the petitioner. Delivering the majority judgment. Bhagwati J. (as he then was) asked – Is the
prescription of some sort of procedure enough or must the procedure comply with any particular
requirement? He then added that the procedure contemplated in Article 21 could not be unfair or
unreasonable. And this principle of reasonableness which was an essential element of equality or non-
arbitrariness, pervaded Article 14 like a brooding omnipresence and the procedure contemplated in
Article 21 must answer the test of reasonableness in order to be in conformity with article 14. Hence, any
procedure which permitted impairment of individual’s right to go abroad without giving him a reasonable
opportunity to be heard could not be condemned as unfair and unjust. The order withholding reasons for
impounding the passport was therefore not only in breach of statutory provisions (Passport Act) but also
in violation of the rule of natural justice embodied in the maxim ‘audi alteram parteim’. Although there

CONSTITUTION | 48
are no positive words in the statue (Passport Act) requiring that the party shall be heard, yet the justice of
the Common Law will supply the omission of Legislature. The power conferred under Section 10(3)(c) of
the Act on the passport authority to impound a passport is a quasi-judicial power. The rules of natural
justice would therefore be applicable in the exercise of this power. Natural Justice is a great humanizing
principle intended to invest law with fairness and to secure justice. Fairness in action, therefore, demands
that an opportunity to be heard should be given to the person affected. A provision requiring of such
opportunity to the affected person can and should be read by implications in the Passport Act, 1967. If
such provisions were held to be incorporated in the Act by necessary implications, the procedure
prescribed for impounding passport would be right, fair, and just and would not suffer from the vice of
arbitrariness or unreasonableness.

It said that Article 21 requires the following conditions to be fulfilled before a person is deprived of the
property:
a. There must be a valid law.
b. The law must provide a procedure.
c. The procedure must be just, fair and reasonable.
d. The law must satisfy the requirements of Articles 14 and 17. i.e. it must be reasonable.

1. INTER-RELATIONS OF ARTICLES 14, 19, 21


(Old View) In Gopalan’s case the Supreme Court held that Article 19 has no application to laws
depriving a person of his life and personal liberty enacted under article 21 of the Constitution. It was
held that Articles 19 and 21 dealt with different subjects. Article 19 of the Constitution deals only
with certain (six freedom) important individual rights of personal liberty and the restriction that can
be imposed on them. Article 21 on the other hand enables the State to deprive individual of his life
and personal liberty in accordance with procedure established by law. Thus the view taken by the
majority in A. K. Gopalan’s case was that so long as a law of preventive detention satisfies the
requirement of Article 22, it would not be required to meet the challenges of article 19.

(Present view) in Maneka Gandhi v/s Union of India case the Supreme Court has overruled the view
expressed by the majority in Gopalan’s case was held that Article 21 is controlled by Article 19, that
is, it must satisfy the requirement of Article 19 also. The court observed: “The law must therefore
now be settled that Article 21 does not exclude Article 19 and that even if there is a law prescribing a
procedure for depriving a person of personal liberty and there is consequently no infringement of the
Fundamental Rights conferred by Article 21, such a law in so far as it abridges or take away any
Fundamental Rights under Article 19 would have to meet the challenges of that Article (Article 19).
Thus a law depriving a person of ‘personal liberty’ has not only to stand the test of Article 21 but it
must stand the test of Article 19 and article 14 of the Constitution”.

2. PROCEDURE ESTABLISHED BY LAW


(Old View) In Gopalan’s case the petitioner challenged the validity of his Preventive Detention under
Act on the ground that it was violative of his right to freedom of movement under Article 19 (1) (d)
which is the very essence of personal liberty guaranteed under Article 21 of the Constitution.
Secondly, it said that the Act was in conflict with Article 21 of the Constitution inasmuch as it
provided for deprivation of the personal liberty of a man not in accordance with a ‘procedure
established by law’. It was argued that the word ‘law’ in Article 21 should be understood not in the
sense of an enactment but as signifying the universal principles of natural justice and a law which did
not incorporate these principles could not be valid and

CONSTITUTION | 49
Thirdly that the expression ‘procedure established by law’ meant the same thing as the phrase ‘due
process of law’ in the American Constitution.

The Supreme Court rejected the aforesaid contention and held that the ‘procedure established by
law’ did not mean ‘due process of law’. There was no justification for adopting the meaning of the
word ‘law’ as interpreted by the Supreme Court of America in the expression ‘due process of law’
merely because the word ‘law’ is used in Article 21. This is clear from the report of the Drafting
Committee of the Constituent Assembly in respect of Article 21. The report of the Drafting
Committee shows that the Constituent Assembly had formerly used the American expression ‘due
process of law’ but the deliberately dropped it in favor of the expression ‘procedure established by
law’, which is more specific.

Again in Maneka Gandhi’s case the Supreme Court has overruled the A. K. Gopalan’s case and has
held that the mere prescription of some kind of procedure is not enough to comply with the
mandate of Article 21. The procedure prescribed by the law has to be fair, unjust and reasonable not
fanciful, oppressive or arbitrary, otherwise it should not be on procedure at all and all the
requirements of Article 21 would not be satisfied. What is fair or just? A procedure to be fair or just
must embody the principle of natural justice. Natural justice is intended to invest law with fairness
and to secure justice, the Court said, “law should be reasonable law, and not enacted piece of law”.

3. NATURAL JUSTICE
In Maneka Gandhi case the Supreme Court held that a procedure to be fair or just must embody the
principle of natural justice. Natural justice is intended to invest law with fairness and to secure
justice, the Court said, “law should be reasonable law, and not enacted piece of law”.

In Maneka Gandhi Case the Supreme Court gave a new dimension to Article 21. It held that the following
rights are to be covered under Article 21:
1. Right to live with human dignity
In Maneka Gandhi’s case the Court gave a new dimension to Article 21. It held that the right to ‘live’ is
not merely confined to physical existence but it includes within its ambit the right to live with human
dignity. Elaborating the same view the Court in Francis Coralie v/s Union Territory of Delhi, said that
the right to live is not restricted to merely animal existence. It means something more than just
physical survival. The right to ‘live’ is not confined to protection of any faculty or limb through which
life is enjoyed or the soul communicates with the outside world but it also includes ‘the right to live
with human dignity’, and all that goes along with it, namely the bare necessities of life such as,
adequate nutrition, clothing, shelter and facilities for reading, writing and expressing ourselves in
diverse forms, freely moving about and mixing and commingling with fellow human being.

The Supreme Court also held that non-payment of minimum wages to the workers employed in
various Asiad Projects in Delhi was a denial of them of their right to live with basic human dignity and
violative of Article 21 of the Constitution.

Other rights under Article 21:


2. Right to livelihood
3. No compulsion to wife to live with her husband whom she hates.
4. Right to shelter
5. Right to Privacy
6. Right to health and Medical Assistance

CONSTITUTION | 50
7. Right to sleep
8. Right to members of protective homes
9. No Right to die. No euthanasia
10. Right to get pollution free water and air
11. Protection of ecology and environment pollution
12. Freedom from Noise pollution
13. Right to anticipatory bail
14. Right of Prisoners under article 21
15. Right to free legal aid
16. Right against solitary confinement
17. Right to free and speedy trail
18. Right against inhuman treatment
19. Right of detenue to socialize with family and friends
20. Non-citizens also entitled to right to life
21. No compulsory blood test to determine paternity
22. Right against delayed execution
23. Protection against illegal arrest and detention and custodial death
24. Right to compensation to rape victims
25. Right of rehabilitation of sex workers.
26. Compensation for medical negligence
27. Right to education.

CONSTITUTION | 51
14. PREVENTIVE DETENTION LAWS AND CONSTITUTIONAL RIGHTS OF THE DETAINED PERSONS UNDER ARTICLE
22.
Ans Preventive Detention Laws
 Article 22, Clauses (4) and (7) provide the procedure which is to be followed if a person is arrested under
the law of ‘Preventive Detention’.
 The word preventive is opposite of Punitive. Punitive measure is to punish a person for what he has
already done, whereas the object of preventive detention is not to punish a person for having done
something but to intercept him before he does it and to prevent him from doing it.
 In this no offence is proved nor any charge is formulated. The sole justification of such detention is
suspicion or reasonable probability of the detenu committing some act likely to cause harm to the
society or endanger the security of the Government, and not criminal conviction which can only be
warranted by legal evidence.
 Necessity of Such Provision:
Preventive detention laws are not found in any democratic countries, In England it was resorted to in
time of war.
Indian Constitution recognizes preventive detention in normal times too. In A. K. Gopalan v/s State of
Madras Patanjali Shastri, J. explaining the necessity of this provision said: “this sinister looking feature,
so strangely out of place in democratic Constitution, which invests personal liberty with the
sacrosanctity of a fundamental right, and so incompatible with the promises of its Preamble, is
doubtless designed to prevent the abuse of freedom by anti-social and subversive elements which might
imperil the national welfare of the infant republic”.

There are four Central Legislative which are in force and which provide for preventive detention: They
are:
1. The National Security Act, 1980
2. Conservation of Foreign Exchange and Prevention of Smuggling Activities Act, 1974.
3. Prevention of Black-marketing and Maintenance of Supplies of Essential Commodities Act, 1980
4. Prevention of Illicitly Traffic in Narcotic Drugs and Psychotic Substance Act, 1988.

 The Preventive Detention Act: The Preventive Detention Act was enacted by the Parliament on 26th
February 1950. The object of the Act was to provide for detention with a view to preventing a person
from acting in a manner prejudicial to the defense of India, the relation of India with foreign powers, the
security of India or a State or the maintenance of public order, the maintenance of supplies and services
essential to the community. This Power is given to both the Centre and the States. This Act was only for
a year. Later it took various forms.
 In 1980 the President issued National Security Ordinance latter Act providing for preventive detention of
persons responsible for communal and caste riots and other activities prejudicial to the country’s
security. It provides for detention of 12 months but does not bar the detenu from challenging his
detention in a court of law on grounds, amongst others, of infringement of his Fundamental Rights. The
detenue will be convened the grounds of the detention within 10 days of his detention. He shall receive
the right to represent to the Advisory Board against his detention. It also provides for detention of a
person from acting in a manner prejudicial to the security of State or the maintenance of public order,
the maintenance of supplies and services essential to the community. Section 5A which was inserted by
the National Security Act 1984 stipulates that “where a person has been detained in pursuance of an
order of detention under section 3 which has been made on two or more grounds, such order or
detention shall be deemed to have been made separately on each of such grounds and accordingly –
“such order shall not be deemed to be invalid or inoperative merely because one of some of the

CONSTITUTION | 52
grounds is or are – vague, non-existent, non-relevant, not connected or not proximately connected with
such person or invalid of any other reason whatsoever.” This Act was later amended.
 In A. K. Roy v/s Union of India popularly known as the NSA case, the Supreme Court by 4:1 majority
upheld the Constitution validity of the NSA and the Ordinance which preceded the Act. The Court held
that Act was neither vague nor arbitrary in its provisions providing for detention of person on certain
grounds, as acting in a manner prejudicial to the ‘defence of India’, ‘security of India’, security of State’
and the relations with foreign power’. A number of guidelines were issued for safeguarding the interests
of the detenue detained under NSA.
1. That immediately after detention his kith and kin should be informed in writing about his detention
and his place of detention.
2. The detenu must be detained in a place where he habitually resides unless exceptional
circumstances require detention at some other place.
3. That detenu is entitled to his book and writing material, his own food, visits from family and friends
4. He must be kept separate from those convicted
5. No treatment of punitive character should be meted out to him and treated with dignity.
 Constitutional safeguards against preventive detention:
Article 22 Clauses (4) to (7) guarantee the following safeguards to a person arrested under Preventive
Detention.
1. Review by Advisory Board
2. Communication of grounds of detention to detenue
3. Detenue’s right of representation.

1. Review by Advisory Board


Prior to (44th Amendment) Act, 1978: Article 22, Clause 4 has been amended by the Constitution
(44th Amendment) Act, 1978. It provides that no law providing preventive detention shall authorize
the detention of a person for a longer period than ‘three months’ unless an Advisory Board
constituted of person who are or have been qualified to be High Court Judge has reported before
the expiry of the said period of three months that there is in its opinion sufficient cause for such
detention. If in the opinion of Advisory Board the detention was not justified the Government was
bound to revoke the detention order. If the Advisory Board reported that the detention was
justified then the detaining authority would determine the period of Detention. However the
period of detention could not be indefinite. It cannot exceed the maximum punishment prescribed
by law made by Parliament for that class of detenu under the sub clause (b) of Clause (7). Any law
providing for detention for a longer period of 3 months without obtaining the opinion of the
Advisory Board must provide the class or classes and also the circumstances under which a person
may be detained for more than 3 months. Clause (7) of Article 22 contained an exception (4). It
empowered Parliament to enact a law and provide for a detention for more than three months
without the opinion of the Advisory Board. Thus the opinion of the Advisory Board was not
necessary in the cases (1) when period of detention does not exceed three months and (2) when
Parliament by law prescribed the maximum period for which a person may be detained under
clause (4) (b) and clause (7) (a) (now deleted). The amendment is not yet in force.

After 44th Amendments Act, 1978 there have been changes.


 The maximum period of detention without obtaining opinion of Advisory Board was 2 months
rather than 3 months. If more than 2 months than take opinion of Advisory Board.
 The amendment changed the composition of the Advisory Board. The Advisory Board shall
now be constituted in accordance with the recommendation of the Chief Justice of the
appropriate High Court. It shall consist of a Chairman and not less than two other members.

CONSTITUTION | 53
The Chairman of the Advisory Board shall be sitting Judge of the appropriate High Court and
the other members shall be a sitting or retired judges of any High Court. It would be an
independent and impartial body i.e. free from executive control.
 It abolished the provision for preventive detention without reference to an Advisory Board as
provided in unamended sub-clause (a) of clause (7) of Article 22. Thus now a person can be
detained beyond the period of two months only after obtaining the opinion of the Advisory
Board.
 This amendment provided for two categories of detention: 1) detention for a maximum period
of two months under a law made by the legislature and 2) detention of a period longer than
two months provided the Advisory Board gives its opinion in favour of it. If Advisory Board says
that it is not justified then the person is to be released.
 The Advisory Board is bound to submit its report before the expiration of the said period of
two months. Failure to do so would render the detention illegal. In absence of Advisory
Board’s clear opinion that there was sufficient cause for the continued detention of the
detenu, it was held that their detention for more than one year was without legal sanction and
hence illegal.

2. Communication of grounds of detention to detenue.


Article 22 (5) give two rights to the detenu:
a. the authority making the order of detention must “as soon as may be” communicate to the
person detained the grounds of his arrest, that is the grounds which led to be subjective
satisfaction of the detaining authority and
b. to give the detenu “the earliest opportunity” of making a representation against the order of
detention, that is, to be furnished with sufficient particular to enable him to make a
representation.

The clause (5) imposes an obligation on the detaining authority to furnish to the detenu the
grounds for detention ‘as soon as possible’. The grounds of detention should be clear and easily
understandable by the detenu in the language which he understands. If it is only given orally and
nothing in writing is left with him in a language which he understands then that purpose is not
served and the Constitutional mandate in Article 22 (5) is infringed.

The grounds of detention must be in existence at the time of making the order. No part of such
ground can be held back nor can new ground be added thereto. In Shibban Lal v/s State of U.P. the
petitioner was supplied with two grounds of his detention. But later on the detaining authority
revoked one of the grounds. The detenu challenged the detention as illegal. The State contended
that the remaining ground was sufficient to sustain the detention. The court held the detention
illegal and observed. “To say that the other ground which still remains, is quite sufficient to sustain
the order, would be to substitute on objective judicial test for the subjective decision of the
executive authority which is against the legislative policy underlying the statute. In such cases, we
think, the position would be the same as if one of these grounds was irrelevant for purpose of the
Act, or was wholly illusory and this would vitiate the detention order ‘as a whole”.

The grounds supplied to the detenu must not be ‘vague’, ‘irrelevant’ or ‘non–existent’. If the
grounds are vague or irrelevant to the object of the legislation the right of the detenu under clasue
(5) is violated. A ground is irrelevant when it has no connection with the satisfaction of the
authority making the order of detention. In Ram Bahadur v/s State of Bihar it was held that where

CONSTITUTION | 54
the order of detention was based on distinct and separate grounds and if any of the grounds is
vague or irrelevant the entire order would fail.
In Fogla and S.K. Jalil v/s State of West Bengal where one of the reasons for detention was not
communicated to the detenu, it was held that the detenu had no opportunity to make an effective
representation to the Government and therefore the detention was violative of Article 22 (5) and
must be set aside.

There is no obligation to provide to detenue grounds of detention prior to his arrest. It is to be


given only after his arrest this is provided in Article 22 (5).

A person already in custody can be detained. In Huidrom Konnungjao Singh v/s State of Manipur
the son of the appellant was arrested by the Police under Section 302 IPC read with Section (1-C) of
the Arms Act. The District Magistrate, Imphal West passed the detention order under the NSA 1980
on various grounds, with an apprehension that the accused had been enlarged on bail. The detenue
in this case had not applied for bail. The Supreme Court held that there is no prohibition to pass the
detention order in respect of a person who is already in custody in respect of criminal case.
However if the detention order is challenged, the detaining authority has to satisfy the court, the
following facts:
1. the authority was fully aware of the fact that the detenu was in custody
2. There was reliable material before the said authority on the basis of which he would have
reason to believe that there was real possibility of his release on bail and further on being
released, he would probably indulge in activities which are prejudicial to public order.
3. In view of the above, the authority felt it necessary to prevent him from indulging in such
activities and therefore detention order was necessary.

3. Detenue’s right of representation.


There is no Constitutional mandate under clause (5) or Article 22 to consider the representation
made by the detenu before confirmation of detention order.

He is to be given an earliest opportunity after detention to make a representation against the


detention order. The detenu must be furnished with sufficient particular of ground of his detention
to enable him to make a representation which on being considered may give him relief.

The ‘grounds’ under Article 22 (5) means all the ‘basic facts’ and materials which have been taken
into account by the detaining authority in making the order of detention and on which, therefore,
the order of detention is based. Nothing less than all the basic facts and materials which influenced
the detaining authority in making the order of detention must be communicated to the detenu.

Secondly, the detenu shall be afforded the earliest opportunity of making representation against
the order of detention. No inordinate delay, no shortfall in the materials communicated shall stand
in the way of the detenu in making an earlier, yet comprehensive and effective representation in
regard to all basic facts and materials which may have influenced the detaining authority in making
the order of detention depriving him of his freedom.

The reasons why the grounds are required to be communicated ‘as soon as possible’ is twofold:
a. It acts to check against arbitrary and capricious exercise of power. The detaining authority may
not act according to their own whim and fancies and put any person under detention.
b. To help the detenu to make a representation as its earliest.

CONSTITUTION | 55
The ‘materials and documents’ relied on in the order of detention must be supplied to the detenu
along with the grounds. This would give him not a real but merely an illusory opportunity to make a
representation and would thus make the procedure unjust and unreasonable which would be liable
to be struck down. Inordinate delay in consideration the representation under Section 8 of N.S.A.
by the detaining authority will make detention order invalid. Similarly, the non-supply of the copies
of documents relied on by detaining authority will make the detention illegal.

The word “and shall afford” in Article 22 (5) cast a duty on detaining authority to inform the detenu
while serving the order or detention that he has a right to make representation against the order of
detention and also a right to be heard by the Advisory Board. This procedural safeguard must be
observed strictly and the failure to comply with this requirement would vitiate the order of
detention.

The government is bound to consider the petitioners representation as expeditiously as possible.


Delay would make detention illegal. Delay in considering the representation of the detenu must be
properly explained otherwise it will make the detention illegal. It has been held that where there is
unexplained long delay in disposal of representation the order of detention will be invalid. In T. D.
Abdul Rahman v/s State of Kerala the court held that the deal of 72 days in the absence of
satisfactory explanation was too long a period for ignoring the indolence on the part of the
concerned authority and hence the order of detention was invalid.

Similarly, it has been held that refusal on the part of the detaining authority to send the
representation of the detenue to the Central Government amounts to denial of the right conferred
on him under Article 22 (5), hence the order of detaining would be illegal and liable to be struck
down.

In Kamla v/s State of Maharashtra, the Supreme Court has expressed great concern about the
non-compliance of the Constitutional safeguards contained in Article 22 (5) by the detaining
authorities. Despite repeated warnings of the court in a series of decision, the detaining authorities
do not take care to comply these requirements. It the detaining authority is really seriously in
passing a valid order of detention there he should get Photostat copies of the documents and
materials which form the basis of detention and attach the same. If any detention is struck down by
court it means that the detaining authority has not applied his mind and he must be held personally
responsible and action should be taken against them for not complying with the Constitutional
requirements.

CONSTITUTION | 56
15. EXPLIN THE SAFEGUARDS PROVIDED BY THE CONSTITUTIONS TO PERSON ACCUSED OF CRIMES – ARTICLE 20?
Ans Article 20 of the Indian Constitution provides the following safeguards to the persons accused of crimes:
a. Ex Post Facto Law (Article 20, Clause 1).
b. Double Jeopardy (Article 20, Clause 2).
c. Prohibition against self-incrimination (Article 20, Clause 3).

a. Ex Post Facto Law (Article 20, Clause 1).


- An ex post facto law is a law which imposes penalties retrospectively i.e. on acts already done and
increases the penalty for such acts.
- But, Article 20 (1) provides that, “No person shall be convicted of any offence except for violation of a
law in force at the time of the commission of the act charged as an offence, nor be subjected to a
penalty greater than that which might have been inflicted under the law in force at the time of the
commission of the offence.”
- The legislative authority cannot make retrospective criminal laws, however, it does not prohibit
imposition of civil liability retrospectively i.e. with effect from a past date. So tax can be imposed
retrospectively.
- There are two parts to this law:
 No person shall be convicted of any offence except for violation of a law in force at the time of the
commission of the act charged as an offence. This means that if an act is not an offence at the date of
its commission it cannot be an offence at the date subsequent to its commission.
In Pareed Lubha v/s Nilambaram, it was held that if non-payment of the Panchayat Tax was not an
offence on the day it fell due, the defaulter could not be convicted for the omission to pay under a
law passed subsequently even if it covered older dues.
This Article cannot be claimed in case of preventive detention or demanding security from a person.
It is only for conviction and sentence and not for prosecution and trial under a retrospective law.
 A person cannot be subjected to a penalty greater than that which might have been inflicted under
the law in force at the time of the commission of the offence.
In Kedar Nath v/s State of West Bengal, the accused committed an offence in 1947, which under the
Act then in force was punishable by imprisonment and fine or both. The Act was amended in 1949
which enhanced the punishment for the same offence by an additional fine equivalent to the amount
of money procured by the accused through the offence. The Supreme Court held that the enhanced
punishment could not be applicable to the act committed by the accused in 1947 and hence set aside
the additional fine imposed by the amended Act.

Beneficial provisions: But the accused can take advantage of the beneficial provisions of the ex post
facto law. The rule of the beneficial construction requires that ex post facto law should be applied to
mitigate the rigorous (reduce sentence) of the previous law on the same subject.
In Ratanlal v/s State of Punjab, a boy of 16 years was convicted for committing an offence of house-
trespass and outraging the modesty of a girl age 7 years. The Magistrate sentenced him for 6 months
rigorous imprisonment and also fine. After the judgment, the Probation of Offenders Act 1958 came
into force. It provided that a person below 21 years of age should not ordinarily be sentenced to
imprisonment. The Supreme Court by a majority of 2:1 held that rule of beneficial interpretation
required that ex posto facto law could be applied to reduce the punishment.

a. Double Jeopardy (Article 20, Clause 2).


- Article 20(2) of the Constitution says that “no person shall be prosecuted and punished for the same
offence more than once”.

CONSTITUTION | 57
- This clause embodies the common law rule of nemo debet vis vexari which means that no man should
be put twice in peril for the same offence. If he is prosecuted again, then he can take the complete
defence of his former acquittal or conviction.
- In American and British Constitution the protection against double jeopardy is given for the second
prosecution for the same offence irrespective of whether an accused was acquitted or convicted in the
first trail. But under Article 20(2) the protection against double punishment is given only when the
accused has not only been ‘prosecuted’ but also ‘punished’ and is sought to be prosecuted second time
for the same offence. The use if the word ‘prosecution’ thus limits the scope of the protection under
clause (1) of Article 20. If there is not punishment for the offence as a result of the prosecution clause 2
of Article 20 has no application and an appeal against acquittal, if provided by the procedure is in
substance a continuance of the prosecution.
- The word ‘prosecution’ as used with the word ‘punishment’ embodies the following essentials for the
application of double jeopardy rule. They are:
1. The person must be accused of an ‘offence’. The word offence means any act or omission made
punishable by law for the time being in force.
2. The proceedings or the prosecution must have taken place before a court or judicial tribunal.
3. The person must have been ‘prosecuted and punished’ in the previous proceedings.
4. The offence must be the same for which he was prosecuted and punished in the previous
proceedings.
- In Maqbool Hussain v/s State of Bombay, the appellant brought some gold into the country. He did not
declare that he had brought gold with him to the customs on airport. The confiscated under the Sea and
Customs Act. He was later again charged with Foreign Exchange Regulation Act. The second prosecution
was in violation of Article 20(2) as it was the same offence. The court held that the Sea and Customs Act
did not constitute a judgment of judicial character necessary to take the plea of double jeopardy. Hence,
the prosecution under Foreign Exchange Regulation Act was allowed.
- Any proceedings before departmental and administrative authorities cannot be a proceeding of
judicial nature.
In Venkataraman v/s Union of India, the appellant was dismissed for service as a result of an inquiry
under the Public Service Enquiry Act, 1960, after the proceedings were held before the Enquiry
Commission. Later on he was prosecuted for having committed the offence under IPC and the
Prevention of Corruption Act. The court said that it did not amount to double jeopardy. The previous
was fact finding for disciplinary action the second was prosecution.

b. Prohibition against self-incrimination (Article 20, Clause 3).


- This article provides that no person accused of any offence shall be compelled to be a witness against
himself.
- No person should be compelled to give testimony which may expose him to prosecution for crime. This
is based on the fact that ‘he is presumed to be innocent till the contrary is proved’. It is the duty of the
prosecution to prove his guilt.
- Explaining the scope of this clause in M.P. Sharma v/s Satish Chandra, the Supreme Court observed that
this right embodies the following essentials:
1. It is right pertaining to a person who is ‘accused or an offence’.
2. It is a protection against ‘compulsion to be witness’.
3. It is a protection against such compulsion relating to his giving evidence ‘against himself’.

1. It is right pertaining to a person who is ‘accused or an offence’.


The words ‘accused of an offence’ make it clear that this right is only available to a person accused
of an offence. A person is said to be an accused person against whom a formal accusation relating

CONSTITUTION | 58
to the commission of an offence has been levelled which in normal course may result in his
prosecution and conviction. It is necessary that the actual trial or inquiry should have begun before
the court.

2. It is a protection against ‘compulsion to be witness’.


In M.P. Sharma v/s Satish Chandra, the Supreme Court interpreted the expression ‘to be a witness’
very widely so as to include oral, documentary and testimonial evidence. Thus the compulsory
taking of finger impression or specimen handwriting of an accused would come within the mischief
of Article 20(3).
But in State of Bombay v/s Kathi Kalu, the Supreme Court held that the interpretation of the
phrase ‘to be witness’ given in Sharma’s case was too broad and required a qualification. To be
witness is not equivalent to ‘furnishing evidence’. Self-incrimination can only mean conveying
information based upon personal knowledge of the person giving information and cannot include
merely the mechanical process of producing documents in court which may throw light on any
point in controversy, but which do not contain any statement of the accused based on his personal
knowledge. Thus giving finger print impressions or specimen writing is not included in the term ‘to
be witness’. They are materials for comparison.

3. It is a protection against such compulsion relating to his giving evidence ‘against himself’.
This protection is given to accused to give ‘against himself’ only. The person may give up this by
entering into the witness-box or by giving evidence on request. Request is not compulsion.
Compulsion can also mean duress which entails threatening, beating or imprisoning the wife,
parent or child of a person.
In Nandini Satpathy v/s P. L. Dani, the Supreme Court widened the scope of this article. The phase
‘compelled testimony’ must be read as evidence procured not merely by physical threats or
violence but by psychic (mental) torture, atmospheric pressure, environmental coercion, tiring
interrogatives, overbearing and intimidatory methods and the like.
In V.S. Kuttan Pillai v/s Ram Krishnan, the Court held that search of premises occupied or in
possession of person accused or seizure of anything from there was not violative of Article 20(3) of
the Constitution. If any document is recovered as a result of search and seizure it can be produced
before the court as an evidence against the accused as he is not compelled to give witness against
himself.

CONSTITUTION | 59
UNIT VI
16. WHAT IS DIRECTIVES PRINCIPLES OF STATE POLICY?
Ans The Directives Principles of State Policy are contained in Part IV of the Constitution. Articles 36 to 51.
 It sets out the aims and objectives to be taken up by the States in the governance of the country.
 It was borrowed from Ireland which had copied from Spanish Constitution
 The idea of welfare State can only be envisaged by our Constitution can only be achieved if the States
endeavor to implement them with a high sense of moral duty. The welfare State promotes the prosperity
and well-being of the people.
 The directive principles are the ideals with the Union and State Government must keep in mind while they
are formulating policy or passing a law. They lay down certain social, economic and political principles
suitable to peculiar conditions prevailing in India.
 They main objectives in enacting the directive principles appear to have been to set standards of
achievement before the legislature and the Executive, the local and other authorities, by which their
success of failure can be judged.

The directives are classified into the following groups:


a. Social and Economic Charter
b. Social Security Charter
c. Community Welfare Charter

A. SOCIAL AND ECONOMIC CHARTER:


Articles 38 and 39 embody Distributive Justice. It means the removal of economic equalities rectifying the
injustice resulting from dealings and transactions between unequals in society.
5. Social order based on Justice
 Article 38 (1) provides that the State strive to promote the welfare of the people by securing
and protecting as effectively as it may, a social order in which justice – social, economic and
political – shall inform all the institutions of national life. i,.e secure for the individual’s social,
economic and political justice.
 The Constitution (44th Amendment) Act, 1978 inserted a new directive principle in Article 38
which provides that the State, in particular, strive to minimize inequalities in income and
endeavor to eliminate inequalities in status, facilities and opportunities, not only for
individuals but also groups residing in different areas or engaged in different vocations. It aims
at equalities in all spheres of life.
 In Air India Statutory Corporation v/s United Labour Union a three judge bench of the Supreme
Court has explained the concept of social justice in Art 38 as: the concept of social justice
consists of diverse principles essential for the orderly growth and development of personality
of every citizen. Social Justice is the integral part of Justice in general. It is required to elevate
the poor, weak, dalits, tribals and deprived sections to live a life of dignity of persons.
 The Constitution therefore mandates the State to accord justice to all members of society in all
facets of human activity.

6. Principles of Policy to be followed by the State for securing economic justice


Article 39 specifically requires the State to direct its policy towards securing the following principles:
a. Equal rights of men and women to adequate means of livelihood.
b. Distribution of ownership and control of the material resources of the community to the
common good.

CONSTITUTION | 60
c. To ensure that the economic system should not result in concentration of wealth and means of
production to the common detriment.
d. Equal pay for equal work for both men and women.
e. To protect health and strength of workers and tender age of children and to ensure that they are
not forced by economic necessity to enter into work not suited to their age or strength.
f. Children given opportunities and facilities to develop in a healthy manner with freedom and
dignity, not exploited and protected against moral and material abandonment.

Material Resources: Article 39 (b). It covers land held by private owners also. Private Land can be
acquired by Government for public purpose such as developing, constructing homes, public
amenities, etc. PIL v/s Union of India: State legal owner of the Natural resources as a trustee of the
people. Distribution should be guided by Constitutional principles of equality and larger public good.
Spectrum belongs to the people – spectrum is natural resources that belongs to the people. State
and its instrumentalities are licensee – they hold it on behalf of people.

Equal Pay for equal work. Article 39 (d): This can be judicially enforceable by court. In Randhir Sing
v/s Union of India the Supreme Court declared that even though principle of equal pay for equal
work is not a fundamental right it is a Constitutional goal laid down in article 32. This cannot be a
fixed principle. It depends on reasonable classification. Different pay scales are awarded if difference
in nature of work, reliability and responsibility.

Employment of Children: Article 39 (f) (42nd Amendment) 1976. Child below the age of 14 is not to
be employed in hazardous works and mines. M. C. Mehta v/s State of Tamil Nadu – (child labour
abolished case). It laid down certain rules:
 Court directed for setting up of child labour rehabilitation welfare fund and asked the offending
employers to pay each child 20,000 and rehabilitate them in phased manner.
 Liability of the employer would not cease even if child is discharged from work. The government
to ensure that adult member of the child’s family gets a job in a factory or anywhere in lien of the
child.
 Where it would not be possible to provide jobs the appropriate government would as its
compensation, deposit Rs. 5000 in the fund of each child employed in the factory or mine or in
any other hazardous employment.
 In case adult works and the child needs alternative source of income interest then deposit 25,000
to the child’s family till he continues his study upto age of 14.
 The employment given is manual in nature then the nearest place of residence of the family be
given.
 Penal provisions contained in the 1986 Act will be used where the employment of a child labour
prohibited by the Act is found.
 The secretary of the Ministry of labour of the Union Government is directed to file an affidavit
within a month before the court about the compliance of the directions issued in this regard.

B. SOCIAL SECURITY CHARTER


1. Participation of workers in management of Industries (Art. 43) – State secure suitable legislature to
participation of workers in management of undertakings, establishment or other organizations
engaged in any industry.
2. Right to work, education, and public assistance in certain cases (Art. 41). State provide
employment, education, public assistance to the unemployed, old age, sick and weaker sections.

CONSTITUTION | 61
3. Just and Human conditions of work (Art. 42) - Especially maternity relief.
4. Living wage for workers (Art. 43) - directs that State ensure decent standard of living and full
enjoyment of leisure and social and cultural opportunities. It assures living wage and not minimum
wages.
5. Provision for early childhood care and education to children below 6 years (Art. 45) – In Unni
Krishnan v/s Andhra Pradesh directs right to education upto 14 years.
6. Duty to raise standard of living and improve health (Art. 47) – State raise the level of nutrition and
the standard of its people and improvement of public health. Prohibit intoxicating drinks and drugs
except to be used as medicinal purpose.
7. Promotion of educational and economic interest of weaker sections (Art. 46) – especially the
schedule castes and tribes and to protect them from social injustice and of all forms of exploitation.
8. Equal justice and free legal aid to economically backward classes (Art. 39 A) – directs the State to
ensure that the operation of the legal system promote justice on the basis of equal opportunity and
shall, in particular, provide free legal aid, see that justice is not denied to any citizen by reason of
economic or other disabilities. It also ensure speedy trial to all prisoners which is fundamental right
under Article 21 of the Constitution. Provide lawyer to the poor person and it must pay to the lawyer
his fee as fixed by the Court.
In Centre of Legal Research v/s State of Kerala it has been held that in order to achieve the
objectives of Article 39-A the State must encourage and support the participation of voluntary
organizations or social action groups in operating the legal aid programme.
In State of Maharashtra v/s Manubhai Pragaji Vashi the Supreme Court held that Art. 21 to be read
with Art. 39A casts a duty on the State to afford grant-in-aid to recognized private law colleges,
similar to other faculties, which qualify for receipt of grant. Provide equal justice and provide free
legal aid by suitable legislation or by schemes or in any other way.

C. SOCIAL SECURITY CHARTER


1. Uniform Civil Code (Art. 44) – requires the State to secure for the citizens a Uniform Civil Code
throughout the territory of India.

In historical judgment in Sarla Mudgal v/s Union of India the Supreme Court has directed the Prime
Minsters Narsimha Rao to take fresh look at Art. 44 of Constitution which enjoins the State to secure
a uniform civil code which, accordingly to the Court is imperative for both protection of the
oppressed and promotion of national unity and integrity. The Court directed the Union Government
through the Secretary to Ministry of Law and Justice, to file an affidavit by August 1995 indicating the
steps taken and efforts made, by the Government, towards securing a uniform civil code for the
citizens of India.

The above direction was given by the Court while dealing with case where the question for
consideration was whether a Hindu husband married under Hindu law, after conversion to Islam,
without dissolving the first marriage can solemnize a second marriage. The court held that it was
illegal and the husband can be prosecuted for bigamy under section 494 of IPC.

In this regard 4 PILs were filed: One by Registered Society working for welfare of women, Second by
Meena Mathur, Sunita alias Fatima and Gita Rani. The Court held that Hindu marriage continued to
exist even after one of the spouses converted to Islam. There is no automatic dissolution of Hindu
marriage. It can be dissolved by decree of divorce as provided in section 13 otherwise will be liable
for punishment under section 494 Bigamy.

CONSTITUTION | 62
The division bench (Kuldip Singh and R. M. Sahoi jj) said that the government failed to make effort to
promote Uniform Civil Code. Due to this many Hindus and become Muslims in order to get second
marriage and escape the consequences of Bigamy. Muslim law permits more than one wife to the
extent of four. Judge Kuldip said that there is no necessary connection between religion and personal
law in a civilized society. Marriage, succession and like matters are of a secular nature and therefore
they can be regulated by law. He pointed out that in America Polygamy is injurious to “Public
Morals”. This provided hope that one of the greatest evils of society will be removed, but the court in
one case clarified that its direction was only an obiter dicta and not legally binding on the
Government. This clarification was given by Kuldip Singh j himself.

In Noor Saba Khatoon v/s Mohd. Quasim, the Supreme Court has held that a divorced Muslim
woman is entitled to claim maintenance for her children till they become major. The court held that
both under the Muslim Personal law and under section 125 of the Criminal Procedure Code, 1973 the
obligation of the father was absolute when the children are living with the divorced wife. He has to
provide till they attain majority or are able to maintain themselves and in case of female, till they get
married. The Court also maintained that ‘we have opted for a secular republic, secularism under the
law means that the State does not owe loyalty to any particular religion and there is no State religion.
The Constitution gives equal freedom to all religions and everyone has the freedom to follow and
propagate his own religion, but the religion of individual or denomination has nothing to do in the
matter of socio-economic laws of the State.’. The freedom of religion under the Constitution does
not allow religion to infringe adversely on the secular right of the citizens and the power of the State
to regulate the socio-economic relations.

In John Vallamatton v/s Union of India a three Judge bench of the Supreme Court consisting of C.J V.
N. Khare S. B. Sinha and Dr. A. B. Lakshaman JJ. Has once again expressed regret for non-enactment
of Common Civil Code. In the instant case the petitioners have challenged the validity of section 118
of the Indian Succession Act on the ground that it was discriminatory under Art. 14 as well as
violative of Art. 25 and 26 of the Constitution. Section 118 of the Act imposed restriction on a
Christian having a nephew or a niece or any other relative as regards his power to bequeath his
property for religious or charitable purpose. The definition in the Act did not include wife of a
testator as near relative while an adopted son was included as a relative. So a Christian testator
having a nephew or niece must execute the will at least 12 months before his death and deposit it
within 6 months otherwise the bequest for religious or charitable use would be void. This restriction
did not apply to a person having a wife. The Court held that section 118 of the succession Act is
unconstitutional being violative of Article 14 of the Constitution. The Chief Justice of India in view of
the facts of this instant case forcefully reiterated the view that the Common Civil Code be enacted as
it would solve such problems. He said, ‘Article 44 is based on the premise that there is no necessary
connection between religion and personal law in a civilized society. Marriage, succession and like
matters are of a secular character cannot be brought within the guarantee enshrined under Articles
25 and 26 of the Constitution. It is a matter of regret that Article 44 of the Constitution has not been
given effect to. A Common Civil Code will enhance the cause of national integration by removing the
contradictions based on ideologies.

All Marriages must be registered: First step towards Uniform Civil Code – In a landmark Judgment
the Supreme Court has held that all marriages, irrespective of their religion be compulsorily
registered. Moved by the plight of women fighting for their rights under marriage like maintenance
and custody of their children, a bench of two judges directed the Centre and State Governments to
amend the law or frame rules and notify them within three months. The Court directed the

CONSTITUTION | 63
Government to provide for “consequences of non-registration of marriage” in the rules which should
be formulated after inviting public response and considering them. The Court felt that this was
necessary as unscrupulous husbands deny marriages leaving the spouses in the lurch, be it for
seeking maintenance, custody of children or inheritance of property. The benefits of this ruling were:
It will
a. Prevent child marriage
b. Check bigamy and polygamy
c. Help women to exercise their rights under marriage – maintenance, custody of children
d. Enable widows to claim inheritance
e. Deter husbands from deserting their wives.

2. Promotion of Co-operative Societies – By the Constitution 97th Amendment Act, 2011, Article 43B
has been inserted which provides – the State shall endeavor to promote voluntary formation,
autonomous functioning, democratic control and professional management of co-operative
Societies.

3. Organization of agriculture and animal husbandry – Article 48 directs the State to take steps to
organize agriculture and animal husbandry on modern and scientific lines. In particular, it should take
steps for preserving and improving the breeds and prohibiting the slaughter of cows and calves and
other milch and draught cattle.

4. Protection and improvement of forests and wild life - Article 48 A requires the State to take steps to
protect and improve the environment and to safeguard the forests and wild life of the country. In M.
C. Mehta (II) v/s Union of India, the Supreme Court relying on Article 48-A gave directions to the
Central and the State governments and various local bodies and Boards under the various statues to
take appropriate steps for the prevention and control of pollution of water.

5. Protection of Monuments and places and objects of national importance – Article 94 requires the
State to protect every monument or place or object fo artistic or historic interest (declared by or
under law made by Parliament), to be of national importance from spoliation, disfigurement,
destruction, removal, disposal or export.

6. Separation of Judiciary from Executive – Article 50 requires the State to take steps to separate the
Judiciary from the Executive in the public services of the State. To promote the rule of law.

7. Promotion of International peace and security - Article 51 provides that the State should strive to (a)
promote international peace and security, (b) maintain just and honorable relations between
nations, (c) foster respect for international law and treaty obligations in the dealings of organized
peoples with one another, and (d) encourage settlement of international disputes by arbitration.

In pursuant to the directions enshrined in Article 51 of the Constitution the Parliament has passed
“The protection of Human Rights Act, 1993. The Act provides for the setting of a National Human
Rights Commission and Human Rights Court to meet the growing concern for human rights in the
country and abroad.

8. Organization of Village Panchayats - Article 40 directs the State to take steps to organize village
panchayats and endow them with such powers and authority as may be necessary to enable them to

CONSTITUTION | 64
function as units of self-government. The object is to introduce democracy at the grass roots. They
are supposed to train people to develop democratic traditions.

DIRECTIVE PRINCIPLES OF STATE POLICY (IN SHORT)


State to secure a social order for the promotion of welfare of the people.

Certain principles of policy to be followed by the State.


a. that the citizens, men and women equally, have the right to an adequate means of livelihood;
b. that the ownership and control of the material resources of the community are so distributed as best to sub-serve
the common good;
c. that the operation of the economic system does not result in the concentration of wealth and means of
production to the common detriment;
d. that there is equal pay for equal work for both men and women;
e. that the health and strength of workers, men and women, and the tender age of children are not abused and that
citizens are not forced by economic necessity to enter avocations unsuited to their age or strength;
f. that children are given opportunities and facilities to develop in a healthy manner and in conditions of freedom
and dignity and that childhood and youth are protected against exploitation and against moral and material
abandonment.]

1. Equal justice and free legal aid. (Article 39A)


39. The State shall secure that the operation of the legal system promotes justice, on a basis of equal opportunity,
and shall, in particular, provide free legal aid, by suitable legislation or schemes or in any other way, to ensure
that opportunities for securing justice are not denied to any citizen by reason of economic or other disabilities.

2. Organization of village panchayats. (Article 40)


40. The State shall take steps to organize village panchayats and endow them with such powers and authority as
may be necessary to enable them to function as units of self-government.

3. Right to work, to education and to public assistance in certain cases. (Article 41)
41. The State shall, within the limits of its economic capacity and development, make effective provision for
securing the right to work, to education and to public assistance in cases of unemployment, old age, sickness and
disablement, and in other cases of undeserved want.

4. Provision for just and humane conditions of work and maternity relief. (Article 42)
42. The State shall make provision for securing just and humane conditions of work and for maternity relief.

5. Living wage, etc., for workers. (Article 43)


43. The State shall endeavor to secure, by suitable legislation or economic organization or in any other way, to all
workers, agricultural, industrial or otherwise, work, a living wage, conditions of work ensuring a decent standard
of life and full enjoyment of leisure and social and cultural opportunities and, in particular, the State shall
endeavor to promote cottage industries on an individual or co-operative basis in rural areas.

6. Participation of workers in management of industries. (Article 43A)


43A. The State shall take steps, by suitable legislation or in any other way, to secure the participation of workers
in the management of undertakings, establishments or other organizations engaged in any industry

7. Uniform civil code for the citizens. (Article 44)


44. The State shall endeavor to secure for the citizens a uniform civil code throughout the territory of India.

CONSTITUTION | 65
8. Provision for free and compulsory education for children. (Article 45)
[45. The State shall endeavor to provide, within a period of ten years from the commencement of this
Constitution, for free and compulsory education for all children until they complete the age of fourteen years.

9. Promotion of educational and economic interests of Scheduled Castes, Scheduled Tribes and other weaker
sections. (Article 46)
46. The State shall promote with special care the educational and economic interests of the weaker sections of
the people, and, in particular, of the Scheduled Castes and the Scheduled Tribes, and shall protect them from
social injustice and all forms of exploitation.

10. Duty of the State to raise the level of nutrition and the standard of living and to improve public health. (Article
47)
47. The State shall regard the raising of the level of nutrition and the standard of living of its people and the
improvement of public health as among its primary duties and, in particular, the State shall endeavor to bring
about prohibition of the consumption except for medicinal purposes of intoxicating drinks and of drugs which are
injurious to health.

11. Organization of agriculture and animal husbandry. (Article 48)


48. The State shall endeavor to organize agriculture and animal husbandry on modern and scientific lines and
shall, in particular, take steps for preserving and improving the breeds, and prohibiting the slaughter, of cows and
calves and other milch and draught cattle.

12. Protection and improvement of environment and safeguarding of forests and wild life. (Article 48 A)
48A. The State shall endeavor to protect and improve the environment and to safeguard the forests and wild life
of the country.

13. Protection of monuments and places and objects of national importance. (Article 49)
49. It shall be the obligation of the State to protect every monument or place or object of artistic or historic
interest, 2[declared by or under law made by Parliament] to be of national importance, from spoliation,
disfigurement, destruction, removal, disposal or export, as the case may be.

14. Separation of judiciary from executive. (Article 50)


The State shall take steps to separate the judiciary from the executive in the public services of the State.

15. Promotion of international peace and security. (Article 51)


The State shall endeavor to — (a) promote international peace and security; (b) maintain just and honorable
relations between nations; (c) foster respect for international law and treaty obligations in the dealings of
organized peoples with one another; and (d) encourage settlement of international disputes by arbitration.

CONSTITUTION | 66
17. DISCUSS IN DETAIL THE VARIOUS FUNDAMENTAL DUTIES OF THE CITIZEN UNDER THE INDIAN
CONSTITUTION?
Ans The fundamental duties of the citizens under the Constitution

 Article 51-A contains the fundamental duties of the citizens.


 This new Part IV-A which consists of only one Article 51-A was added to the Constitution by the 42nd
Amendment Act, 1976. This Article for the first time specifies a code of ten fundamentals duties for citizens.
They are:
1. to abide by the Constitution and respect its ideals and institutions, the National Flag and the National
Anthem;
2. to cherish and follow the noble ideals which inspired our national struggle for freedom;
3. to uphold and protect the sovereignty, unity and integrity of India;
4. to defend the country and render national service when called upon to do so;
5. to promote harmony and the spirit of common brotherhood amongst all the people of India
transcending religious, linguistic and regional or sectional diversities; to renounce practices derogatory
to the dignity of women;
6. to value and preserve the rich heritage of our composite culture;
7. to protect and improve the natural environment including forests, lakes, rivers and wild life, and to
have compassion for living creatures;
8. to develop the scientific temper, humanism and the spirit of inquiry and reform;
9. to safeguard public property and to abjure violence;
10. to strive towards excellence in all spheres of individual and collective activity so that the nation
constantly rises to higher levels of endeavor and achievement.

 The Constitution (86th Amendment) Act, 2002 has added a new clause (k) to Article 51-A which provides
(11) who is parent or guardian to provide opportunities for education to his child or as the case be, ward
between the age of six and fourteen years.

 Needs for fundamental duties: Rights and duties are correlative.


 The fundamental duties are intended to serve as a constant reminder to every citizen that while the
Constitutions specifically conferred on them certain fundamental rights, it also requires citizens to
observe certain basic norms of democratic conduct and democratic behavior.
 All are called to fulfill their duties which has been dictated in ancient scriptures as ‘Kartavya’. The
Gita and the Ramayana enjoin people to perform their duties without caring for their rights or fruits.
 No one can complain that Constitution has only rights and not duties. The fundamental rights are
not absolute. The State is empowered to impose reasonable restrictions and curtail these rights in
the interest of society.

 Source of fundamental duties:


 The fundamental duties are not found in any Constitutions of western countries. Among the
democratic Constitutions of the world, we find mention of certain duties of the citizens in the
Japanese Constitution.
 In Australia and Canada and Britain they are found in Common Law and judicial decisions.
 The Constitution of Socialist countries however lay great emphasis on the citizen’s duties. Article 32
of the Yugoslavian Constitution lays down, ‘the freedom and rights shall be achieved in solidarity
among the people by the fulfillment of their duties towards each other’.

CONSTITUTION | 67
 But among the socialist countries, the Soviet Constitution contains comprehensive Chapter on the
citizen’s duties. Chapter 7 of the Soviet Constitution lays down fundamental rights and duties. Art 66
provides that every citizen of the USSR are obliged to preserve and protect the Socialist property.
Any encroachment would be punished. Art 62 provides that defense of the Motherland is the sacred
duty of every citizen of the USSR and therefore military service compulsory and honorable duty.
Betrayal of the Motherland is the gravest of crimes against the people.
 China too lays down specific duties upon the people. Citizens have the right to work that is the right
to guaranteed employment and payment for the work in accordance with its quality and quantity.
 The right to work is still an important omission in Indian Constitution. Poverty is a curse.

 Enforcement of fundamental duties:


 The fundamental duties incorporated in the Constitution by the 42nd Amendments are statutory
duties and shall be enforceable by law. Parliament by law will provide penalties to be imposed for
failure to fulfill those duties and obligations.
 For proper enforcement of duties, it is necessary that it should be known to all. This should be done
by systematic and intensive education of the people that is by publicity or by making it a part of the
syllabus and curriculum of education.
 In M. C. Mehta (2) v/s Union of India the Supreme Court has held that under Art. 51-A (g) it is the
duty of the Central Government to introduce compulsory teaching of lessons at least for one hour in
a week on protection and improvement of natural environment in all the educational institution of
the country. Text books should be prepared and distributed free of cost. In order to arouse amongst
the people, the consciousness of cleanliness of environment, it suggested the desirability of
organizing keep the city clean week, keep the town clean, keep the village clean week in every city
throughout India at least once a year.

 Fundamental duties – an aid to interpretation of Constitutional provisions:


 In AIIMS Students Union v/s AIIMS speaking about the importance of fundamental duties enshrined
in Article 51-A while striking down the institutional reservation of 33% in AIIMS coupled with 50%
reservation discipline wise as violative of Article 14, the Supreme Court said that they are equally
important like fundamental rights. Though not enforceable like ‘fundamental’ rights, but they cannot
overlook as duties in Part IV A is prefixed by the same word ‘fundamental’. They provide a valuable
guide and aid to interpretation of Constitutional and legal issues in case of doubt. Constitutional
enactment of Fundamental duties if it has to have any meaning must be used by courts as a tool to
tap, even a taboo, on State action drifting away from Constitutional values, the judge declared.
 In Aruna Roy v/s Union of India, the validity of National Curriculum framework for School education
was challenged on the ground that it was violative of Art 28 of the Constitution and anti-secular. It
provides imparting of valued development education relating to basics of all religions. The Court held
that Art 51-A (e) which provides to promote harmony and the spirit of common brotherhood
amongst all transcending religious, linguistic and regional or sectional diversities; to renounce
practices against the dignity of women. Teaching of universal values such as truth, peace love, non-
violence are not violative and not against secularism.

CONSTITUTION | 68
18. EXPLAIN THE RELATIONSHIP BETWEEN DIRECTIVES PRINCIPLES OF STATE POLICY AND FUNDAMENTAL
RIGHTS?
Ans
 The directive principles differ from fundamental rights on the basis that Fundamental Rights are Justiciable,
whereas Directive principles are non-justiciable.

 According to Article 37, the Directive Principles, though they are fundamental to the governance of the
country and it shall be the duty of the State to apply these principles in making law, but they are expressly
made non-justiciable. On the other hand Fundamental Rights are enforceable by courts (Article 32) and the
courts are bound to declare as void a law that is inconsistent with the Fundamental Rights. Whereas, the
Directive Principles are not enforceable by courts nor can the courts declare as void any law which is
otherwise valid on the ground that it contravenes any of the directives.

 Dr. Ambedkar said that “The Directive Principles of State Policy is a new element for the Constitution of
parliamentary democracy”.

 The creators of the Indian Constitution, taking into consideration the object of ideal and prosperous
welfare State, have kept the entry in the Constitution for ‘directive principles’ of State policy. Whereas, the
fundamental rights, are the announcement if the Constitution and for the thorough development of the
citizens these rights are necessary.

 The directive principles are equal to the rights that are received by law. Unlike the directive principles the
fundamental rights have special legal importance.
In Madras v/s Champakam Dorairajan case the Supreme Court observed that fundamental rights are
sacrosanct and not liable to be abridged by legislative or executive or orders to the extent provided in the
appropriate Article in Part III. The directive Principles of State Policy have to conform and to run as
subsidiary to the Chapter on Fundamental rights. As long as there is no infringement of any fundamental
rights to the acting in accordance with the directive principles set out in Part IV, but subject again to the
legislative and executive powers and limitations conferred on the State under different provisions.

In case of conflict between fundamental rights and directive principles the fundamental rights would
prevail.

 Later in Re Kerala Education Bill, the Supreme Court observed that though the directive principles cannot
override the fundamental rights, nevertheless in determining the scope and ambit of fundamental rights
the court may not entirely ignore the directives principles but should adopt ‘the principles of harmonious
construction and should attempt to give effect to both as much as possible’. Therefore they should
supplement each other in attaining the goal. They are conscience of our Constitution said Granville Austin.

 In Kesavananda Bharati v/s State of Kerala the Supreme Court has said that fundamental rights and
directive principles aim at the same goal of bringing about a social revolution and establishment of welfare
State and they can be interpreted and applied together. They are supplementary and complimentary to
each other. The directives are the goal to be attained and the fundamental rights lay down the means by
which that goal is to be achieved.

 In Unni Krishnan v/s State of A. P, the Supreme Court has reiterated the same principle that the
“Fundamental Rights and directive principles are supplementary and complementary to each other and the

CONSTITUTION | 69
provisions of Part III should be interpreted having regard to the Preamble and Directive Principles fo the
State Policy”.

 For violation of the principles of State policy Courts cannot be resorted to, whereas for violation of
fundamental rights, courts can be resorted to.
Thought these directives are not enforceable by the courts, yet these principles have been declared to be
fundamental in the governance of the country (Art 37). It is the duty of the State to apply these principles in
making laws. If any Government ignores them they will certainly have to answer for them before the
electorate at the time of election. They are not merely moral precepts. This was made clear by Dr.
Ambedkar.

 In case of State of Tamil Nadu v/s L. Abu Kavur Bai a five bench of Supreme Court held that although the
directive principles are not enforceable yet the court should make a real attempt at harmonizing and
reconciling the directive principles and the fundamental rights and any collision between the two should be
avoided as far as possible.

 The directives show the path of moral duties to the State, but it cannot be considered to be the legal duty
of the State. It is the duty of the State to guarantee for the protection and implementation of the
fundamental rights. The protection of the fundamental rights is the legal duty of the Government.

 The parliament does not have the right to make changes in these principles and thus it has been given
special importance.

 Article 31-C and Directive Principles – Article 35-C added by Constitution 25th Amendment Act, 1971. The
amendment enhanced the importance of the directive principles. The object of the amendment was that
this was enacted to get over the difficulties placed in the way of giving effect to the directive principles of
State policy. The first part provides that no law which is intended to give effect to the directive principles
contained in Art 39 b, c shall be deemed to be void on the ground that it is inconsistent with or takes away
or abridges any of the rights conferred by Article 14 or Article 19. The second part provided that ‘no law
containing a declaration that is for giving effect to such policy can be called in question on the ground that
it does not in fact give effect to such policy (Invalid). This restored the primacy of the directive principles,
but not over the fundamental rights. The fundamental rights are justifiable rights and the directives are
non-justifiable rights.

 Directive Principles given status of Fundamental Rights. In a number of cases the Supreme Court has given
many directive principles of State Policy the status of fundamental rights. In Unnikrishnan v/s State of AP,
it was held that children from age of 6-14 have a fundamental right to free and compulsory education.
Similarly, equal pay for equal work has been held to be a fundamental right in Randhir Singh v/s Union of
India and therefore enforceable by law. Similarly, fair and speedy trial to prisoners too have been given
special status and can be enforced by law.

CONSTITUTION | 70
19. INTRODUCTION OF FUNDAMENTAL RIGHTS OF THE CITIZEN UNDER THE INDIAN CONSTITUTION AND NEW
TRENDS IN INTERPRETING PROVISION OF PART III: MANEKA GANDHI, SUNIL BATRA, HASKOT AND
HUSSAINNARA KHATOONS TREND?
Ans Introduction to the fundamental rights of the citizens:

1. Origin and development of Fundamental Rights:


 The fundamental rights are enshrined in Part III of the Constitution.
 It is also called as the Magna Carta of India. It is a written document. This was the first written document
relating to the fundamental rights of citizens. There after the king had to keep up the rights of his
subjects. In 1689 the Bill of Rights was written consolidating all important rights and liberties of English
people. In France declaration of rights of Man and the Citizens (1789) declared the natural, inalienable
and sacred rights of Man. Following this the Americans too incorporated the Bill of Rights in their
Constitution. They were the first to give it Constitutional status. Keeping this in mind the framers of the
Indian Constitution incorporated a full chapter in the Constitution. It is the most elaborate and
comprehensive than any other country.
 The aim was to declare certain elementary rights such as right to life, liberty, freedom of speech,
freedom of faith and so on as inviolabe under all conditions and the legislature of the Country should
not have a free hand in interfering with these fundamental rights.

2. Need for Fundamental Rights:


 Fundamental rights were deemed essential to protect the rights and liberties of the people against
encroachment of the power delegated by them to their Government.
 They are limitations upon all the powers of the Government, legislative as well as executive and they are
essential for the preservation of public and private rights, notwithstanding the representative character
of political instruments.
 Speaking of the important of the Fundamental rights in the historic judgment of Maneka Gandhi v/s
Union of India Bhagwati J. observed: These fundamental rights represent the basic values cherished by
the people of this country since the Vedic times and they are calculated to protect the dignity of the
individual and create conditions in which every human being can develop his personality to the fullest
extent. They impose obligations of the State not to encroach on individual liberty in its various
dimensions.
 They are regarded as fundamental because they are most essential for the attainment by the individual
or his full intellectual, moral and spiritual status. It is a reminder to the Government that they are to be
respected and not act according their own wishes.
 The main object is to establish ‘a Government of law and not of man’ a government system where the
tyranny of the majority does not oppress the minority. It is based on Rule of Law.
 It helps to provide security and equality and also provide certain standards of conduct, citizenship,
justice and fair play.
 These rights are not absolute. Absolute and unrestricted rights cannot exist in any modern society. It
becomes a license and jeopardize the liberty of other. This would ruin the nation create chaos and
anarchy. On the other hand if State had absolute power then it would lead to tyranny. Therefore the
Indian Constitution set limits to the liberties of people. This was reiterated in the case of A. K. Gopalan
v/s State of Madras.

CONSTITUTION | 71
3. New trends in interpreting provision of part III: Maneka Gandhi, Sunil Batra, Haskot and Hussainnara
Khatoons trend.
1. Widest Interpretation of Provisions of Part III – In Maneka Gandhi’s case the Supreme Court has held
that the provisions of Part III should be given widest possible interpretation. Bhagwati j said, ‘the
correct way of interpreting the provisions of Part III is that attempt of the court should be to expand
the reach and ambit of the fundamental rights rather than to attenuate (weaken, lessen) their meaning
and content’. Each article is not separate and mutually exclusive (as was told in Gopalan’s case A. K.
Gopalan v/s State of Madras) but they form a single scheme in the Constitution. The waters must mix
to constitute a grand flow of unimpeded and impartial justice. Isolation is not realistic nor beneficial but
would defeat the objects of such protection. Even if any aspect is not mentioned it should form the
integral part of the named fundamental right or partake of the same basic nature and character as that
fundamental right. Eg. Right to travel abroad, free legal aid, protection of prisoners from degrading
inhuman treatment thought not specifically mentioned, are fundamental rights under Article 21 of the
Constitution.
2. Natural Justice and due process – In Maneka Gandhi’s case the Supreme Court held that the
‘procedure’ depriving a person of his ‘life or personal liberty must be fair, just and reasonable’. It must
satisfy the requirement of natural Justice which is an essential component of fair procedure under Art.
21.
3. Prisoners right and Prison Reforms – the Supreme Court has considerably widened the scope of Article
21 and has held that its protection will be available for safeguarding the fundamental rights of
prisoners and for effecting prison reforms. They are also humans and until they are hanged they live in
prison as human beings and not slaves. Inhuman and barbarous treatment with solitary confinement,
hand-cuffing degrading jobs and punishment in jail with judicial approval violate the mandate of Article
21. Speedy trail and legal aid to poor prisoners are Constitutional rights.
4. Expanding role of writ of Habeas Corpus – the dynamic role of judicial remedies after Sunil Batra’s case
imparts to the habeas Corpus with a versatile vitality and operational utility as bastion of liberty even
within the jails. Whenever the rights of prisoners either under the Constitution or under other law are
violated the writ power of the Court can and should run to rescue. It is not only for releasing a person
from illegal detention but also for the good of the prisoners so that they are provided necessary
amenities, protected from inhuman and barbarous treatment.
5. Human rights Jurisprudence – in its decisions one finds extensive reference to the human rights by the
Supreme Court, particularly for protecting prisoners from various inhuman and barbarous treatment.
Krishna Iyer J. in Sunil Batra’s case says, ‘today human rights jurisprudence has Constitutional status. In
1979 India became party to the International Covenant on Civil and Political Rights. Article 10 of the
International Covenant provides that ‘all persons deprived of their liberty shall be treate3d with
humility and with respect for the inherent dignity of the human person. Art. 5 says that ‘no one shall
be subjected to torture or to cruel inhuman or degrading treatment or punishment’.

4. Suspension of Fundamental Rights


 The fundamental rights are not absolute.
 The Constitution provide for the curtailment or the suspension of the Fundamental Rights in the
following circumstances –
 Article 358 provides that when the proclamation of emergency is made by the President under
Article 352 the freedoms guaranteed by Article 19 are automatically suspended and continue to
do so for the period of emergency. Any order made by the State during this period cannot be
challenged on the ground that they are inconsistent with the rights guaranteed by Article 19.
Such laws shall, however, cease to have effect as soon as the proclamation ceases and then

CONSTITUTION | 72
Article 19 automatically revived and begins to operate. Things done during emergency can’t be
challenged even after the emergency.

5. Classification of Fundamental Rights:


The Fundamental Rights are classified under the following six groups:
2. Right to equality (Article 14-18)
3. Right to freedom (Article 19-22)
4. Right against exploitation (Article 23-24)
5. Right to freedom of religion (Article 25-28)
6. Cultural and educational Rights (Article 29-30)
7. Right to Constitutional remedies (Article 32-35)

CONSTITUTION | 73
20. CONSTITUTIONAL REMEDIES PROVIDED IN ARTICLE 32 TO 35.
WRITE SHORT NOTES ON WRITS, PIL, JUDICIAL ACTIVISM
Ans Dr. Ambedkar truly said, “If I was asked to name any particular Article in this Constitution as the most important
– an Article without which this Constitution would be a nullity – I could not refer to any other Article except this
one. It is the very soul of the Constitution and the very heart it.”
 It is true that declaration of fundamental rights is meaningless unless there is an effective machinery for the
enforcement of the rights. It is the remedy that makes the right real.
 Article 32 itself is a Fundamental Right. Article 226 also empowers all the High Courts to issue writs for the
enforcement of Fundamental Rights.
- Article 32 (1) guarantees the right to move the Supreme Court by ‘appropriate proceedings’ for the
enforcement of Fundamental Rights conferred by Part III of the Constitution. There is no limitation in
regard to the kind of proceedings, except that it should be ‘appropriate’. There is no particular form of
proceeding for the enforcement of Fundamental Rights.
- Article 32 (2) of Article 32 confers power on the Supreme Court to issue appropriate directions or
orders or writs, including writs in the nature of habeas corpus, mandamus, prohibition, quo-warranto
and certiorari for the enforcement of any of the rights conferred by Part III of the Constitution. The
writ jurisdiction of the High Court is wider than the writ jurisdiction of the Supreme Court. The
Supreme Court has power to issue writs only for the purpose of enforcement of Fundamental Rights
whereas the HC has power to issue writs for the purposes of enforcement of Fundamental Rights and
also other purpose.
- Article 32 (3) provides that Parliament may by law empower any other court to exercise within the
local limits of its jurisdiction all or of the powers exercisable by the Supreme Court under clause 2.
- Article 32 (4) says that the right guaranteed by Article 32 shall not be suspended except as otherwise
provided for the Constitution.
 PIL (PUBLIC INTEREST LITIGATION): Who can apply? Locus Standi – the traditional rule is that the right to
move the Supreme Court is only available to those whose Fundamental Rights are infringed. The power
vested in the Supreme Court can only be exercised for the enforcement of Fundamental Rights. The writ
under which the remedy is asked under Article 32 must be correlated to one of the Fundamental Rights
sought to be enforced.

But in PIL (Public Interest Litigation) the traditional rule of locus standi that Article 32 can only be filed by a
person whose Fundamental Rights is infringed has now been considerably relaxed by the Supreme Court in
its rulings.
In A.B.S.K. Sangh (Rly) v/s Union of India, it was held that the Akhil Bharitiya Soshit, Karmachari Sangh,
though an unregistered association could maintain a writ petition under Article 32 for the redressal of a
common grievance.
In the Judges Transfer case, a 7 member Bench of the Supreme Court has firmly established the rule
regarding the public interest litigation. The court held that any member of the public having ‘sufficient
interest’ can approach the court for enforcing Constitutional or legal rights of other persons and redressal
of a common grievance.
Abuse of PIL not to be allowed - Guidelines: while extending the Locus standi, Justice Bhagwati expressed
a note of caution. He said, “But we must be careful to see that the member of the public, who approaches
the court in case of this kind, is acting bona fide and not for personal gain or private profit or political
motivation or other oblique consideration. It should not be abused by politicians and others…”.
In Bihar Legal Support Society v/s Chief Justice of India, the court made it clear that the strategy of public
interst litigation has been evolved by this court with a view of bringing justice within the easy reach of the
poor and the disadvantaged sections of the community.

CONSTITUTION | 74
In M. C. Mehta v/s Union of India, the Supreme Court has further widened the scope of public litigations
under Article 32. Bhagwati J., speaking for the majority laid down the following guidelines:
1. The court held that the letter to any judge is to be approved. It need not be to the court only.
2. The court could grant remedial relief in the form of compensation appropriate in each case where the
Fundamental Rights of the poor are violated.
3. The court held that the court can appoint socio-legal commissions or devise any procedure and forge
any tools it deems appropriate for the enforcement of Fundamental Rights.
In State of Uttaranchal v/s Balwant Singh Chaufal, the Supreme Court has reiterated the details of the
origin and development of PIL and has laid down important guidelines for checking its misuse. The case was
PIL against the appointment of Attorney General as he was overage. The guidelines given were as follows:
1. The court must encourage genuine and bona fide PIL and effectively discourage and curb PIL for
extraneous consideration.
2. Instead of every individual judge to devise his own procedure of dealing with PIL the HC to properly
formulate rules for encouraging the genuine PIL and discouraging PIL filed for oblique (bad) reasons.
3. The courts should prima facie verify the credentials of the petitioner before entertaining a PIL.
4. The court should be prima facie satisfied regarding the correctness of the contents of the petition
before entertaining it.
5. The court should be fully satisfied that substantial public interest is involved before entertaining the
petition.
6. The court should ensure that the petition involves larger public interest, gravity and must be given
priority over other petitions.
7. The court before entertaining a PIL is aimed at redressal of genuine public term of public injury. The
court should also ensure that there is no personal gain, private motive or oblique motive behind filing
the PIL.
8. The court should also ensure that the petition filed by busy body for extraneous and ulterior motives
must be discouraged by imposing exemplary costs or by adopting similar novel methods to curb
frivolous petitions filed for extraneous considerations.
 Where directions should be issued? In Pravasi Bhalai Sangathan v/s Union of India, which was a PIL for
declaring that hate/derogatory speeches made by people politicians/religious leaders on religion, caste,
region and ethnic lines are violative of Articles 14,15,16,19,21, 38,51a, b,c,f,i, j of the Constitution. The
Court said that PIL could be resorted to only when there is inaction on the part of the executive for
whatever reasons, only then the courts have to step in. It can also take over if any action is arbitrary,
unreasonable, or otherwise in contravention of any statutory provisions or penal law. Secondly, the court
should not pass judicially unmanageable order which is incapable of enforcement.

 JUDICIAL ACTIVISM:
- Judiciary is considered to be the third important branch of the government wherein the authority of
the Supreme Court is holding great importance with reference to the maintenance of the
Constitutional rights of the citizens.
- It involves taking a positive action with a view to securing enforcement of the Fundamental Rights. The
judiciary has been assigned this active role under the Constitution. They are not expected to sit in an
ivory tower like an Olympian closing their eyes uncaring for the problems faced by the society. They
have to exercise their judicial powers for protecting the Fundamental Rights and liberties of citizens of
the country.
- This exercise of authority of the judiciary is not for vain glory but it is discharge of its Constitutional
obligation. When the executive and legislature are unconcerned and fail to discharge their
Constitutional obligations. The bureaucracy shows a total indifference and insensitivity to its

CONSTITUTION | 75
mandatory duties. This affects the basic rights of the people. When the law enforcing authorities show
their brutality in the process of implementation of law, the judiciary should check the excesses and
also direct the authorities to effectively implement the welfare legislature.
- The Supreme Courts role in scams cases, judgments ranging from the need for Uniform Civil Code,
pollution control, preservation of historical monuments, cleaning and keeping the big cities more
hygienic, directing removal of encroachments, interim compensation to victims of rape protecting
working women from harassment, punishing senior Karnataka IAS officer, puncturing the ego of Chief
Election Commissioner T. N. Seshan have attracted praise.
- In Hussainara Khatoon v/s State of Bihar, the Supreme Court has held that speedy trial is an essential
and integral part of Fundamental Rights to life and liberty. In Bihar a number of under trials were kept
in jail for several years without trial. The court ordered their names to be submitted to the court.
- Other achievements of Judicial Activism are:
Ban on smoking in public places hearing the PIL against smoking filed by Maharashtra Congress
Leader Murli Deora the court order all States to submit status report of action taken against cigarette
manufacturers violating advertising code. The banning of smoking in hospitals, health institutions,
public offices, public transports including railways, court buildings, educational institutions, libraries
and auditoriums.
Protection against inhuman treatment in Jail – In Sunil Batra v/s Delhi Administration, it was held
that the writ of habeas corpus can be issued not only for releasing a person from illegal detention but
also for protecting the prisoners from inhuman and barbarous treatment. The prisoners can file a writ
petition if his Fundamental Rights are violated.
Child Welfare – In Lakshmi Kant Pandey v/s Union of India, a writ petition was filed on the basis that
there were malpractices indulged by social and voluntary organizations regarding Indian children being
adopted by foreigners. They were exposed to long dreadful journey and if they survived they were not
provided shelter and relief homes and in the course of time they became beggars and prostitutes for
want of proper care. Justice Bhagwati laid down rules to be followed in adoption.
In Sheela Barse v/s Union of India, the court directed the central Government to pay to the petitioner
a social worker Rs. 10,000 for the expenses and to extend all necessary assistance who offered to
personally visit different parts of the country to verify whether the information submitted to the
authorities regarding children below the age of 18 years detained in jails was correct or not.
In M. C. Mehta v/s State of Tamil Nadu, it has been held that the children cannot be employed in
match factories which are directly connect with the manufacturing process as it was hazardous. They
can be employed in packing process but not exposed to accidents.
Protection of Ecology and Environment – In Rural Litigation and Entitlement Kendra v/s State of U.P,
the court ordered the closure of certain lime stone quarries on the ground that there were serious
deficiencies regarding safety and hazards in them. The court appointed a committee to inspect certain
lime stone-quarries. The committee suggested the closure of certain categories of stone quarries
having regard to them adversely affecting safety and health of the people living in the area.
Rape on working woman – In Delhi Domestic Working Women’s Forum v/s Union of India, a PIL was
filed suggesting that these working women were exposed to sexual assault by seven army personnel.
The incident happened in a train in Muri Express from Ranchi to Delhi. The court provided guidelines
with a view to assist rape victims. They are: That the complaints of sexual assault cases should be
provided with legal representation. The advocate explain the proceedings and guide her. Legal

CONSTITUTION | 76
assistance provided at the police station supported by lawyer. The police inform the victim of her right
to representation. Lawyer appointed by court if not available. Provide financial compensation.
Electoral Reforms – In Union of India v/s Association for Democratic Reforms, the petitioners for
Democratic Reforms filed and PIL and for the implementation of Law Commission’s recommendations
in its 170th report on March 2, 2002. The Supreme Court directed the Election Commission to issue a
notification making it compulsory for those contesting elections to make available information about
their educational qualification, assets, liabilities and criminal antecedents at the time of nomination for
benefit of voters. The Election commission did as instructed. The Parliament amended the electoral
law and negated the judgment and Election commissions’ notification. The Peoples Union for
Democratic Reforms filed a PIL in court challenging it. The Supreme Court held that the Parliament has
no legislative competence to direct the state authorities to disobey the order of the court and
therefore the amendment was violative of Fundamental Rights of voters to know their representatives
and hence invalid and restored it March 2, 2002 order.
Directions to make CBI independent and efficient – In Vineet Narian v/s Union of India, the Supreme
Court has issued directions to make the CBI independent agency so that it may function more
effectively and investigate crimes and corruption at high places in public life which poses a serious
threat to the integrity. The matter was brought as PIL under Article 32. It was held that these agencies
have failed to perform their duties and legal obligations inasmuch as they have failed to investigate
matters arising out of the seizure of the ‘Jain diaries’ and to prosecute all person s who were found to
have committed an offence.

CONSTITUTION | 77
21. WRITE SHORT NOTES ON JUDICIAL REVIEW, DOCTRINE OF SEVERABILITY, DOCTRINE OF ECLIPSE, DOCTRINE OF
WAIVER.
Ans MEANING AND BASIS OF JUDICIAL REVIEW:
 Judicial review is the power of courts to pronounce upon the Constitutionality of legislative act which
fall within their normal jurisdiction to enforce and power to refuse to enforce such as they find to be
unconstitutional and hence void.
 Justice Khanna has said that Judicial Review has become an integral part of our Constitutional system
and a power has been vested in the High Court and the Supreme Court to decide about the
Constitutional validity of the provisions of statutes. If the provisions of the statutes are found to be
violative of any of the articles of the Constitution which is the touchstone for the validity of all laws the
Supreme Court and the High Court are empowered to strike down the said provisions.
 Power corrupts and absolute power corrupts absolutely which ultimately leads to tyranny, anarchy and
chaos has been sufficiently established in human history. When Montesquieu gave his doctrine of
separation of powers, he was obviously moved by his desire to curb on absolute and uncontrollable
power in any one organ of the Government.
 Judicial review is thus the interposition of judicial restraint on the legislative as well as executive organs
of the Government.
 The concept of judicial review has the origin in the theory of limited Government
 This doctrine of judicial review was first time propounded by the Supreme Court of America in the
historic case of Marbury v/s Madison. The facts of the case are as follows: the Federalist had lost the
election of 1800, but before leaving the office they had succeeded in creating several new judicial posts.
Among these were 42 justices of peace, to which the retiring Federalists President John Adams
appointed forty-two Federalists. The appointment of commission were confirmed by the Senate and
they were signed and sealed, but Adam’s Secretary of State, John Marshal, failed to deliver certain of
them. When the President, Thomas Jefferson, assumed office, he instructed his Secretary of State,
James Madison, not to deliver seventeen of these commissions including one for William Marbury.
Marbury filled a petition in the Supreme Court for the issue of a writ of mandamus to Secretary Madison
ordering him to deliver the commissions. He relied on section 31 of the Judiciary Act of 1789 which
provided: The supreme Court shall have the power to issue…writs of mandamus, in cases warranted by
the principle and usages of law.. to persons holding office, under the authority of the United States. The
Court speaking through Marshall, who had now become Chief Justice, held that Section 13 of the
Judiciary Act was repugnant to Article III, Section 2 of the Constitution inasmuch as the Constitution
itself limited the Supreme Court’s original jurisdiction to cases, “affecting ambassadors, other public
minsters, consults and those to which a State is party”. Since Marbury fell in none of these categories
the court had no jurisdiction in his case.
 In the Indian Constitution there is an express provision for Judicial Review, and in this sense, it is no
more solid than it is in America. In the State of Madras v/s V. G. Row Patanjali Sastri CJ observed: “Our
Constitution contains express provision for Judicial Review of legislation as to its conformity with the
Constitution unlike in America where the Supreme Court has assumed extensive powers of reviewing
legislative acts under cover of the widely interpreted ‘due process’ clause in the Fifth and Fourteenth
Amendments. If the, the courts in India face up to such important and none too easy task, it is not out of
any desire to tilt at legislative authority and a crusaders spirit, but in discharge of duty plainly laid upon
them by the Constitution. But even without a provision for Judicial review eth courts would have been
able to invalidate a law which contravened any constitutional provisions, for, such power of judicial
review follows from the very nature of constitutional law.
 In A. K. Gopalan v/s State of Madras Kania CJ pointed out that it was only by way of abundant caution
that the framers of our Constitution, inserted the specific provisions in Article 13. He observed: In India,

CONSTITUTION | 78
it is the Constitution that is Supreme and that a statute law to be valid, must be in all conformity with
the Constitutional requirements and it is for the judiciary to decide whether any enactment is
constitutional or not.
 In Kesavananda Bharati case it has been held that Judicial Review is the ‘basic feature’ of the Indian
Constitution and therefore it ‘cannot be damaged or destroyed by amending the Constitution under
Article 368 of the Constitution.
 Again in L. Chandra Kumar v/s Union of India the Supreme Court has held that the power of Judicial
Review of legislative action as vested in the High Court under Article 226 and in the Supreme Court
under Article 32 is part of the basic structure of the Constitution and can’t be ousted or excluded even
by the Constitutional amendment.

DOCTRINE OF SEVERABILITY:
 When a part of the statute is declared unconstitutional then a question arises whether the whole of the
statute is to be declared void or only that part which is unconstitutional should be declared as such. To
resolve this problem, the Supreme Court has devised the doctrine of severability or separability.
 This doctrine means that fi an offending provision can be separated from that which is constitutional
then only that part which is offending is to be declared as void and not the entire statue.
 Article 13 of the Constitution uses the words ‘to the extent of such inconsistency be void’ which means
that when some provision of the law is held to be unconstitutional then only the repugnant provisions of
the law in question shall be treated by courts as void and not the whole statute.
 In A. K. Gopalan v/s State of Madras, the Supreme Court while declaring Section 14 of the Preventive
Detention act, 1950 as ultra vires (beyond the power or done without authority), observed “the
impugned Act minus this section can remain unaffected. The omission of the section will not change the
nature or the structure of the subject of the legislation. Therefore the decision that section 14 is ultra
vires does not affect eth validity of the rest of the Act.
 In State of Bombay v/s Balsara, a case under Bombay Prohibition Act, 1949, it was observed that the
provisions which have been declared as void do not affect the entire statute, therefore, there is no
necessity for declaring the statute as invalid.
 This his however subject to one exception. If the valid portion is so closely mixed up with invalid portion
that it cannot be separated without leaving an incomplete or more or less mingled remainder, then the
court will hold the entire Act, void. It must be seen that the remains are inextricably mixed and cannot
survive independently.
 The Doctrine of severability was elaborated in R.M.D.C. v/s Union of India. In that case, section 2 (d) of
the Prize Competition Act, which was broad enough to include competitions of a gambling nature as
well as competition involving skill, was involved. The Supreme Court held that the provisions of the Act
were several and struck down those provisions which related to competition not involving skill. In this
case it was held that where after removing the invalid provision what remains constitutes a complete
code there is no necessity to declare the whole Act invalid. But if what remains cannot be enforced
without making alteration the whole Act should be declared as void. Severability is the question of
substance and not of form, and in determining the intention of the legislature it is legitimate to take into
account the history of the legislation and the objects as well as the title and Preamble.
 In Kihota Hollohan v/s Zachithu it was held that Section 10 of the Tenth Schedule minus para 7 remains
valid and Constitutional. Para 7 which has been declared unconstitutional is severable from the main
provisions of the Tenth Schedule. The other part stand independent of para 7 and is complete in itself
and workable.

CONSTITUTION | 79
DOCTRINE OF ECLIPSE:
 The doctrine of eclipse is based on the principle that a law which violates Fundamental Rights is not
nullity or void ab initio but becomes only unenforceable i.e. remains in a moribund condition. “It is
overshadowed by the Fundamental Rights and remains dormant; but is not dead”.
 Such laws are not wiped out entirely from the statute book. They exist for all past transactions, and for
the enforcement of rights acquired and liabilities incurred before the present Constitution came into
force and for determination of right of persons who have not been given Fundamental Rights by the
Constitution i.e. non-citizens. It is only as against the citizens that they remain in a dormant or moribund
condition but they remain in operation as against non-citizens who are not entitled to Fundamental
Rights.
 Can such a law which becomes unenforceable after the Constitution came into force be again revived
and made effective by an amendment in the Constitution? To answer this question the Supreme Court
formulated the doctrine of eclipse in Bhikaji v/s State of MP. In that case provisions of C.P. and Berar
Motor Vehicles Amendment Act 1947 authorized the State Government to make up the entire motor
transport business in the Province to the exclusion of motor transport operators. This provision though
valid when enacted, became void on the common into force of the Constitution in 1950 as they violated
Article 19 (1)(g) of the Constitution. However in 1951 clause (6) of Article 19 was amended by the
Constitution (1st Amendment) Act, so as to authorize the Government to monopolize any business. The
Supreme Court held that ‘the effect of the Amendment was to remove the shadow and to make the
impugned Act free from all blemish or infirmity’. It became enforceable against citizens as well as non-
citizens aft her Constitutional impediment was removed. This law was merely eclipsed for the time being
by the fundamental rights. As soon as the eclipse is removed the law begins to operate from the date of
such removal. This law was merely eclipsed for the time being by the Fundamental Rights. As soon as
the eclipse is removed the law begins to operate from the date or such removal.
 Does the doctrine of eclipse apply to a post-constitutional law? – in Deep Chand v/s State of U.P., the
Supreme Court held that a post-constitutional law made under Article 13 (2) which contravenes a
Fundamental Rights is nullity from its inception and a still-born law. It is void ab initio. The doctrine of
eclipse does not apply to post-constitutional laws and therefore, a subsequent Constitutional
amendment cannot revive it.
 The Minority however expressed the view that the doctrine of eclipse is applicable even to post-
constitution law. In Mahendra Lal Jain v/s State of U.P. the Supreme Court approved the majority view
expressed in Deep Chand’s case and held that the doctrine of eclipse applies only to pre-constitution law
under Art. 13 (1) and not to post-constitution law under Article 13 (2). The voidness of the pre-
constitutional law is not from its inception but only from the date of the commencement of the
Constitution. On the other hand the voidness of a post-constitutional law is from its very inception and
such a law cannot therefore exist for any purpose.
 In State of Gujarat v/s Ambica Mills, the Supreme Court modified its view as expressed in Deep Chand
and Mahendra Las Jain’s case and held that a post-constitutional law which is inconsistent with
Fundamental Rights is not nullity or non-existent in all cases and for all purposes. The doctrine of
absolute nullity is not a universal rule and there are exceptions to it. A post-constitution law which takes
away or abridges the right conferred by Article 19 will be operative as regards to non-citizens because
fundamental rights are not available to them to non-citizens. Such a law will become void or non-
existent only against citizens because Fundamental Rights are conferred on them. Non-citizens can’t
take advantage of the voidness of the law.
 In Dulare Lodh v/s IIIrd Additional District Judge, Kanpur, the Supreme Court applied the doctrine of
Eclipse to post-constitutional law even against citizens. In that case the respondent-landlord filed a suit

CONSTITUTION | 80
for ejectment in 1971 and obtained a decree for ejectment against the appellant-tenant. By virtue of the
provisions of 1972 Act the case was transferred to the court of the Judge Small Causes, who tried the
case and passed a decree in favor of the respondent 1973. The landlord filed execution petition but his
attempt was stalled by objections of tenant or amendments in law with a result that even after 13 years
of litigation, the landlord was not able to get possession. This was because of the statutory amendment
which made the decree inexecutable and was therefore lying dormant and ineffective. But by the virtue
of Amendment Act of 1976, Section 9 of the U.P. Civil Laws (Amendment) Act of 1972 was made
applicable with retrospective effect which was enacted to remove the injustice and to remedy mischief
caused to the decree-holder. Its object was to revive the decrees which had become inexecutable. It was
held that by virtue of the Amendment Act, 1976 the degree which was lying dormant and could not be
executed became executable. Once the bar placed by the 1972 Act is removed by virtue of the doctrine
of eclipse will revive and become operative and executable.

DOCTRINE OF WAIVER:
 Can a citizen waive his Fundamental Rights? The doctrine of waiver has no application to the provisions
of law enshrined in Part III of the Constitution. It is not open to an accused person to waive or give up his
Constitutional rights and get convicted.
 The question of waiver directly arose in Basheshar Nath v/s Income Tax-commissioner. The petitioner
whose case was referred to the income-tax investigation commissioner under section 5 (1) of the Act
was found to have concealed large amount of income. He, thereupon, agreed at a settlement in 1954 to
pay Rs. 3 lakhs in monthly instalments by way of arrears to tax and penalty. In 1955 the Supreme Court
in Muthiah v/s IT Commissioner, held that section 5 (1) of the Taxation of Income (investigation
commission) Act was ultra vires of Article 14. The petitioner then challenged the settlement between
him and the Income Tax Investigation Commission. The respondent contended that even if Section 5 (1)
was invalid, the petitioner by entering into an agreement to pay the tax had waived his Fundamental
Right guaranteed under article 14.
 The Court held that it is not open to a citizen to waive any of the Fundamental Rights conferred by Part
III of the Constitution. These rights have been put in the Constitution not merely for the benefit of the
individual but as a matter of public policy for the benefit of the general public. It si an obligation
imposed upon the State by the Constitution. No person can relieve the State of this obligation, because
a large majority of our people are economically poor, educationally backward and politically not yet
conscious of their rights. In such circumstances it is the duty of this court to protect their rights against
themselves.

CONSTITUTION | 81
NOT PRESENT IN ANY UNIT
22. RIGHT TO EDUCATION A FUNDAMENTAL RIGHTS
Ans The right to education under Article 12A
 The Constitution (86th Amendment) Act, 2002 has added a new Article 21A after Article 21 and had made
education for all children of the age of 6 to 14 years a Fundamental Rights. It provides that “the State shall
provide free and compulsory education to all children of the age of 6 to 14 years in such a manner as the
State may, by law determine”.
 It is well know that education is a basic human right. For the success of democratic system of
Government, education is one of the basic elements. An educated citizen has to choose the
representatives who form the Government. Education gives a person human dignity who develops
himself as well as contributes to the development of his country.
 The framers of the Constitution have imposed a duty on the State under Article 45 as one of the directive
policy of State to provide free and compulsory education to all children until they complete the age of 14
years within 10 years from the commencement of the Constitution. The object was to eradicate illiteracy
from the county. But this was not followed by the politicians of our Country.

 Judicial Initiative –
In Mohini Jain v/s State of Karnataka, the matter was raised by the petitioners that the right of
education is a Fundamental Rights under Article 21 of the Constitution which cannot be denied to a
citizen by charging high fee known as capitation fee.

Importance of education was emphasized in the Neethi Shatamakam by Bhartiharitirsh Secretary in the
following orders, “Education is the special ministration of man. Education is the treasure which can be
preserved without the fear of loss. Education secures material pleasure, happiness and fame. Education is
God in carnate. Education secures honor of the hands of the state, not money. A man without education
is equal to animal. Education means knowledge and knowledge itself is power.”

The right to life under the Article 21 and the dignity of the individual cannot be realized unless it is
accompanied by the right to education. The Petitioners was Mrs. Mohini Jain who was denied admission
because of high capitation fee of Rs. 4 and half lakhs per annum in private medical college in Karnataka.
The Court held that right to education at all level is a Fundamental Rights under Article 21 of the
Constitution and charging high capitation fee for admission is illegal and amounted to denial of citizens’
right to education. The education in India has never been a commodity. However, the Court did not say
up to what age a citizens has right to education guaranteed by Constitution.

In Unni Krishnan v/s State of A.P., the matter was raised again, where the Court specifically held that the
right to education for the children of the age of 6 to 14 years is a Fundamental Right. The Court did not
agree with the decision of the Mohini Jain’s judgment that the children of all ages have the right to
education but held that the right to free education is available only to the children up to the age of 14 and
overruled the Mohini Jain’s case on the point. The Court has said that after the 14 years of age the
obligation of the State depended on the economic capacity and development. Article 21 A makes it
obligatory for the Government to enact a Central legislation to give effect to the Constitutional
amendment. The legislation will create a mechanism by which a citizen who is aggrieved that the right to
education has not been fulfilled should be able to get relief by filling a writ petition in the High Courts and
the Supreme Court.

CONSTITUTION | 82
Later in 2009, the Right of Children to Compulsory Education Act, 2009 was passed by the Parliament to
give effect to the 86th Amendments Act, 2002. It provides the responsibility of the Central and State
Government, teachers, parents and community members in ensuring that all children of the age of 6 to 14
years receive free and compulsory elementary education.

CONSTITUTION | 83
23. SHORT NOTES ON CITIZENSHIP
Ans Indian citizenship
 Meaning of Citizenship: The population of State is divided into two classes – citizens and aliens. The
citizens of a State is a person who enjoys full civil and political rights. They have certain advantages
conferred by the Constitution. Aliens do not enjoy these advantages.

The Following Fundamental Rights are available to the citizens:


1. The right not to be discriminated on grounds of religion, race, caste, sex or place of birth (A. 15)
2. The right of equality of opportunity in the matter of public employment (A. 16).
3. The right to six freedoms enumerated in Article 19 i.e. freedom of speech and expression, assembly,
association, movement, residence and profession.
4. Cultural and Educational rights conferred by Articles 29 and 30.
5. There are certain offices under the Constitution which can be occupied by citizens only i.e. office of
the President (Article 58 (1)(a), Vice President (Article 67 (3)(a), Judges of the Supreme Court (Article
124 (3) or of a High Court (Article 217 (2); Attorney General (Article 76(1), Governor of a State (Article
157), Advocate-General of a State (Article 165).
6. The right to vote for elections to the House of the People and the Legislative Assemblies of States is
available to the citizens only and only they can become members of the Union and State Legislatures.
 Constitutional Provisions: The Constitution does not lay down a permanent or comprehensive provision
relating to citizenship in India. Part II of the Constitution simply describes classes of persons who would be
deemed to be the citizens of India at the commencement of the Constitution, the 26th January 1950 and
leaves the entire law of the citizenship to be regulated by law made by Parliament. Article 11 expressly
confers power on the Parliament to make laws to provide for such matters.

Citizenship at the commencement of the Constitution i.e. January 26th 1950: the following persons under
Article 5 to 8 of the Constitution of India shall become citizens of India at the commencement of the
Constitution.
1. Citizens by domicile (Article 5)
2. Citizens of emigrants from Pakistan (Article 6)
3. Citizens of migrants to Pakistan (Article 7)
4. Citizens of Indians abroad (Article 8)

1. CITIZENS BY DOMICILE (ARTICLE 5)


According to Article 5 a person is entitled to citizenship by domicile if he fulfils the following two
conditions:
a) He must at the commencements of the Constitution have his domicile in the territory of India.
b) Such person should fulfill the following three conditions: 1) he was born in India; 2) either of
his parents was born in India, 3) he must have been ordinarily resident in the territory of India
for not less than five years immediately before the commencement of the Constitution.
Domicile is essential requirement for being Indian Citizen. Domicile of a person is in that country in
which he either has or is deemed by law to have his permanent house. Domicile is different from
Residence. To have a domicile the person should have residence and have the intention to make it
his permanent home.

There are two types of domicile i.e. domicile of origin and domicile of choice. Domicile or origin is by
birth. Domicile by choice is acquired by residence in territory subject to a distinctive legal system,
with the intention to reside there permanently. In Pradeep Jain v/s Union of India the Supreme
Court has held that in India Article 5 recognizes only one domicile i.e. domicile of India. It does not

CONSTITUTION | 84
recognize State Domicile. If a person goes from one state to another there is no need to acquire new
domicile by choice. India has one citizenship. It enumerated that there are two things necessary for
the existence of domicile: 1) a residence of a particular kind and 2) an intention of a particular kind.

The residence need not be continuous but it must be indefinite, not purely fleeing. The intention
must be a permanent intention to reside forever in the country where the residence has been taken
up. There must be both factum and animus to constitute the existence of domicile for neither
domicile nor mere residence is sufficient to make him an Indian citizen.

In Louis De Raedt v/s Union of India, the petitioners who were foreign nationals challenged the
order to the Central Government expelling them from India on their failure to acquire Indian
Citizenship. The petitioners came to India before Independence and were staying continuously on
the basis of foreign passport and residential permits. They were engaged in Christian Missionary
work. They contended that they became citizens of India by virtue of Article 5 (c) of the Constitution
as they were staying in India for more than 5 years immediately before the commencement of the
Constitution. The court held that they failed to establish that they had an intention to reside in India
permanently. The petitioners did not have their domicile in India. For the acquisition of a domicile of
choice, it must be shown that the person concerned had a certain state of mind, the animus
manendi. The intention to reside here permanently is very necessary for domicile.

2. CITIZENS OF EMIGRANTS FROM PAKISTAN (ARTICLE 6)


Persons who have migrated from Pakistan to India have been classified into two categories for the
purpose of citizenship: 1) Those who came to India before July 19, 1948 and 2) those who came on or
after July 19, 1948.
Those who migrated to India from Pakistan shall be deemed to be a citizen of India at the
commencement of the Constitution i.e. on 26th January, 1950 if he or either of his parents or any of
his grandparents were born in India as defined in the Government of India Act, 1953, and in addition
to above conditions, which applies in both cases fulfils one of the following two conditions:
a) In case he migrated to India before July 19, 1948 (the date on which the permit system for such
migration was introduced) he has been ordinarily residing in India since the date of his
migration; or
b) In case he migrated on or after July 19, 1948 he has been registered as citizen of India by an
officer appointed by the Government of India for the purpose.
Provided that no person shall be so registered unless he has been residing in India for at least six
months immediately before the date of his application for registration. If the above conditions
are satisfied a person shall be deemed to be a citizen of India.

3. CITIZENS OF MIGRANTS TO PAKISTAN (ARTICLE 7)


Under Article 7 a citizen by domicile (Article 5) or by Migration (Article 6) ceases to be a citizen if he
has migrated to Pakistan after March 1, 1947. An exception is, however made in favor of a person
who has returned to India on the basis of permit for resettlement in India. Such a person is entitled
to become a citizen of India if he fulfils other conditions necessary for immigrants from Pakistan,
after July 19, 1948 under Article 6. He can register himself as citizen of India in the same manner as a
person migrating from Pakistan after July 19, 1948.

Article 7 thus overrides Article 5 and 6. Both Article 6 and 7 use the term ‘migrated’. The meaning of
the term ‘migrated’ came for consideration before the Supreme Court Kulathi v/s State of Kerala.
According to the Court the term ‘migrated’ used in Article 6 and 7 has to be construed with reference

CONSTITUTION | 85
to the context and purpose and the prevailing political condition at the time the Constitution was
being enacted. So interpreted, the word ‘migrated’ could mean nothing except voluntarily going from
India to Pakistan permanently or temporarily. The majority held that the word ‘migrate’ was used in
a wider sense of moving from one country to another with the qualification that such movement was
not for a short visit or for a special purpose.

In State of Bihar v/s Kumar Amar Singh, one Kumar Rani who was admittedly born in the territory of
India and claimed to be the lawfully wedded wife of an Indian citizen whose domicile was India at all
material times, left India for Pakistan in July 1948, returned to India in December, 1948 on a
temporary permit and went back to Pakistan in April 1949. On May 14, 1953 she came back to India
under permanent permit obtained from High Commissioner for India in Pakistan, which was
cancelled on July 12, 1950, because it was wrongly issued without the concurrence of the
Government as required by the rules made under the Influx from Pakistan (Control) Act, 1949. She
contended first that she had never ceased to be a citizen of India because she was born in India and
her domicile was the domicile of her husband who was an Indian and consequently she was a citizen
of India. She contended, alternately that the proviso to Article 7 applied to her since she had
returned to India on a permanent permit and the subsequent cancellation of the permit was illegal
and irrelevant. It was held that ‘there could be no doubt that the lady must be held to have migrated
from the territory of India after 1st March 1947, although her husband stayed in India, but she could
not prove that she went to Pakistan for a temporary purpose’. The Supreme Court observed: “Article
7 clearly overrides Article 5. It is pre-emptory in its scope and makes no exception for such a case, i.e.
of the wife migrating to Pakistan leaving her husband in India”.

In Bhawanrao Khan v/s Union of India the Supreme Court has held that those who had voluntarily
migrated to Pakistan and became citizens of Pakistan cannot claim the citizenship of India on the
ground that they had been living in India for a long time and their names have been included in the
voters list. In this case, a lady Bhanwaroo Khan had claimed Indian Citizenship on the ground that she
had returned to India in 1955 and had been living here since then. The Court held that ‘a foreign lady
cannot claim Indian citizenship merely on the ground of long stay and inclusion of her name in the
voters’ list.

4. CITIZENS OF INDIANS ABROAD (ARTICLE 8)


Article 8 provides that any person or either of either of whose parents or any those grand-parents
was born in India as defined in the Government of India Act, 1935, and who is ordinarily residing in
any country outside India, shall be deemed to be a citizen of India as if he has been registered as a
citizens of India by the Diplomatic or Consular representatives of India in the country where he is for
the time being residing – on application made by him to such diplomatic or consular representative,
whether before or after 26th January, 1950 in the form and manner prescribed by the Government of
the Dominion of India or Government of India. Article 8 confers citizenship on Indian Nationals
residing abroad on their complying with its provisions.

Article 9 provides if a person’s voluntarily acquires the citizenship of any foreign State he shall not be
able to claim citizenship of India under Article 5, 6, and 8. It deals only with voluntary acquisition of
citizenship of a foreign State before the Constitution came into force. Cases of voluntary acquisition
of a foreign citizenship before the commencement of the Constitution will have to be dealt with the
Government of India under the Citizenship Act of 1955.

CONSTITUTION | 86
Citizenship under the Citizenship Act, 1955 – Parliament in exercise of the power given to it under
Article 11 of the Constitution, has passed the Citizenship Act, 1955, making provisions for acquisition
and termination of citizenship after the commencement of the Constitution. The Act provides for the
acquisition of Indian citizenship after the commencement of the Constitution in five ways i.e. birth,
descent, registration, naturalization and incorporation of territory.

Amendment of Citizenship Act – The Citizenship (Amendment) Act, 2003 has almost overhauled the
Citizenship Act, 1955. Its main aim is to provide double citizenship to ‘overseas citizenship of India’ of
specified countries mentioned in the new Fourth to the Act added by the amendment. The Act has
substituted new clauses (b) and (c) and proviso to (c) of Section 2 which says that ‘illegal migrants
means who entered into India – i) without valid passport or other travel documents as provided by
any law ii) and remained therein beyond the permitted period of time.

CONSTITUTION | 87
24. OVERSEAS CITIZENS OF INDIA
Ans
 Meaning of Overseas Citizenship: The new clause (ee) of Section 2 says that overseas citizen of India
means a person who i) is of Indian origin being citizen of a specified. ii) was a citizen of India immediately
before becoming a citizen specified country and is registered as an overseas citizen of India by the Central
Government under sub-section (1) of Section 7A. New Clause (gg) says that ‘specified country’ means a
country specified in the Fourth Schedule. The Fourth schedule mentions the following countries –
Australia, Canada, Finland, France, Greece, Ireland, Israel, Italy, Netherland, New Zealand, Portugal,
Republic of Cyprus, Sweden, Switzerland, United Kingdom and United States of America.
 Citizenship by Birth (Section 3) – The new Section 3 for the exiting Section 3 provides that except as
provided in sub-section (2) every person born in India (a) on or after 26th January, 1950 but before 1st July
1987 (b) on or after 1st July, 1987 but before the commencement of the Citizenship (Amendment) Act,
2003 and either of whose parents is a citizen of India at the time of his birth (c) on or after the
commencement of the Citizenship (Amendment) Act, 2003 where (i) both of his parents are citizens of
India or (ii) one of whose parents is a citizen of India and other is not an illegal migrant at the time of his
birth, shall be a citizen of India by birth.

 Citizenship by descent (Section 4) – According to new sub-section (1) of Section 4 – A person born outside
India shall be citizen of India by descent – (a) on or after 26th January 1950 but before 10th 1992 if his
father is a citizen of India the time of his birth, or (b) on or after 10th day of December 1992 if either of his
parents is a citizens of India at the time of his birth.
Provided that if the father of a person referred to in clause (a) was a citizen of India by descent only, that
person shall not be a citizen of India by virtue of this section unless (a) his birth is registered at the Indian
Consulate within one year of its occurrence or the commencement of this Act, whichever is later, or with
the permission of the Central Government, after the expiry of the said period, or (b) either of his parents
is a the time of his birth in service under Government in India.
Provided also that on or after the commencement of the Citizens (Amendment) Act, 2003 a person shall
not be a citizen of India by virtue of this section unless his birth is registered at an Indian Consulate in
such form and manner as may be prescribed (i) within one year of its occurrence or the commencement
of the Citizenship (Amendment) Act, 2003 whichever is later or (ii) with the permission of the Central
Government, after the expiry of the said period.
But no such birth shall be registered unless the parents of such parents declare that the minor does not
hold the passport of another country. A minor who is citizen of India by virtue of this section and is also
citizen of India if he does not renounce the citizenship of another country within six months of his
attaining his full age.

 Citizenship by Registration (Section 5) – The prescribed authority may on application register a citizen of
India any person not being an illegal migrant who is not already such citizen by virtue of the Constitution
or of any other provisions of this Act if he belongs to any of the following categories –
b. A person of Indian origin who is ordinary resident of India for seven years before making an
application for registration.
c. Persons of Indian origin who is ordinarily resident in any country or place outside undivided India.
d. Person who is married to a citizen of India and is ordinarily resident of India for seven years before
making an application for registration.
e. Minor children of persons who are citizens of India.

CONSTITUTION | 88
f. A person of full age and capacity whose parents are registered as citizens of India under Clause (a) of
this sub-section or sub-section (1) of Section 6 (f) a person of full age and capacity who either of his
parents, was earlier citizen of Independent India and has been residing in India for one year
immediately before making an application for registration (g) a person of full age and capacity who
has been registered as an overseas citizen of India for five years, and who has been residing in India
for two years before making an application for registration.
For purpose of Clause (a) and (c) an applicant shall be deemed to be ordinarily resident of India (i) if he
has resided in India for 12 months immediately before making an application for registration (ii) he has
resided in India during 8 years immediately preceding the said period of 12 months for a period of not less
than 6 years.

 Citizenship by Naturalization – Where an application in the prescribed manner is made by any persons of
full age and capacity, ‘not being an illegal migrant’ the Central Government may, if satisfied that the
applicant is qualified for naturalization, grant him a certificate of naturalization. The qualification for
naturalization are the following:
a. He must not be a citizen of a country where Indian citizens are prevented from becoming citizens by
naturalization.
b. He has renounced the citizenship of the other country.
c. He has either resided in India or has been in Government service for 12 months before the date of
making the application for naturalization, or during 7 years prior to these 12 months, he has resided
or has been in the Government service for not less than four years.
d. He must take an oath of allegiance.
e. He is of good character.
f. He has an adequate knowledge of a language recognized by the Constitution.
g. That after naturalization being granted to him, he intends to reside in India or to serve under the
Government of India. However, if in the opinion of the Central Government the applicant has
rendered distinguished service to the cause of science, philosophy, art, literature, world peace or
human progress, it may waive all or any of the above conditions for naturalization in his case (Section
6).

 Citizenship by incorporation of territory – if any new territory becomes a part of India, the Government
of India shall specify the persons of the territory to be citizens of India (Section 7).

Overseas Citizen (New Sections 7-A, 7-B, 7-C, 7-D added by the Amendment Act, 2003)
 Registration of overseas citizenship (Section 7-A):
1. when the application is made in this behalf, the Central Government may, subject to conditions and
restrictions including the condition of reciprocity as may be prescribed, register any person as an
overseas citizen of India if –
a. That person is on Indian Origin, of full age and capacity who is a citizen of a specified country,
b. That person is of full age and capacity who has obtained the citizenship of a specified country on
or after the commencement of the citizenship (Amendment) Act, 2003 and who was a citizen of
India immediately before such commencement, or
c. That person is a minor of a person mentioned in clause (a) or clause (b)
2. The person registered as an overseas citizen of India under sub-section (1) shall be an overseas
citizen of India as from the date on which he is registered.
3. No person who has been deprived of his citizenship under this Act shall be registered as an overseas
citizen of India under sub-section (1) except by an order of the Central Government.

CONSTITUTION | 89
 Conferment of rights on overseas citizens of India (Section 7 – B) – An overseas citizens of India shall be
entitled to such right notwithstanding anything in any law for the time being in force, (other than the
rights specified under sub-section (2), as the Central Government may by official gazette specify in this
behalf.

An overseas citizen of India shall not be entitled to the rights conferred on a citizen of India –
a. Under Article 16 with regard to equality of opportunity in matters of public employment,
b. Under Article 58 for election as President
c. Under Article 66 for election of vice-president
d. Under Article 124 and 217 for appointment of Judges of the Supreme Court and High Courts,
e. Under section 3 and 4 of Representation of Peoples Act, 1951 for being members of the House of
People or the Council of States
f. Under Sections 5, 5-A and 6 of the Representation of Peoples Act, 1951 to be eligible for being
member of the Legislative Assembly or a Legislative Council of States they cannot be eligible for
appointment to public services and posts under Union or States except for appointment for such
services and posts as the Central Government may specify by order. Every such notification must be
laid before the Parliament. (Section 7B (1) and (2)).

 Renunciation of overseas citizenship (Section 7 – C) – If any overseas citizen of India of full age and
capacity makes in the prescribed manner a declaration renouncing his overseas Citizen of India, the
declaration shall be registered by the Central Government and upon such registration, that person shall
cease to be an overseas citizen of India. Where such person ceases to be an overseas citizen of India every
minor children of that person shall thereupon cease to be an overseas citizen of India.

 Cancellation of registration as overseas citizen of India. (Section 7 – D) – The Central Government may
by order cancel the registration if it is satisfied that (a) the registration as an overseas citizen of India was
obtained by means of fraud, false representation or the concealment of any material fact, or (b) the
overseas citizen of India has shown disaffection towards the Constitution of India, or (c) the overseas
citizen of India has during any war in which in India may be engaged unlawfully traded or communicated
with an enemy or been engaged in or association with any business or commercial activities that was to
his knowledge carried on in such manner as to assist an enemy in that war, (d) he has within 5 years after
his registration been sentenced to imprisonment for a term of not less than two years, or (e) it is
necessary so do in the interest of the sovereignty any integrity of India, the security of India, friendly
relationship of India with any foreign country or in the interest of general public. The Amendment has
omitted Section 11 and 12 of the main Act. Section 14 has been amended and for Sections 5, 6 and 6A
have been substituted.

 Issue of National Identify Cards (New Section 14 – A) – This section provides that the Central
Government may compulsory register every citizen of India and issue national identity card to him. The
Central Government may maintain a National Register of Indian citizen and for that purpose establish a
National Registration Authority. From the commencement of the Citizen (Amendment) Act, 2003 the
Registrar General of India appointed under sub-section (1) of Section 3 of Registration of Births and
Deaths Act, 1969 shall act as the National Registration Authority and he shall function as the Registration
General of Citizen Registration. The Central Government may appoint other staff to assist the Registrar
General of Citizen Registration in discharging his functions. The procedure to be followed for compulsory
registration of Citizen of India shall be prescribed.

CONSTITUTION | 90
 Termination of Citizen – The Citizen Act, 1955, also lays down how the citizenship of India may be lost
whether it was acquired under the Citizenship Act, 1955 or prior to it, under the provisions of the
Constitution. It may happen in any of the three ways:

(a) Renunciation of Citizenship: An Indian citizen of full age and capacity can renounce his Indian
citizenship by making a declaration to that effect and having it registered.
(b) Termination of Citizenship: If a citizen of India voluntarily acquires the citizenship of another country
he shall cease to be a citizen of India.
(c) Deprivation: Deprivation is a compulsory termination of the citizenship of India. A citizen of India can
be deprived if he has got registered or got naturalization through fraud, false representation or
concealment of any material facts. If he has acted or spoken to be disloyal of shown dissatisfaction
towards the Constitution. Unlawfully communicated or traded with enemy during war. Within five
years of naturalization has been imprisoned for not less than two years.

CONSTITUTION | 91

You might also like